You are on page 1of 142

KEVAN AKRAMI MD, LAUREN EDWARDS MD,

MONIQUE MAHLUMMD AND EB SLADEK MD


i
It is impossible to begin to learn that which one thinks one already knows.
- Epictetus
The editors and authors of this handbook would like to thank the following faculty and staff for
their assistance with this publication:
Derek Patel MD Ronelle Campbell MD Lori Wan MD
Peter Fedullo MD Joshua Fierer MD Timothy Morris MD
Melly Sani MD Hiroke Shike MD Thomas Lane MD
C. Ann Reardon MD Dzung T. Le MD Lane Campbell
Eliza Wang PharmD Gregory Maynard MD Maria Stubbs, RPh.
Brian Clay MD Ian Jenkins MD Sundar Mudaliar MD
Michael Docherty MD Elaine Muchmore MD Kalli Hose MD
Gregory Seymann MD Daniel Davis MD
Previous Editors: Carrie Chun MD (2004-2005), Keola Beale MD (2005-2006), Carrie Constantini
MD (2006-2007), Ni-Cheng Liang MD (2007-2008), Erica Boettcher MD (2008-2009), Jonathan
Lyons and Tricia Santos (2009-2010), Matthew Crull and Emmet Ratigan (2010-2011)
ii
PAPERWORK, COMPUTERS, AND DICTATION 1
NATALIE GARCIA MD AND CHHAVI GANDHI MD
QUICK REFERENCE GUIDE - VA 1
QUICK REFERENCE GUIDE - UCSD 3
COMPUTING TIPS 4
WEBSITES 6
DISCHARGE PROCEDURES 7
VA FORMULARY 8
DEATH PROCEDURE 8
TRANSFERS AND DISCHARGES 9
GRADUATION REQUIREMENTS 10
PULMONARY CRITICAL CARE 12
PATRICK LEWMD, DEBBY NGO MD, MARISA MAGANA MD, REBECCA SELL MD, THOMAS KIM MD, JOSHUA STERNBACH MD, CAROLINE SUL PHARMD, ANDJENNIFER REAL MD
MECHANICAL VENTILATION 12
VENT TERMS & DEFINITIONS 12
COMMON MODES OF VENTILATION 13
HIGH AIRWAY PRESSURES 15
LIBERATION 16
NON-INVASIVE POSITIVE PRESSURE VENTILATION (NIPPV) 16
ARDS AND ACUTE LUNG INJURY 18
HYPOXIA 20
SHORTNESS OF BREATH 21
ANAPHYLAXIS 21
SEPSIS 22
EARLY GOAL DIRECTED THERAPY 23
CORTICOSTEROIDS 25
ACTIVATED PROTEIN C 25
PRESSORS 26
PARALYTICS 27
SEDATIVES 28
SEVERE METABOLIC ACIDOSIS 29
VENT TROUBLESHOOTING 30
PULMONARY FUNCTION TESTING 31
COPD 32
CYSTIC FIBROSIS 33
CODE ALGORITHMS 35
DANIEL DAVIS MD
PERFUSING PATIENT 35
ARRESTED PATIENT 36
CARDIOLOGY 37
Derek Juang MD, Leo Slavin MD, Ramin Shadman MD, Jonathan Lyons MD, Thomas Ro MD, Cynthia Kao MD, Gautam Lalani MD, Charles
Parise MD, Mitul Patel MD, Andrew Dublin MD and Hong Nguyen MD
CHEST PAIN AND ACUTE CORONARY SYNDROME 37
EKGS 38
CHF 40
CARDIAC BIOMARKERS 42
TACHYCARDIA AND TACHYARRHYTHMIAS 42
ATRIAL FIBRILLATION 43
ANTIARRHYTHMIC THERAPY 44
HYPERTENSION 44
HYPOTENSION 45
PACEMAKERS 46
STRESS TESTS 46
THE "POST CATH CHECK" 46
HEMODYNAMICS AND SWAN-GANZ CATHETERS 47
HYPERLIPIDEMIA MANAGEMENT 48
PERIOPERATIVE CARDIAC RISK ASSESSMENT 49
CARDIOLOGY LITERATURE 51
PROCEDURES 56
JONATHAN LYONS MD, REBECCA SELL MD, AND BRIAN KOLSKI MD
CENTRAL VENOUS CATHETERS: AN INTRODUCTION 56
INTERNAL JUGULAR VENOUS CATHETER 57
SUBCLAVIAN VENOUS CATHETER 57
FEMORAL VENOUS CATHETER 59
iii
CENTRAL LINE REMOVAL 59
THORACENTESIS 59
PARACENTESIS 60
LUMBAR PUNCTURE 60
SUBACROMIAL INJECTION 61
KNEE ASPIRATION/INJECTION 62
INFECTIOUS DISEASE 63
PHILIP LEDERER MD, BENJAMIN WESTLEY, MD, ERIC ROELAND MD, RONELLE CAMPBELL MD, DAVID WYLES MD, DAVID BUTLER MD, MAILE YOUNG MD, JI CHAE
MD, AND JENNIFER BLANCHARD MD
FEVER 63
CLINICAL PEARLS 63
MENINGITIS 64
PNEUMONIA 64
LINE ASSOCIATED INFECTIONS 67
UTI 68
PATHOGENIC ORGANISMS BY GRAM STAIN 68
HIV 69
OPPORTUNISTIC INFECTION: PRIMARY PROPHYLAXIS 70
OPPORTUNISTIC INFECTION: SECONDARY PROPHYLAXIS 71
OPPORTUNISTIC INFECTION: TREATMENT 71
ANTIBIOTICS 72
GASTROENTEROLOGY 74
CAINAN FOLTZ MD, ARTHUR YAN MD, JULIO GUTTIEREZ MD PHD, MICHAEL DOCHERTY MD, ERICA BOETTCHER MD, DEREK PATEL MD, TRENT TAYLOR MD
CALLING A CONSULT 74
ACUTE PANCREATITIS 74
ACUTE GI BLEED 75
CLOSTRIDIUM DIFFICILE COLITIS 78
GERD AND PEPTIC ULCER DISEASE 78
ULCER PROPHYLAXIS 79
SMALL BOWEL OBSTRUCTION 80
HEPATOLOGY 81
MATTHEW CRULL MD, ARTHUR YAN MD, JULIO GUTTIEREZ MD PHD, MICHAEL DOCHERTY MD, ERICA BOETTCHER MD, DEREK PATEL MD, TRENT TAYLOR MD
END STAGE LIVER DISEASE & CIRRHOSIS 81
ASCITES 81
SPONTANEOUS BACTERIAL PERITONITIS 82
HEPATIC ENCEPHALOPATHY 83
ESOPHAGEAL VARICES 83
ABNORMAL LFTS 84
LESS COMMON LIVER DISEASE 85
RENAL 87
Emmett Ratigan MD, Graham Abra MD, Ken Park MD, and Carrie Chun MD
CALLING A CONSULT 87
ACUTE KIDNEY INJURY 87
HEPATORENAL SYNDROME 90
CHRONIC KIDNEY DISEASE 91
CONTINUOUS RENAL REPLACEMENT THERAPY 92
MAINTENANCE INTRAVENOUS FLUIDS 93
CONTRAST INDUCED NEPHROPATHY 92
NEPHROGENIC SYSTEMIC FIBROSIS 93
ELECTROLYTE REPLACEMENT 93
SODIUM BALANCE
HYPERNATREMIA 94
HYPONATREMIA 95
POTASSIUM BALANCE
HYPERKALEMIA 96
HYPOKALEMIA 96
CALCIUM BALANCE
HYPERCALCEMIA 97
HYPOCALCEMIA 97
PHOSPHATE BALANCE
HYPERPHOPHATEMIA 98
HYPOPHOSPHATEMIA 98
HYPOMAGNESEMIA 98
iv
EQUATIONS 99
ACID-BASE DISTURBANCES 99
METABOLIC ACIDOSIS 100
METABOLIC ALKALOSIS 100
RESPIRATORY ACIDOSIS 101
RESPIRATORY ALKALOSIS 101
GENERAL CONCEPTS 101
ENDOCRINE 103
PATRICIA LIAO MD, MATTHEW CRULL MD, KIMBERLY MAXON, MD, JEN KERNS MD, AND LI TAI MD
CORRECTING SUGARS 103
DIABETES MELLITUS 103
INPATIENT DIABETES MANAGEMENT 105
MANAGEMENT OF DKA & HHS 106
ADRENAL INSUFFICIENCY 107
PRIMARY HYPERPARATHYROIDISM 108
OSTEOPOROSIS 109
VITAMIN D DEFICIENCY 110
HEME/ONC 111
SOAMES BOYLE MD, JUSTIN PERSICO MD, KRISTEN RICE MD, MARYANNE BHOJWANI MD, PAUL CHENG MD PHD, AND STINA MUI MD PHD
ANEMIA 111
DISSEMINATED INTRAVASCULAR COAGULATION 112
HYPERCOAGUABLE DISORDERS 113
VENOUS THROMBOEMBOLISM PROPHYLAXIS 113
THROMBOCYTOPENIA 114
BLOOD PRODUCTS 115
DEEP VEIN THROMBOSIS 115
CHEMOTHERAPY & COMPLICATIONS 117
NEUROLOGY 120
TERRY FAN MD
ALTERED MENTAL STATUS 120
SEIZURES & STATUS EPILEPTICUS 121
STROKE CODE 122
PALLIATIVE CARE 123
TERRY FAN MD
CONSTIPATION 123
INSOMNIA 123
NAUSEA 123
PREVENTATIVE MEDICINE/EBM/STATS 124
JEFF WEISSMAN MDAND TERRY FAN MD
CARDIOVASCULAR RISK MODIFICATION 124
PSYCHOSOCIAL SCREENING 124
CANCER SCREENING 125
PRIMARY PREVENTION 126
EVIDENCE BASED MEDICINE & STATISTICS 127
MEDICAL SPANISH: THE BASICS 128
WINSTON TILGHMAN MD
CONVERSIONS & TABLES 130
CORTICOSTEROID CONVERSION TABLE 130
INTRAVENOUS FLUIDS 130
NARCOTIC & OPIOID CONVERSION 130
ENTERAL FORMULARY 132
PHONE NUMBERS 133
VAMC 133
HILLCREST 134
THORNTON 135
OUTSIDE HOSPITALS 136
DOOR CODES 137
1
PAPERWORK, COMPUTERS, AND DICTATION
QUICK REFERENCE GUIDE - VA
Admissions:
1. Medicine Admit pager 619-290-7173
2. The PHYSICIAN ADMISSION H&P template must be filled out for each admission UNLESS
a patient has had one filed within 30 days of their admission. If there is an H&P done within 30
days, an accept note can be written, referencing the prior H&P and with any new or changed
information.
3. If your patient is not assigned a room yet (located on top of the screen), write admission orders
under the Delayed Orders tabif you dont, you will find that your orders have been lost and
then must be rewritten.
4. If your patient is assigned a room and it states it on the top of his screen (eg 3N, DOUMED)
write admission orders under the Active Orders tab.
5. Admission order sets under medicine specialties help ensure you dont miss anything.
6. All patients admitted through the ED require orders before going to their assigned floor.
7. Food for thought: (guidelines, not necessarily rules)
- Afib/flutter, CHF requiring DOU level of care -> VICU team admits
- CHF admitted from cardiomyopathy clinic -> VICU team admits
- Post procedure cardiac patients, EP patients -> VICU team admits
- Unstable angina requiring heparin drip -> VICU team admits
Blood Draws/Labs:
1.
2. Lab draws are done @ 6am, 10am, 2pm, 4pm (drug levels only), 6pm, 10pm, or 2am, 7 days a
week. If you need a stat lab, call x3668 or page the phlebotomist.
3. Floor RNs DO NOT DRAW BLOOD, so if you need it stat, call the phlebotomist (above) or
you can always do it yourself. DOU and ICU RNs will do blood draws.
4. Floor RNs also DO NOT DRAW FROM CENTRAL LINES. If a patient has a Groshong or
Hickman for instance, you have to do their blood draws. They can draw from portacaths and
PICC lines.
Central Lines:
1. ICU RNs can d/c IJ or SC central lines but NOT FEMORAL lines.
2. Floor RNs CANNOT d/c ANY central line; you have to d/c it yourself (under supervision
please see procedure section)
CPRS: see Section: Computing Tips
DISPO Clues
- Talk with SW early and often. Try to anticipate placement needs at admission. Patients
usually CANNOT be transferred to SNFs over the weekend and SW is not in house on the
weekend. For help with SNF placement (including 4 South):
- 4S transfers/admissions 2279
- Home health needs are taken care of by the discharge planners:
- 3NLinda Suchocki 2973; 5EPat Pelican 3094
- ALL patients will need a DISCHARGE SUMMARY completed prior to transfer
- to a SNF or 4S (SW needs to fax to the facility for the patient to be considered).
Echocardiograms: Order in CPRS under CONSULTS cardiology echo. If urgent, call the Echo
Lab (1127) and/or page the Cardiology Fellow. Prelim results are available at the heart station
(4N); final reads usually take 3-5 days to show up in CPRS and a viewable under Tools > Vista
Imaging.
EKGs: Order in CPRS under Orders ->Cardiology->Procedure Orders->ECG/Holter Menu
Done by EKG techs M-F 6am-7pm and 6am-12pm on weekends and holidays (x3644, x6363); after
7pm and on the weekends/holidays after 12pm, call the ICU Gas Lab (x7264, pager 66131) for stat
EKGs.
Grand Rounds: Short & Post call teams are excused. Wednesdays at 7:30am on the med school
campus. Grand rounds is now also being broadcast to Hillcrest and Thornton and can be watched
from any internet-connected computer using:
http://neurofms.ucsd.edu/streaming/live.html, password: vmg001
2
Home Oxygen: Pts only qualify if they have on ABG PaO2 < 55 on RA or RA finger sat SaO2 <88%.
Consider getting this info before ordering a Home Oxygen Consult.
IV MEDS (including antibiotics): All IVs expire after 96 hours. For antibiotics, total duration can be
written in the comments section, which will extend expiration date. Certain antibiotics require ID
approval or they will expire after 24hrs. Page the on-call fellow for approval.
IV Push Medications on Floor: For Metoprolol, Labetalol, Verapamil, Diltiazem and Adenosine you
must place an order for telemetry in CPRS, notify the nurse youll be giving med, contact pharmacy
for the med, administer the med yourself and remain on the unit for at least one hour. The patient
should remain on telemetry or be moved to higher level floor. Not applicable in the DOU or ICU.
Library: Located on 4W; can be accessed after-hours with your ID badge.
Meals: Cafeteria hours: 6:30am-6pm M-F (Grill closes at 5:30pm), closed Sat and Sun. Meals
provided for on-call housestaff Sat/Sun in the kitchen of the call rooms suite. Meal times: breakfast at
8am; lunch at 12pm; dinner at 5:45pm.
Meal cards: $5 deposit, see personnel in the medicine office by CMRs office on 3rd floor.
PICC Lines: Look for lists of PICC nurses in team offices. Place order for PICC AND call a PICC
nurse. IR wont do them unless a PICC nurse has tried and failed, and left a note in the chart saying
they have done so. Expect long waits and try to order them early if possible. At times PICC lines can
be speeded up if they are hindering discharge by calling the bed coordinator and letting them know
about the patient.
Radiology:
1. PACS and Web 1000 access/login: There is a PACS station across from the DOU on the 5th
floor and several computers now have PACS loaded on them (look for the green rectangle
icon). Log-in/password: er/vhasdc#1. For Web 1000, in CPRS, go to "Tools" menu, there is
an option for "Online Radiology Web1000, use your vhasdc login/password
2. Impax: Use Weboutlook email login and password.
3. Radiology Rounds: Friday morning at 9am after morning conference. Keep a running list of
interesting or confounding images to give to your CMR each week.
4. Ultrasound: techs work on Saturdays, but only come in on Sundays for EMERGENCIES, page
the radiologist on call 619-290-5063.
5. Prelim reads: Look at the I-box on the PACS system, or call radiology.
6. Central lines: PICC lines, GROSHONGs, HICKMANs can be ordered under
ANGIO/NEURO/INTERVENTIONAL in the imaging orders.
7. DEXA scans: can be ordered under GENERAL RADIOLOGY.
8. IR Procedures: Know the patients coags/platelets and be sure they are NPO prior to the
procedure. Go to the IR suite or call (x6915) to discuss your case.
9. Radiologist on-call: page 619-290-5063 (after 5pm on weekdays/weekend reads)
Transfers:
1. Inter-team transfers:
- VICUWard team transfers: The VICU team writes transfer orders.
- Ward teamVICU transfers: The VICU team writes transfer orders.
- Code Status and Diagnosis orders drop off when transferred and need to be rewritten.
2. Level of care transfers:
NEW transfer orders must be written when patients transfer between DOU and FLOOR. New
orders are NOT needed between TELE and FLOOR tele orders can just be discontinued or
added.
3. Transfer to 4S (Extended Care Center):
4 South is considered an outpatient facility; when patients are transferred there, it is actually a
DISCHARGE and requires DISCHARGE ORDERS. The DISCHARGE SUMMARY must be
written PRIOR to discharge as the 4S team uses it to determine whether or not they can/will
accept the patient.
3
QUICK REFERENCE GUIDE - UCSD
Admissions:
1. Medicine Admit pager 290-1150 for Hillcrest
2. Orders need to be done before ED patients can go to the floor.
3. Each patient MUST have a UCSD H&P in EPIC. This MUST be completed by either the intern
or the resident NOT the medical student (unless it is a sub-intern).
4. Orders for patients stuck in the ED have become somewhat confusing since the transition to
EPIC. EDIP refers to ED Inpatient. If a patient is EDIPd, the nurse should be following the
admission orders in EPIC (for dosing meds, labs, meals, etc). To add new orders to such
patients, re-enter the Admissions Tab and enter single orders. If a patient is NOT EDIPd, but
still in the ED, then orders should be entered in Webcharts until the patient makes it up to the
floor. Call to clarify with the nurse taking care of the patient where active orders should
be entered for ED patients. It is not always clear is a patient is EDIPed or if the EPIC orders
are actually being heeded.
5. Food for thought (guidelines, not necessarily rules):
- New onset heart failure CCU team admits
- Rule out MI CCU team admits
Antibiotics: Simply type in the name of the antibiotic you wish to use in the orders section. When
filing in the details of administration, you will see whether or not an antibiotic is restricted under the
Order Inst tab. You must speak with the ID fellow on call within 24 hours of starting a restricted
antibiotic to continue its use.
Blood Draws/Labs: 24 hour phlebotomy. Nurses will draw from central lines. DO NOT ACCESS
DIALYSIS CATHETERS. Only dialysis nurses should draw from them.
Call rooms: Located on the 3
rd
floor main hospital access from the patient elevators. Keys available
from med office will open large double doors outside of elevators and call rooms. Rooms have a
computer, phone and shared bathroom.
Central Lines: Floor RNs DO NOT d/c central or arterial lines. ICU nurses do.
Copiers: There are copiers on most floors; use 5436272 as password (the medicine dept. phone).
DISPO Clues The Case Managers are your KEY to DISPO. They generally think ahead for you,
but keep in touch with them because their help can MAKE or BREAK getting a patient dispod. There
is a weekend Social Worker and Case Manager on call, but its easiest to have things arranged
already.
Pts going to SNF need to be sitter free for 24 hours prior to leaving.
Not every SNF will take patients with feeding tubes. Ask.
Patients going to Hospice or SNF need d/c summaries done prior to transfer.
Echocardiograms:
- Order in PCIS, may want to call the Heart Station (3-6399) to confirm. For urgent echos,
page the Cardiology Fellow.
- Prelim reads can be found in the Reading Room @ the Heart Station located on the3
rd
floor
near the noon conference room.
ER Computers:
- Online access at: webcharts.ucsd.edu. This can be accessed from most hospital computers.
Talk to any ER attending for a login and password.
EKGs:
- Online access at: webcharts.ucsd.edu.
- Once in ANY pts record, click on EKG on top menu bar.
- Login: ad/emergency Password: Emer!gency
Library: (UCSD Medical Library); located behind the hospital, Mon-Fri 7am-8pm, Sat-Sun 1pm-5pm
4
Meals:
- Scan your ID card to pay for meals. Money at the end of the month keeps rolling over from
month to month but is zerod at the beginning of the academic year.
- Cafeteria Hours
Hillcrest: 6:30am-8pm, then 11:30pm-2:30am, 7 days a week.
Thornton: 6:30am-8pm (grill closed 3-4:30, after 7pm) M-F, 6:30am-7pm Sat/Sun.
Page Forwarding: Each team intern must forward pages to the covering physician after sign out. All
pagers start with 290. It is optional for residents to forward pagers, however if not forwarded, pages
must be returned.
- Dial your pager, then enter 01234 (access code)
- Enter 166 (to access system and forward; 163 to cancel page forward)
- Enter the 7 digit pager number to which you are forwarding followed by #.
- Re-Enter the 7 digit pager number again; press # (to confirm)
Parking:
1. Parking Office @ Hillcrest in Bachman Structure (2
nd
Floor). Mail Code 8205; x36524.
Housestaff must obtain parking permits from this office (not from office on UCSD campus).
You have the option of paying via automatic paycheck deduction. Get chip from this office to
place on back of ID badge so that you may open security gate.
2. Renew your parking pass each June; youll get a renewal form in UCSD mailbox.
PICC Lines: Order in PCIS, will usually be done in 1-2 days by PICC nurse. If urgent, page the
infusion nurse under vascular access team in webpaging. IR wont do it unless the infusion center
has tried.
Phones:
1. To dial out: 8 + 1 Phone Number. Some phones require you to hit 9 first, to turn tone sender
on on the phone.
2. To page: go to webpaging.ucsd.edu.
3. To tag your pages: always include your call-back phone number AND your pager number.
Radiology: PACS Access: Same user name and password as your UCSD email account.
1. MRIs: You can ONLY get MRI with sedation on Wednesdays from 9:30am-2:30pm. They will
collaborate with Anesthesia. Keep this in mind if you have a claustrophobic
patient. Write an order in the chart AND call 32940 for scheduling.
2. IR Procedures: Know the patients coags/platelets and be sure they are NPO prior to the
procedure. Go to the IR suite or give them a call (x32138/32476/35747) to discuss your case.
3. Radiologist on-call: 290-5063 (for reads after 5pm, and weekend reads)
Swallow studies: speech path x36530 Write an order in PCIS. You can page them if urgent
by calling the operator to see which pathologist is on duty.
Transfers:
1. Inter-team transfers: The accepting team is responsible for writing TRANSFER orders.
2. Level of care transfers: NEW transfer orders must be written when patients transfer between
IMU, FLOOR and TELEMETRY.
COMPUTING TIPS
CPRS Quick Tips:
1. HELP = vaww.san-diego.med.va.gov/ITS/cprs or call x4767
2. ACCESS from home/UCSD medicine office computers is available. Visit Jesse Christmas
(ext 7802) at Computer Information Office in SCI basement to get paperwork. You must have
paperwork completed to access UCSD computers even if you dont have home access.
3. Discharge orders must be written under ACTIVE ORDERS; if you use the DISCHARGE
section, nurses cant view them, and youll have to re-write them.
4. Try to anticipate a discharge time and date and order it under Discharge time for planning.
5. Transfer orders must be written under TRANSFER section as DELAYED.
6. How to create your own inpatient list: go to TOOLS, then OPTIONS, then Lists/Teams, then
Personal Lists, then New List (must name the list first), then begin adding patients, click Save
to save changes.
5
7. How to create a list of commonly accessed tests
- On LABS tab, choose WORKSHEET
- List up to 7 labs that you would like to group together
- In top right-hand corner, choose NEW and your group has been created.
- Every time you use the worksheet function, your test groups will be listed in the top half of the
window for easy access.
8. How to create templates
- Under OPTIONS header, select CREATE NEW TEMPLATE
- Type in the name of your template
- Begin typing your template in the bottom half of the window
- Press OK to save it.
9. How to get to the medicine resident note template
- This will automatically import the most recent vitals, chem., CBC, and coags
- Go to Shared Templates at the side tool bar
- Select General Medicine
10. How to create a list of your most frequently used notes
- Under TOOLS header, select OPTIONS
- Choose Document Titles
- Select the type of note you would like added to your list (ie, MED/GEN/RESIDENT)
- Press ADD to add it to your list
- Press SAVE CHANGES
- Now your list will pop up whenever you start a new note, and youll have ready-access to the
most titles of the notes you most frequently use
11. Accessing remote records in CPRS
- Choose REPORTS tab
- If remote records are available, a tab in the upper right-hand corner of the screen will have
Remote Records in blue; if it is in grey, then there are none available.
- Click on the Remote Records tab and activate the sites in which you want.
- In the Left-Hand column, you will see Clinical Reports
- Expand the menu and choose items of interest.
12. How to alert yourself when lab/imaging/consult results are available
- Select the lab/image/consult that you want to be alerted about.
- Select the VIEW header.
- Choose Alert When Results
13. Access to UCSD Webproxy: for Online Clinical Library on your VA desktoptype in
ww.cwp.ucsd.edu. Same username and password are the same as your weboutlook email.
14. Access to UCSD Webpaging: Same as at Hillcrest or Thortontype in webpaging@ucsd.edu.
EPIC Quick Tips:
1. 24 hour help: ie. If you cant remember login/pswd x37474
2. Home access: @ www.cwp.ucsd.edu. Click on EPIC icon to enter. Use your AD username and
password.
3. How to create your own inpatient list:
-click on My Patient Lists on the upper left column in the Patient Lists tab (the
nameless icon tab to the far left)
-click on Create and then type in the name you want for your list and select
parameters to be displayed (especially useful are Patient Name/Age/Sex, Room, MRN,
Arrival Date and whatever else you see fit)
-add patients by either clicking on Add Patient from the icons at the top and entering
your patients name or MRN or by copy and pasting them from the Systems Lists on
the left hand column. Most useful System Lists include Units Hillcrest (choose a floor
or ED Inpatient for patients still downstairs) and Service Teams (scroll down to find the
Med Hillcrest team your patients belong to)
4. Find finger stick blood sugars under Results Review tab at the bottom of the list: Glycemic
Control POCT
5. Inpatient medication profile: under the Medications tab. The best way to review which
medications your patient has actually received is to click on the Patient Summary tab and
then type in IP MAR Current or IP MAR History into the white box next to the word report.
You can add this to your permanent tabs on this screen by clicking on the wrench icon next to
the box and type in your choice into the matrix.
6. Outpatient medication profile: under the Medications tab. Select Prior to Admission
medications.
7. Lab Reference Guide: click on Web Ref (just next to the word EPIC at the top left of the
screen) and choose Laboratory Information and then Lab Reference Guide.
6
8. Nursing Policies: there are a number of useful resources listed under the Nursing Standards
and Guidelines tab of the Web Ref (see #8 above). Choose Standards and you can then
select Level of Care Criteria, which helps you determine whether a patient with a certain drip
or nursing requirement is appropriate for a certain floor (the message may not be consistent
among all nurses asked) or Paging Standard RN Physician Team to determine whether
or not it is appropriate to correct a nurse about a particular instance of paging.
9. How to find out who a patients primary care physician is and what their insurance is: click on
the Facesheet icon at the top of the Patient Summary tab.
WEBSITES
1. Organizations & Services
American College of Physicians www.acponline.org
California Medical Board www.medbd.ca.gov
CPRS (remote access) citrix.san-diego.med.va.gov
DEA www.deadiversion.usdoj.gov
Epocrates www.epocrates.com
Hopkins Curriculum www.hopkinsilc.org/
JAMA jama.ama-assn.org
New England Journal of Medicine www.nejm.org
New Innovations www.new-innov.com
PCIS (remote access) cwp.ucsd.edu
PubMed www.pubmed.gov
UCSD ER Charting System webcharts.ucsd.edu
UCSD Healthcare health.ucsd.edu
UCSD Library gort.ucsd.edu/clinlib
UCSD Medicine Residency imresidency.ucsd.edu
UCSD Pharmacy health.ucsd.edu/pharmacy
UCSD Webmail (Popmail) acs-webmail.ucsd.edu
UCSD Weboutlook weboutlook.ucsd.edu
UCSD Webpaging webpaging.ucsd.edu
UCSD Web-based Signouts www.mypatientyourpatient.com/ucsd
UCSD Web1000 web1000.ucsd.edu/Launch
Up-to-Date www.utdol.com
Verizon Wireless Online Paging www.myairmail.com
2. Evidence Based Medicine
ACP Journal Club www.acpjc.org
Center for Evidence Based Medicine www.cebm.net
Cochrane Library www.cochrane.org
3. Palm Resources
- palm.stanford.edu/quicksoftware.html - Stanford Meds palm resource page.
- www.guideline.gov/resources/pda.aspx - A list of pocket guidelines for your PDA. There are a
lot so you may have to pick a few to download and try.
- www.statcoder.com - Check out the free cardiac clearance, LDL cholesterol, and JNC 7
guidelines.
- www.timi.org - The TIMI risk calculator is an interactive and easy to use tool developed by
the authors of the TIMI trials.
- www.memoware.com - A collection of free software, including some medical software.
- www.freewarepalm.com - Check out eponyms and some of the useful calculators.
- eponyms.net/palm.htm - A useful website filled with links to several palm resources.
- www.healthypalmpilot.com - A useful website with downloads to tons of resources.
4. Miscellaneous
Balboa Park Visitors Information www.balboapark.org
Federal Loan General Website www.dl.ed.gov
Federal Loan Consolidation www.loanconsolidation.ed.gov/index
San Diego Special Events Calendar www.sandiego.gov/specialevents/index
San Diego Union Tribune signonsandiego.com
7
DISCHARGE PROCEDURES
UCSD
1. Residents are responsible for writing discharge summaries in EPIC. Patients being
discharged to a SNF or hospice need a DISCHARGE SUMMARY written that can go with
them at the time of discharge. Often times this is the discharge documentation so the
description needs to be complete.
2. Security Prescription Forms: The State of California has mandated that as of January 1,
2005, ALL controlled drugs in schedules 2, 3, 4, and 5 must be prescribed on a tamper
resistant security form. All attendings will have security forms and the residency program
provides licensed residents with a DEA number and their own forms as well.
3. Handwritten Prescription Forms: For patients going to SNF, select the Yes button on the
Facility Discharge? tab from the Discharge menu. This will default all prescriptions to
handwritten format. Once the discharge orders are signed, a prescription sheet will print at the
nursing station on the patients floor and you must sign this form in orderyou're your patient to
successfully leave the hospital.
4. ALL PATIENTS MUST HAVE SOME SORT OF FOLLOWUP.
VAMC
1. Discharge planning rounds for residents are every Wednesday.
2. Discharge summaries are REQUIRED for all patients (regardless of length of stay) and are
due within 24 hours of discharge.
3. If a pt is going to 4S/SNF, a discharge summary will be needed prior to DC.
4. Interns are responsible for all D/C summaries, unless they have >6. In this case, the
residents are responsible for pts with SSN ending in numbers 0-4; interns are responsible for
pts whose SSN ends with the numbers 5-9.
5. If the patient is discharged on the interns day off, the resident is responsible for the summary
and vice versa.
6. Medical students following the patient should write the summary; however, they cannot write it
under the discharge summary tab. They should write it as a regular progress note but should
not sign it. The note can then be pasted into the discharge summary note by the
intern/resident with credit being given to the student. The draft progress note can then be
deleted.
7. Discharge orders should be written under active impatient orders. (Under Main Inpatient
\)Menu choose Discharge Instructions to launch the order set.)
8. Medications at discharge:
- Review your pts outpt medication profile and compare it to the inpt med regimen before
writing your orders. Discontinue any meds no longer required.
- Order only meds the patient requires at discharge.
- Do not discontinue and re-enter all of your patients meds if nothing has changed.
- Refill any active outpatient prescription. This avoids unnecessary co-pays.
- Renew expired or zero refill prescriptions if appropriate.
- If you have a STAT discharge, then enter your meds in STAT (recommend 2 hours of turn
around time).
- IV antibiotics are coordinated through the discharge planner.
- Patients being sent to a nursing facility need a 5-day supply of medications.
- Patients being sent to a board and care facility need a 30-day supply of meds.
9. For patients being sent to 4 South (VAMC Extended Care Center), this is a discharge.
However, you do not need to order their medicationsjust type a list of their medications in
your discharge summary.
8
VA Formulary
Pearls:
1. Meds that are non-formulary (NF) require consult approval before they can be dispensed.
2. ID restricted antibiotics can be given for 24 hours but then will require ID approval within
24h.
To access the formulary:
1. CPRS Tools Menu: select Pharmacy References and Formulary, then click Formulary
2. VA Web page: Services PharmacyFormulary (on left side menu)
Submitting an NF/PA Consult:
1. Go to ORDER tab, select CONSULT (inpt or outpt use)
2. Click on NON-Formulary/Prior Auth Drug Request. There are specific consults for select
medications and a generic NF form for all other medications.
3. You must enter a CPRS medication order for the requested drug. Page your area pharmacist if
you cannot enter the drug order in CPRS.
Most common consults:
1. ARBs: valsartan for CHF, losartan for DM, need consult.
2. Ezetimibe: monotherapy in pt with CI to statin and niacin. Combo with max statin and CI to
niacin/cholestyramine OR needs a 3
rd
agent.
3. Glargine: recurrent symptomatic hypoglycemia OR intensive insulin therapy and documented
improvement during first 6 months.
4. Tamsulosin: documented orthostatic hypotension/syncope on alpha-blocker.
5. Zolpidem: NF consult needed, except in sleep apnea w/documented desaturation.
6. Plavix/Lovenox/Formoterol/ IV PPI (pantoprazole)/COX 2
Cant find your med to order
1. You are not alone. Some meds are only found under Quick PO Meds, in the Meds menu.
These include: Clopidogrel, Carvedilol, Rosiglitizone,Simvastatin, Zolpidem
2. Controlled substances: not orderable via CPRS. Give green script form to pharmacist.
3. IV antibiotics: many are listed underOrders tabMeds (inpt)quick IV abx.
Medication Conversions: see Pharmacist for questions on conversion.
Death Procedure
Prior to pronouncing the death:
1. Speak to nursing staff to get details about the patient, family, family dynamics and the
circumstances of the death.
2. Determine if there are any special problems or concerns.
3. Assess the immediate situation.
- Was the death expected or unexpected?
- Was the family present? If not have they been notified?
4. Find out about autopsy and organ donation from next of kin.
- Has family requested an autopsy? Is there an indication for an autopsy?
- Is the patient an organ donor? If not, has family considered organ donation? If patient is a
possible organ donor, contact Life Share (ask nursing staff for details).
Evaluate the patient:
1. Identify the patient by the hospital identification tag or bracelet.
2. Note the general appearance of the patients body.
3. Determine that the patient will not rouse to verbal or tactile stimuli.
4. Listen for the absence of heart sounds; feel for the absence of a carotid pulse.
5. Look and listen for the absence of spontaneous respirations.
6. Check the position of the pupils and the absence of the pupillary light reflex.
7. Note the time your assessment was completed. This is recorded as the official time of death.
Contact any next of kininquire about autopsy, organ donation. Contact Organ Donation/Life
Share if indicated.
9
Put Death Note in the medical record
1. State name of the provider. Write Called to pronounce (Insert patient's name).
2. Chart findings of physical examination, including absence of pulse, respirations, pupil
response.
3. Note date and time of death.
4. Indicate in chart if patient's family was notified.
5. Chart if family declines or accepts autopsy.
6. Document if the coroner or medical examiner was notified. If the death might be a
coroner/medical examiner case, contact the medical examiner. If the medical examiner waives
the case, get a waiver number for the death packet.
7. Document notification of attending, pastoral care, social work, or others as indicated.
8. Fill out Death Packet (different for UCSD Hillcrest/Thornton and VA).
Notify the patients attending: Even if it isnt your attending and it is 3am, send them a text page.
TRANSFERS AND DISCHARGES
Transfer Summaries:
- Date of admission / Date of transfer
- Attending Physician / Resident / Intern
- Procedures/Radiologic studies
- Consultations
- Problem List
- Brief HPI / PMH / PSH / FamHx / SocHx
- Hospital course by problems
- Medication list
Discharge Summaries:
- Date of admission / Date of discharge
- Attending Physician / Resident / Intern
- Procedures/Radiologic studies
- Consultations
- Problem List
- Brief HPI / PMH / PSH / FamHx / SocHx *
- Admission physical exam *
- Initial laboratory studies, radiology *
- Hospital course by problems
- Disposition
- Condition on discharge
- Discharge medication list
- Followup
* If there is a dictated History and Physical in the computer you may reference it
10
Graduation Requirements
.
PROCEDURES:
Required: Elective (examples):
Abdominal paracentesis (3) Arterial line placement (5)
**ACLS certification Bone marrow aspiration (5)
**Arterial puncture (5) *Central venous line placement: Femoral (5)
Arthrocentesis of the knee (3) Elective cardioversion (requires supervision)
*Line placement: IJ (5) or Subclavian (5)
*Central line removal (5)
Endotracheal intubation (req. supervision)
**IV access (5) Flexible sigmoidoscopy (25)
Lumbar puncture (5) Holter monitor interpretation
**Pap smear and endocervical cx (5) Nasogastric intubation (3)
Thoracentesis (5) S-G catheter placement (req. supervision)
**Venopuncture Temp. pacemaker (requires supervision)
Treadmill testing and interpret
Vascath placement (requires supervision)
*Placement of central lines in the UCSDMC CCU requires direct supervision by the fellow or
attending physician regardless of the number of procedures previously done by the resident.
**Required procedures to sit for the American Board of Internal Medicine (ABIM)
CLINICAL EVALUATION EXERCISES (CEX):
1. One long CEX
2. Two mini CEXs/year
3. Med/peds: one long and three mini total
EVIDENCE BASED MEDICINE:
1. 2
nd
/3
rd
year ward residents
2. 5 minute presentation at morning report addressing a focused clinical question
3. Email a copy to medicine office to be filed away
4. A minimum of five EBMs to graduate (for both categorical and med/peds residents)
MECHANISM OF MEDICINE PRESENTATIONS:
1. 2nd/3
rd
year consult residents
2. 5 minute presentation at morning report addressing mechanism (molecular, cellular, or
physiologic) and, if applicable, therapeutic potential.
3. Email a copy to medicine office to be filed away
JOURNAL CLUB PRESENTATIONS: One presentation is required of all residents completing the
program.
SCHOLARLY ACTIVITY REQUIREMENT (SAR): One scholarly activity project is required for all
residents completing training in Internal Medicine. Options for completing the project include the
following which must be done and published by May 15
th
of the graduating year:
1. Research Project: Culminating in written abstract, poster presentation, original manuscript for
submission, or written description in manuscript, and presenting your findings at the Annual
Resident Research Symposium.
2. Case Report and case conference presentation
3. State of the Art Review of Topic: Review article and presenting your findings at a noon
conference or grand rounds
11
JOHN HOPKINS CURRICULAR REQUIREMENT FOR AMBULATORY CARE:
Categorical Medicine Residents Required Modules (15 total out of 33):
PGY-1: 4 modules
Cancer screening
Immunizations
Professionalism
Low Back Pain
PGY-2: 7 modules (3 new + 4 were assigned last year as PGY-1s)
All of the above 4 modules AND
Preoperative Assessment
Any 2 of the 30 modules not done above
PGY-3: 15 modules (8 new + 7 were assigned as PGY-1-2s)
All of the above 7 modules AND
Dermatitis
Ophthalmology
Any 6 not done above
Med/Peds Required Modules (10 total out of 33):
PGY-1: 3 modules
Immunizations
Professionalism
Cancer Screening
PGY-2: 4 modules (1 new +3 were assigned last year as PGY-1s)
All of the above AND
Lower Back Pain AND
PGY-3: 5 modules (1 new + 4 were assigned as PGY-1-2s)
All of the above AND
Preoperative Assessment
PGY-4: 10 modules (5 new + 5 were assigned last year as PGY-1-3s)
All of the above AND
Dermatitis
Ophthalmology
Any 3 not done above
12
Pulmonary/Critical Care
Mechanical Ventilation
Indications for Intubation:
1. Airway protection from obstruction or aspiration
- GCS <8, airway obstruction, or laryngeal edema
2. Failure to ventilate. PaCO2 > 55 mmHg with acute acidosis (pH < 7.25)
- Hypoventilation, neurologic compromise, muscular weakness, anatomical restrictions,
gas exchange abnormality.
3. Failure to oxygenate. PaO2 < 60 mmHg despite max FiO2 delivered non-invasively
- Diffusion abnormality, V/Q mismatch, inability to extract O2 at cellular level.
4. Clinical judgment. Note that these are not hard and fast rules, just some parameters.
Trending of these parameters should be analyzed. Decision for intubation and initial vent
settings should be discussed with the ICU Fellow/Attending.
- Respirations > 35/min for prolonged period of time (leads to exhaustion)
- PaO2 < 55 mmHg
- Minute ventilation > 10 L/min
- A-a gradient* > 350 mmHg on 100% oxygen
- PaCO2 (arterial partial pressure of carbon dioxide) > 55 mmHg (except in chronic
retainers)
*A-a gradient = Difference between Alveolar oxygen and arterial oxygen, or P
A
O
2
P
a
O
2
.
P
A
O
2
= (FiO
2
x (P
atm
P
H2O
)) (Pa
CO2
/0.8) where P
atm
-P
H2O
= 760-47 = 713
Expected A-a gradient = (patient's age divided by 4) + 4
VENT TERMS/DEFINITIONS
1. Control/mode: parameter that determines how the ventilator controls pressure, volume, and
flow within a given breath as well as how it manages the sequence of mandatory and
spontaneous breaths.
2. Cycling: parameter that determines when the change from inspiration to exhalation occurs
- Time-cycled
- Volume-cycled (can also add inspiratory pause to make it both volume- and time-cycled)
- Flow-cycled as in pressure support
3. Triggering: parameter that determines when the ventilator delivers the breath
- Time
- Pressure pt generates negative pressure and breath is delivered. Usually -1 to -2 cmH
2
O.
- Flow most ventilators use flow by which is a constant flow of air within the circuit; when
the pt attempts inhalation there is a change in this flow. This is sensed by vent and a breath
is delivered.
4. Breaths: three types: mandatory, assisted, or spontaneous
- Mandatory = delivered completely by the ventilator, independent of pt
- Assisted = pt effort to breathe is sensed by the ventilator, which then delivers a full breath
- Spontaneous = pt breaths on his/her own, without help from the ventilator
5. Extrinsic PEEP: positive end expiratory pressure provided by the ventilator to prevent alveoli
from collapsing at the end of expiration. This recruits alveoli, thereby improving gas exchange
and therefore oxygenation.
- Indications for PEEP:
- Improve gas exchange (and decrease intrapulmonary shunt as well as decrease the
amount of FiO
2
needed)
- Increase lung compliance
- Prevent lung injury by minimizing volutrauma and stress of shear forces when alveoli
repeatedly open and close
- Prevent end-expiratory airway collapse (as in obstructive lung disease) and thus
decrease work of breathing.
13
- Adverse Effects:
- Increases intrathoracic pressure and reduces venous return to the right heart (can
cause hypotension).
- Can over-distend alveoli and redirect blood to diseased areas (thus increasing dead
space).
6. Plateau Pressure (P
plateau
): pressure needed to keep alveoli open at the end of inspiration.
- P
peak
P
plateau
airflow resistance (normal = 5 mmHg, but definitely <10 mmHg)
- Increased P
peak
with unchanged P
plateau
means increased airflow resistance
DDx: ETT obstruction, increased secretions, bronchospasm
- Increased P
peak
with | P
plateau
decreased distensibility of lung and chest wall
DDx: PTX, atelectasis, acute pulmonary edema, worsening pneumonia or ARDS, increased
abdominal pressure, auto-PEEP
- Decreased P
peak
air leak DDx: disconnected tube, cuff leak, hyperventilation
7. Static Compliance (C
static
) :
PEEP P
TV
plateau

, normal= 50-80 cmH
2
O. A measure of the
distensibility of the lung.
- To calculate Dynamic compliance simply use peak inspiratory pressure (PIP) instead of the
plateau pressure.
- Chest wall can account for 35% of total static compliance.
8. Airway Resistance (R
insp
):
insp
plateau peak
V
P P
, where V
insp
is the inspired volumetric flow rate.
- Minimal flow resistance in typical ETT is 3-7 cmH
2
O/L/sec
9. Determinants of Oxygenation (determinants of PO
2
):
- FiO
2
- PEEP
- Mean Airway Pressure (this is why APRV and inverse ratio can improve oxygenation)
10. Determinants of Ventilation (determinants of PCO
2
):
- Minute Ventilation = RR x TV
- Dead Space (dramatic increases/changes in dead space will effect ventilation a common
example: massive PE increases dead space because alveoli are ventilated but there is no
perfusion).
Common Modes of Ventilation
Volume Control: Mandatory breaths deliver a pre-set volume. Pressure is determined by lung and
chest wall mechanics.
1. Continuous Mandatory Ventilation (CMV): mandatory breaths are delivered at a set RR and
volume. Commonly used in the OR, but not well tolerated in pts with intact respiratory drive.
2. Assist-Control (AC): Mandatory breaths are delivered at a set rate and volume, and
triggered breaths (when pts effort sensed by vent) are assisted in the same manner as the
mandatory breaths (vent delivers same volume or driving pressure) and have the same
limits/cycling as a mandatory breath).
- Advantage: Good for pts with impaired respiratory drive
- Disadvantages:
- Excessive pt-triggered breaths may lead to respiratory alkalosis or breath stacking
(auto-PEEP).
- Usually not well tolerated without sedation.
- May increase the work of breathing if: a)_sensitivity to pt effort is inadequate for the pt
to trigger assisted breaths, or b) flow rate is insufficient for pt comfort and thus pt
exerts persistently increased respiratory effort.
14
- Example of initial settings:
- RR 10-12
- TV 6-12 mL/kg [NOTE: In ARDS, goal is usually 6-8cc/kg (ideal body weight)]
- Normal Flow = 60-80mL/min
- Flow rate can be increased to decrease the Inspiratory time and thereby increase
expiratory time. This is useful in pts with severe obstructive physiology and/or auto-
PEEP.
- PEEP = 5 cmH
2
O (if PEEP is necessary, it often isnt)
- FiO
2
= 1.00. This is then titrated down according to ABG or pulse oximetry.
3. Synchronous Intermittent Mandatory Ventilation (SIMV): Mandatory breath rate is set and
these breaths are volume-controlled. The TV for the spontaneous breaths will be determined
by chest wall/lung mechanics/compliance and pt effort. Spontaneous breaths are therefore
unsupported, but pressure-support can be added to assist these breaths. Mandatory breaths
are synchronized with spontaneous breaths.
- Advantages:
- Very useful in the post-operative period.
- May be helpful in brain injury when there is centrally-mediated increase in respiratory
rate which causes respiratory alkalosis when AC mode is used.
- Mandatory breaths are synchronized with spontaneous breaths, thus preventing
breath stacking (auto-PEEP). This also leads to greater pt comfort. If asynchronous,
then the mandatory breaths can be delivered at any time in the respiratory cycle (pure
IMV).
- Disadvantages:
- If spontaneous breaths are not sufficiently supported with pressure-support (i.e. TV is
low), may stimulate tachypnea and increased work of breathing.
- The work of breathing may also be high if the mandatory rate is set too low.
- Example of initial settings:
- Similar to AC mode (see above).
- In addition, may add pressure support to the spontaneous breaths.
Pressure Control: Delivers a pre-set pressure, usually for a set time. TV is determined by lung and
chest wall mechanics as well pt effort.
1. Pressure Controlled Ventilation (PCV): In our ICU, most commonly used in AC mode. Thus,
for each breath, whether mandatory or pt-triggered, a given driving pressure is used to inflate
lungs. The pressure is held for a certain time (time-cycled) and then released.
- Advantages:
- High flow rates are usually more comfortable and therefore lead to decreased work of
breathing.
- Prolonging inspiratory time allows distribution of ventilation and may improve
oxygenation by allowing recruitment of more stiff alveoli (i.e. increasing mean airway
pressure). This may help refractory hypoxemia. Note that once the inspiratory time
exceeds the expiratory time, then inverse ratio ventilation is being used.
- Disadvantages:
- Poor lung compliance may lead to low TVs or minute ventilation and therefore
hypercapnea (although this is usually well tolerated).
- The longer the inspiratory time, the more improved the oxygenation. However, more
air trapping (auto-PEEP) occurs as the time for expiration decreases.
- Minute ventilation is not guaranteed (TV determined by lung and chest wall
mechanics), therefore more monitoring by physician and RT is needed.
- Example of initial settings: more difficult than in other modalities.
- The pt must be monitored to ensure the desired minute ventilation is being reached
with the set driving pressure or goal TV (usually 6-8 cc/kg).
- Attempts are made to keep Peak Airway Pressures < 35-40 cmH
2
O.
- Inspiratory time (time that pressure is held) must be set as well (usually 1 sec). On
some ventilators this is set as the fraction of an entire inspiration/expiration cycle.
15
1. Airway Pressure Release Ventilation (APRV): A form of time-cycled/pressure-controlled
ventilation. In this mode, inspiratory pressure is kept high through most of the respiratory cycle
with occasional drops in pressure to allow for ventilation. Most often used in severe refractory
hypoxemia and ARDS. See figure below.
- Advantages:
- Pts can usually spontaneously breathe throughout the cycle. Therefore, less sedation
can be used.
- Majority of ventilation occurs during expiratory phase (decreased pressure), thus
preventing barotrauma and volutrauma.
- Disadvantages:
- Cannot use in pt who are difficult to ventilate, such as pts with severe COPD and
cannot empty their lungs quickly or in pts with increased airway resistance.
- Example of initial settings:
- Expiratory time should be short enough to prevent de-recruitment and long enough to
obtain suitable TV. Usually 0.4-0.6 sec. Target TV is 4-6 mL/kg.
- High PEEP usually set at 28 cmH
2
O (to help recruit more alveoli) and titrated down.
- If transitioning from another mode, then use the old mean airway pressure.
- Low PEEP usually set at 0 cmH
2
O.
- Inspiratory time usually set at 4-6 sec. Target rate is 8-12/min.
High Airway Pressures
If sats dropping or hemodynamically
unstable then disconnect circuit and
bag while determining cause.
If stable then check P
peak
, P
plateau
and end-
expiratory pressures (pressure at end-
inspiration, ie. inspiratory hold maneuver,
pressure at end-expiration is determined by
pause at expiration)
Resistance
P
peak
(> 35 cmH
2
O) and
P
plateau
normal ( 35 cmH
2
O)
- mucus plug
suctioning
- biting ETT sedation
- tracheal obstruction
reposition ETT
- bronchospasm
bronchodilators
Decreased Compliance
Peak and Plateau P > 35 cmH
2
O
Check CXR and ABG while you are thinking about these
possible causes:
- ETT dislodgement, ie. in right mainstem bronchus
reposition ETT
- Dysynchrony (bucking the vent, look at flow-pressure
waveforms) sedation
- Auto PEEP (pressure at expiratory pause will be greater
than your set PEEP, or flow at end-expiration is not 0)
increase expiratory time by RR and/or TV, inspiratory
flow rate (decreases inspiratory time), or in severe cases
(hemodynamic effects) disconnect pt from vent and allow
for a full expiration. Sometimes increased sedation (or
paralysis in severe cases) is needed.
- Atelectasis, PNA, CHF treat underlying cause
- Pneumothorax chest tube
HIGH PIP
16
Liberation
This process accounts for up to 40% of time that patient is on the ventilator.
1. Assessing Readiness:
- Respiratory criteria:
o Oxygenation: PaO
2
> 60 mmHg on FiO
2
< 0.40 and PEEP s 5-8 cmH
2
O
o Ventilation: PaCO
2
at baseline (unless permissive hypercapnea)
o Strength: Patient able to initiate respiratory effort/lift head off bed
- Cardiovascular criteria: No evidence of myocardial ischemia; HR s 140; BP normal without or
with minimum vasopressors.
- Mental status: Arousable; GCS 13.
- Initial cause for intubation reversed or significantly improved.
Typical Bedside Weaning Parameters
Parameter Normal Adult Range Weaning Threshold
PaO
2
/FiO
2
Tidal Volume
Respiratory Rate
Vital Capacity
Minute Ventilation
>400
5-7 cc/kg
14-18
65-75 cc/kg
5-7 L/min
200
5 cc/kg
<40
10 cc/kg
< 10 L/min
Max inspiratory Pressure
RSBI (RR/TV ratio)*
> -90 cmH
2
O (women)
> -120 cmH
2
O (men)
< 50/min/L
-25 cmH
2
O
< 100/min/L
2. Sprinting:
- CPAP is often used as a means of minimal pressure support so that the patient can
overcome the resistance of the ventilator circuit. In this mode, the patient is initiating each
breath but ventilatory parameters can still be monitored. RSBI (rapid shallow breathing index,
RR/TV) holds the greatest predictive value for success. For patients with neuromuscular
weakness, maximal inspiratory pressure is also very helpful.
*RSBI > 105 predicts 95% failure rate for weaning.
*RSBI < 105 predicts 80% success rate for weaning.
References: Tobin, MJ. Mechanical Ventilation. N Engl J Med 1994; 330:1056-1061. Marino, P. The ICU Book. 3
nd
Ed. Lippincott. New York
2007: 457-511.
NON-INVASIVE POSITIVE PRESSURE VENTILATION (NIPPV)
Several studies have shown that using NIPPV can prevent intubation and thus decrease length of
hospital stay and mortality. Consider a trial of NIPPV in pts with moderate to severe respiratory
distress and good mental status (they must be able to work with RT in order to be effective). Be sure
to check serial ABGs and dont hesitate to intubate if pt is not improving. (See Cross Cover Clues:
Dyspnea).
**Note that NIPPV does not prevent the need to re-intubate or reduce mortality in a pt with respiratory
failure after extubation (NEJM 2004; 350(24): 2452-60).
1. Indications for NIPPV
- COPD exacerbations. BiPAP has been shown to decrease mortality rates, intubation rates,
length of hospital stay, and complication rates such as PNA. This is probably related to
decreased intubations. BiPAP, specifically EPAP assists in decreasing the work of
breathing.
- Cardiogenic pulmonary edema. CPAP (usually started at 10cm H2O) has been shown to
decrease the work of breathing and thereby increase cardiac output. It is thought to increase
the functional residual capacity and thus place the lung/chest wall system on a more
compliant part of the curve. Decreases rate of intubation with no significant changes noted
with mortality. There may be increased mortality in pts with active ischemia, so use caution in
these pts.
- Immunocompromised patients with hypoxemic respiratory failure. A study with 40 pts
(mostly solid organ transplant pts or hematologic malignancy) showed decreased need for
intubation, serious complications, and mortality.
17
2. Contraindications for NIPPV
- Altered mental status or inability to cooperate
- Severe upper GI bleed
- Cardiac or respiratory arrest or other organ failure
- Inability to clear secretions or high aspiration risk
- Hemodynamic instability
- Surgery or trauma to face or neurological system
- Recent upper airway or GI surgery think carefully: may be at risk of viscus perforation
- Mask or nasal prongs cannot be fitted
3.
4. Types of NIPPV
- Continuous positive airway pressure (CPAP): delivers continuous positive airway
pressure throughout the respiratory cycle. Similar to breathing with your head stuck out of a
moving car.
- Counteracts intrinsic PEEP
- Decreases preload and afterload and improves lung compliance in CHF
- Decreases the work of breathing
- Bi-level positive airway pressure (BiPAP): delivers CPAP but also senses when an
inspiratory effort is being made and delivers a higher pressure during inspiration. Pressure
returns to CPAP level when flow stops.
- Increased positive pressure unloads diaphragm during inspiration thus decreasing the
work of breathing.
- Can be used for chronic respiratory failure due to neuromuscular problems or chest
wall abnormalities.
5. Level of Care:
- VA: ICU or ICU boarder. MUST have Pulmonary consult.
- UCSD: nasal bipap in IMU. Otherwise, ICU. If BiPAP/CPAP is being used to treat respiratory
failure, you should have the ICU team involved or at least aware.
6. Other references:
- www.ccmtutorial.com
- Review of indications and data for NIPPV. Chest 2003; 124: 699-713.
- Non-invasive Positive Pressure Ventilation. Am J Med 2005; 118: 584-91.
18
19
ARDS AND ACUTE LUNG INJURY
Definitions
1. Acute lung injury (ALI) = a syndrome of acute and persistent lung inflammation with
increased vascular permeability. Characterized by three clinical features:
- PaO2/FiO2 = 201-300 mmHg (regardless of PEEP).
- No evidence of left atrial pressure (pulmonary capillary wedge pressure < 18 mmHg).
- Bilateral radiographic infiltrates.
2. Acute Respiratory Distress Syndrome (ARDS) = meets above criteria for ALI, except
PaO2/FiO2 200 mmHg. Additionally:
- For both ALI and ARDS, the degree of gas exchange disturbance does not correlate with the
extent of the underlying pathology.
- Severity of hypoxia on presentation does not predict the clinical course or survival.
- Most cases of ARDS require positive pressure mechanical ventilation. Mechanical ventilation
itself, however, may cause additional lung injury barotrauma (i.e. pneumothorax) or
ventilator-associated lung injury (VALI) from over-distension of non-diseased lung regions.
Lung Protective Ventilation Strategies
1. ARDS Network Trials: low tidal volume ventilation decreased mortality in ALI/ARDS (ARMA
trial). Low PEEP in ALI/ARDS had similar outcomes as higher PEEP (ALVEOLI trial).
- Tidal volume of 6 mL/kg ideal body weight, plateau airway pressure 30 cmH2O
- Average PEEP approximately 9 cmH2O
- ARMA Trial: Ventilation with Lower Tidal Volumes as Compared with Traditional Tidal
Volumes for Acute Lung Injury and the Acute Respiratory Distress Syndrome. N Engl J Med
2000. 342(18): 1301-8.
- Multicenter randomized trial involving 861intubated patients with ALI/ARDS comparing
traditional ventilation treatment with initial tidal volume 12 mL/kg predicted body weight
and plateau pressure 50 cmH2O vs. tidal volume 6 mL/kg predicted body weight and
plateau pressure 30 cmH2O.
- Mortality lower in the lower tidal volume group than in traditional tidal volume group
(31.0% vs. 39.8%, p = 0.007; ARR 8.8%, NNT 11).
- By day 28, 65.7% breathing without assistance and had 1211 ventilator-free days in
low tidal volume group, compared with 55.0% and 1011 days in traditional tidal volume
group (p < 0.001 and p = 0.007, respectively).
- Conclusion: Using tidal volume of volume 6 mL/kg predicted body weight and plateau
pressure 30 cmH2O reduces mortality, decreased time on ventilator, and increased
number of ventilator-free days compared to traditional ventilation volumes.
- ALVEOLI Trial: Higher versus Lower Positive End-Expiratory Pressures in Patients with the
Acute Respiratory Distress Syndrome. N Engl J Med 2004; 351(4): 327-36.
- Multicenter randomized trial involving 549 intubated patients with ALI/ARDS
comparing low PEEP (average 8.33.2 cmH2O) vs. higher PEEP (average 13.2
cmH2O) (p < 0.001). Tidal volume 6 mL/kg predicted body weight with plateau
pressure 30 cmH2O.
- Rates of death before hospital discharge 24.9% in low PEEP group vs. 27.5% in
higher PEEP group (p = 0.48).
- By day 28, low PEEP group with 14.510.4 days breathing unassisted vs. 13.810.6
days in high PEEP group (p = 0.50).
- Conclusion: In ventilated patients with ALI/ARDS, clinical outcomes similar whether
low and high PEEP used, if tidal volume 6 mL/kg predicted body weight and plateau
pressure 30 cmH2O.
2. Permissive hypercapnea: not a ventilatory strategy per se, but is a common consequence of
low tidal volume ventilation. This is acceptable if oxygenation is maintained. Contraindications
include: Predisposition to increased intracranial pressure, hemodynamic instability, and
treatment with |-blockers (adequate catecholamine response is required to maintain
hemodynamic stability during hypercapnea).
20
HYPOXIA
Evaluation: Get other vitals and see patient to assess the level of distress. Many causes overlap
with the Shortness of Breath section (next), but important to think of mechanism.
1. Identify Underlying Cause: Anything creating an Aa-gradient (preventing blood flow or
preventing air flow to alveoli) or depressing Respiratory Drive.
2. Evaluate the Patient: for below causes of SOB. Assess ability to protect airway if no gag,
call ICU resident immediately for possible intubation (see below). Examine other cranial nerves
(e.g. blown pupil concerning for herniation, fixed dilated pupils concerning for opiate overdose).
3. If not dyspneic: Evaluate medications and timing of meds given, also look at renal/hepatic
function for clearance issues.
4. Give Narcan: For suspected Narcotic Overdose (remember the short half-life: may need to
start a gtt).
5. Workup: stat ABG, chem panel, CXR, cardiac markers, ECG, consider AMS work-up.
6. When to intubate: You should have called your resident by now, but if patient looks like tiring
out (persistently tachypneic, retracting, pCO2 up-trending) call ICU team for evaluation.
Respiratory failure is defined as PO2 <50 or PCO2 >50 (exception: chronic CO2 retainers), pH
<7.30, almost all with pH <7.20 (acute change) will require intubation. However, with
resident/ICU input, you may consider a trial of NIPPV if not contraindicated (i.e. not obtunded).
21
SHORTNESS OF BREATH
Evaluation: Get vitals (including oxygen sats), see patient to assess the level of distress.
Things Not to Miss:
1. PE: Check if on proper prophylaxis to stratify suspicion. Check O2 sats, ABG, D-dimer (only
useful to r/o PE if low pre-test probability), EKG for classic S1/Q3/T3, but most often see sinus
tachycardia, and if high suspicion order a CTPA vs. V/Q scan. If you cant give contrast (i.e.
high creatinine) or V/Q (COPD or its midnight) and suspicion high, can treat presumptively with
heparin gtt/enoxaparin 1mg/kg bid. Consider Lower Extremity Dopplers.
2. Lung collapse (mucus plugging): Patients with pneumonia and poor ability to clear secretions.
ON EXAM: Decreased breath sounds with dullness to percussion in focal region, and tracheal
deviation toward region. Get CXR. Call RT for aggressive suctioning & CPT. If collapse is
strongly suspected, call pulm for bronchoscopy.
3. Pneumothorax: Consider risk factors (recent central line, thoracic procedure, right sided
endocarditis, and PE): ON EXAM: Absent breath sounds with hyperresonance to percussion in
region, and tracheal deviation away from region. If evidence of tension PTX: DONT WAIT
FOR CXR emergently decompress with 14 or 16 gauge needle in 2nd intercostal space in
midclavicular line (to avoid internal thoracic artery). Call CT Surg/Pulm for chest tube
placement (stable patients will still need if PTX >20% of lung field).
4. Aspiration: Witnessed? Consider risk factors (emesis, AMS, seizures, ETOH). Keep HOB
elevated while waiting for CXR. Debate continues as to microbiology of aspiration PNA. In
general, use Abx w/ good gram negative coverage (Zosyn/Ceftaz) and consider adding
anaerobic coverage (Clinda/ Flagyl) for pts w/ poor dentition, and if hospital stay >3 days, add
nosocomial coverage (Vanc and Cipro). FYI: Not all aspirations are the same. Be sure to
also consider aspiration pneumonitis, which usually resolves within 24-48 hrs without antibiotic
treatment, however primary team can always reassess in AM and d/c Abx.
5. COPD/Asthma: (use care with O2 as can cause decreased respiratory drive and CO2
retention: goal saturations 90 - 92%) consider IV steroids with Combivent (COPD) or
albuterol (asthma) MDI (if tachycardic with albuterol can substitute with Levalbuterol).
6. Cardiac: Ischemia can cause acute rise in pulm pressures & subsequent SOB. Always beware
of flash pulmonary edema from papillary muscle rupture. Watch for CHF (use IV lasix and
raise head of bed) and tamponade look for signs of R sided failure (JVD, LE Edema).
7. Other: Metabolic Acidosis, Anxiety, and Pain are also common causes of SOB, but other more
serious causes should be excluded first. Sepsis can induce tachypnea in attempt to blow off
CO2 and compensate. Be vigilant of this in the setting of increased RR with good O2
saturations.
ANAPHYLAXIS
Evaluation and Treatment:
- Start with ABCsAirway, Breathing, Circulation
- EPINEPHRINE 0.5 cc of 1:1000 IM or SQ
- Solumedrol 125mg IV x 1; H-2 blocker IV x 1; Benadryl 50mg IV x 1
- Consider albuterol nebulizers if the patient is wheezing or stridorous
- Close observation and call resident f/b anesthesia for intubation if necessary
22
SEPSIS
SIRS (2 of 4)
- Temp > 100.3 or < 96.8
- Heart rate > 90 bpm
- Respiratory Rate > 20 or PaCO2 < 32
- WBC > 12 K or < 4K or > 10% bands
Suspected Infection
Venous Lactate > 35 mg/DL or
> 1 organ dysfunction
Initiate Sepsis Pathway
Corticosteroid Evaluation /
Replacement (B)
Blood Cultures / Early Broad
Spectrum Antibiotic Initiation
Early Goal Directed Therapy
(A)
If ARDS,
LowVolume Ventilation (E)
Xigris Eligibility (C)
Tight Glycemic Control (D)
Systolic Blood Pressure < 90
after fluid bolus
Sepsis Pathway
23
Definitions of Sepsis
1. Bacteremia: Positive blood cultures without evidence of systemic inflammation.
2. Systemic inflammatory response syndrome (SIRS): a widespread inflammatory response
to a variety of severe clinical insults. Must have two or more of the following:
- Temperature > 38.0C (100.4F) or < 36.0C (96.8F)
- Heart rate > 90
- Respiratory rate > 20 or PaCO2 < 32 mmHg
- WBC > 12,000 or < 4000 or > 10% bands
3. Sepsis: clinical signs describing SIRS along with suspected infection
4. Severe sepsis: sepsis associated with organ dysfunction, hypoperfusion, or hypotension (SBP
< 90 mmHg or decrease of 40 mmHg from baseline). For example: lactic acidosis (lactate > 35
mg/dL) or evidence of > 1 organ system dysfunction (e.g. oliguria or acute AMS)
5. Septic shock: severe sepsis with hypotension despite adequate fluid resuscitation (> 20-30
mL/kg) combined with perfusion abnormalities and requirement of inotropic or vasopressor
support.
6. Multiple organ dysfunction syndrome: presence of altered organ function in an acutely ill
patient such that homeostasis cannot be maintained without intervention. End of SIRS/sepsis
spectrum.
- Increasing abnormalities in the following organ-specific parameters are correlated with a
higher mortality: PaO2/FiO2, creatinine, platelet count, Glasgow Coma Scale, total bilirubin,
pressure-adjusted HR (
HR CVP
MAP
)
EARLY GOAL DIRECTED THERAPY
Management of severe sepsis/septic shock: Based on a randomized trial of 263 pts comparing
standard therapy vs. early goal directed therapy (EGDT) in pts with suspected sepsis. The in-hospital
morality rate was 30.5% in the EGDT group vs. 46.5% in the standard group (NNT = 6). Inclusion
criteria included at least 2 of 4 criteria for systemic inflammatory response syndrome (SIRS) plus
either lactate > 4 mmol/L (35 mg/dL) or SBP < 90 mmHg after 20-30 mL/kg fluid resuscitation. The
initial EGDT guidelines were published in 2004 and updated in 2008. Start EGDT within six hours of
diagnosis. All pts will need an arterial or central line in order to monitor BP/CVP and to send blood for
central or mixed venous O2 sat.
1. Initial resuscitation: should be started in the first six hours (1C). Can use either crystalloids
or colloids (no difference). Goals are (1C):
- CVP 8-12 mmHg (12-15 mmHg if mechanically ventilated or pre-existing decreased
ventricular compliance)
- MAP > 65 mmHg
- Urine output 0.5 mL/kg/hr
- Central venous O2 sat 70% or mixed venous O2 sat 65%. If venous O2 sat not met,
then can give more fluid, transfuse PRBCs to Hct 30%, or start dobutamine drip (max 20
mcg/kg/min) (2C)
2. Diagnosis: culture everything immediately (1C). Can perform imaging studies promptly if safe
to do so (1C).
3. Antibiotics: begin broad-spectrum IV antibiotics as early as possible and within the first
hour of recognizing severe sepsis (1D) and septic shock (1B).
- Vancomycin + Zosyn, Imipenem or 3rd/4th generation cephalosporin, etc. (1B)
- Anti-fungals or anti-virals when indicated (1B)
- Reassess regimen daily to optimize efficacy, reduce toxicity, and prevent resistance (1C)
- Consider combination therapy in Pseudomonas infections (2D) or in neutropenia (2D).
- Duration of therapy usually 7-10 days, but can be longer if response is slow (1D)
- Stop antibiotics if cause is non-infectious (1D)
4. Source control: find the focus of infection as soon as possible (1C). If it is potentially infected
(lines, etc), remove it (1C). If there is an abscess, necrotic tissue, etc, then take care of it as
soon as resuscitation is complete (1C).
24
5. Pressors and Inotropes (please also refer Pressors section): maintain MAP 65 mmHg (1C)
- Norepinephrine and dopamine are the preferred agents in septic shock (1C)
- Consider Vasopressin 0.03-0.04 units/min (may decrease stroke volume) (2C)
- Consider Epinephrine if blood pressure poorly responsive to norepinephrine or dopamine
(2B)
- Dobutamine in myocardial dysfunction/low cardiac output (after adequate fluid resuscitation
and left ventricular end diastolic pressure) in combination with vasopressors (1C)
6. Steroids (please also refer to Corticosteroids in the ICU section): do not use corticosteroids to
treat sepsis in absence of shock unless patient has endocrine need or is steroid dependent
(1D)
- Consider IV hydrocortisone if hypotension poorly responsive to adequate fluids and
vasopressors (2C)
- CORTICUS 2008 study demonstrated that hydrocortisone did not improve survival or
reversal of shock in septic shock. However, in those patients who did have eventual reversal
of shock, hydrocortisone did hasten the reversal.
7. Activated protein C (please also refer to Activated Protein C section): patients with low risk of
death (APACHE II <24 or one organ failure) should not receive APC (1A).
8. Blood products: TRICC trial suggested that in absence of significant CAD, acute hemorrhage
or lactic acidosis, transfuse for target Hgb 7.0-9.0 (1B)
- Transfuse to Hct 30% in first 6 hours if mixed venous O2 sat < 70% (2C)
- Give FFP if coags abnormal AND bleeding OR procedure planned (2D)
- Give platelets if < 5000 or < 30,000 and bleeding or < 50,000 and invasive procedure
planned (2D)
- Even in the setting of ischemic cardiac disease, if hemodynamically stable, studies suggest
that transfusions are associated with INCREASED mortality. More definitive studies pending.
9. Sepsis-induced ALI (please also refer to ARDS/ALI section): follow ARDSnet protocol
- Conservative fluid strategy if no evidence of tissue hypoperfusion (1C)
10. Sedation, analgesics, and paralytics (please also refer to Sedatives section):
- Use intermittent boluses or continuous infusions, and have daily sedation vacations (1B)
- Avoid neuromuscular blockade if possible (prolonged skeletal muscle weakness) (1B)
11. Glycemic control: insulin drip with target glucose < 150-200 mg/dL (2C)
- a study by Van den Berghe et al. (2001) showed that tight glucose control (glucose 80-110
mg/dL) with intensive insulin therapy reduced morbidity in all medical ICU patients, but only
reduced mortality in the subset of patients that stayed in the ICU > 3 days (these patients
could not be identified before entering the ICU). In contrast, a study by Van den Berghe
showed a morbidity and mortality benefit in all patients in a surgical ICU.
- The NICE-SUGAR Study (2009) showed that intense glucose control (target of 81 to 108)
INCREASED mortality for adult ICU patients compared with conventional glucose control
(target of 180 or less).
- D5 or D10 glucose infusions or enteral feeding and glucose monitoring while receiving insulin
drip
12. Renal replacement: CVVHD or intermittent HD as needed (see Nephrology section) (2B)
13. Bicarbonate therapy: No evidence that bicarbonate improves outcomes for pH > 7.15 and may
actually worsen outcome in lactic acidosis. Do not use for purpose of improving hemodynamics
or reducing vasopressors requirements (1B).
14. DVT prophylaxis: low-dose UFH or LWMH if not contraindicated (1A). If contraindicated, use
compression stockings or intermittent compression device (1A).
15. Stress ulcer prophylaxis: use H2 blocker (1A) or PPI (1B)
16. Advance care planning: early and frequent discussions with patient and family. Describe
likely outcomes and set realistic expectations (1D)
References: NEJM 2009; 360(13): 1283-1297, NEJM 2008; 358(2): 111-24, NEJM 2006; 354(5): 449-61, NEJM 2001; 345(19): 1359-67.
25
CORTICOSTEROIDS
Use in the ICU
The incidence of adrenal insufficiency (AI) in the ICU is ~30% and even higher in patients with septic
shock. Untreated, AI in the ICU has a high mortality. One should suspect AI in any septic patient that
does not hemodynamically respond to adequate fluid resuscitation (20-30 mL/kg) and becomes
pressor dependent.
1. Septic patients with known AI or at high risk for AI (on chronic steroids) should be started
empirically on hydrocortisone 50-100mg IV Q6-8 hr*.
2. For all others patients, check a random serum cortisol level.
- > 34 mcg/dL = appropriate response in a severely stressed patient no steroids needed
- < 15 mcg/dL = abnormal response start hydrocortisone
- 15-34 mcg/dL* = equivocal response corticotropin (cosyntropin) stimulation test
3. Corticotropin stimulation test: measure serum cortisol give corticotropin IV 250mcg
measure serum cortisol 30-60 min after administration
- If you want to give steroids before the random cortisol level use dexamethasone 4-10mg IV
- - Change in serum cortisol < 9 mcg/dL = relative AI start hydrocortisone
4. Also consider adding fludrocortisone 50mcg po/ngt QD
- Note that a study by Sprung et al published in 2008 demonstrates that hydrocortisone
therapy in septic shock does not provide for a survival benefit, including those patients who
did not appropriately respond to corticotropin. However, hydrocortisone is still widely used.
- Cortisol level of 15-34 mcg/dL are suggested by Cooper and Stewart. Your attending may
disagree, as do other experts, and consider > 25-30 mcg/dL normal.
References: Marik PE and Zaloga GP. Adrenal Insufficiency in the Critically Ill: a New Look at an Old Problem. Chest 2002; 122(5): 1784-96. Keh D
and Sprung CL. Use of Corticosteroid Therapy in Patients with Sepsis and Septic Shock: an Evidence-Based Review. Crit Care Med 2004; 32(11
Suppl): S527-33. Cooper MS and Stewart PM. Corticosteroid Insufficiency in Acutely Ill Patients. N Engl J Med 2003; 348(8): 727-34. Sprung CL, et
al. Hydrocortisone Therapy for Patients with Septic Shock. N Engl J Med 2008; 358(2): 111-24.
ACTIVATED PROTEIN C (DROTRECOGIN ALPHA)
1. PROWESS: randomized, double-blinded, placebo-controlled, multicenter trial (2001) of 1690
patients with severe sepsis
- showed mortality benefit in using activated protein C (drotrecogin alfa, Xigris)
- 28 day mortality was significantly decreased in the activated protein C group: 24.7% vs
30.8% in the control group, a relative risk reduction 19% (NNT=16)
- given as continuous infusion of 24 mcg/kg/hr x 96 hours
- increased incidence of severe bleeding in the study group: 3.5% vs 2% (p=0.06)
- defined as intracranial hemorrhage, life threatening bleed, or transfusion requirement
of > 3 PRBCs/day x 2 consecutive days.
- benefit was only shown in patients at high risk for death
- APACHE II score > 24 AND > 1 organ system dysfunction.
2. ENHANCE: (2005)
- confirmed the results of the PROWESS trial, including the increased risk of bleeding.
- patients treated within the first 24 hours of organ dysfunction had higher survival rates
than patients treated later (22.9% vs. 27.4% mortality)
3. XPRESS: (2007)
- prophylactic heparin safe to use with activated protein C and did not increase the risk of
bleeding.
*Note: At UCSD you must fill out the pre-printed form to use activated protein C
26
- PROWESS Inclusion criteria:
- Known infection, or suspected infection
- At least 3 of the 4 criteria for SIRS
- One of the following criteria for organ system dysfunction:
- Cardiovascular: SBP < 90 or MAP < 70 despite adequate fluids, or use of
pressors
- Respiratory: PaO2/FiO2 < 200 (or < 250 in presence of other system
dysfunction)
- Renal: UOP < 0.5 mL/kg/hr despite adequate fluids
- Hematologic: platelets < 80,000 or decreased by 50% in past 3 days
- Metabolic acidosis: pH < 7.30 and serum lactate > 4.2 mEq/L
- PROWESS exclusion criteria:
- Pregnant or breast-feeding
- Age < 18 or weight > 135 kg
- Platelets < 30,000
- INR > 3
- Conditions with increased risk of bleeding (recent or impending surgery, recent
trauma, GI bleed in past 6 weeks, h/o head trauma, CVA in past 3 months, intracranial
AVM, surgery, or mass lesions)
- Known hypercoagulable state
- Acute pancreatitis
- Cirrhosis, jaundice, portal HTN, or chronic ascites
- LP in past 12 hours
- Heparin in past 8 hrs, LMWH in past 12 hrs, warfarin within past 7 days, elevated INR,
thrombolytics within past 3 days, ASA > 650 mg/day for past 3 days, GP IIb/IIIa
inhibitor within past 7 days, antithrombin III in past 12 hrs, protein C within past 24 hrs
(prophylactic doses of heparin OK)
- Exclusions in original study but not considered absolute: AIDS with CD4 < 50, ESRD
on dialysis, h/o bone marrow, lung, liver, or small bowel transplant
References: Bernard GR, et al. Efficacy and safety of recombinant human activated protein C for severe sepsis. N Engl J Med 2001; 344(10): 699-709.
Laterre P. Clinical trials in severe sepsis with drotrecogin alfa (activated). Crit Care 2007; 11(Suppl 5): S5.
PRESSORS
**Always fill the tank first** i.e. large volume fluid resuscitation
1. Norepinephrine (Levophed) and dopamine are preferred agents in septic shock.
Phenylephrine (Neosynephrine) is used once the patient is on the maximum doses of
norepinephrine and dopamine.
2. Dopamine may be useful if patient is bradycardic.
2. Vasopressin is useful as a second agent, not first line.
3. Epinephrine is used for anaphylactic shock and codes.
4. Dobutamine and isoproteronol are used only in cardiogenic shock
5. Phenylephrine is very useful for sedation associated hypotension peri-operatively and in
liver patients.
Mechanism Major action Dose Use
Phenylephrine
(Neosynephrine)
Pure o | SVR, /| CO 10-200 mcg/min Septic shock, neurogenic shock,
anesthesia-induced hypotension
Norepinephrine
(Levophed)
o >> | | SVR, /| CO 0.01-0.3
mcg/kg/min
Septic shock
Epinephrine |1 >> |2, o | CO, + SVR (low dose)
/| SVR (high dose)
0.01-0.5
mcg/kg/min
Anaphylactic shock, code situations
(last ditch effort)
Dopamine D, |1 | CO (inotrope) 2-10 mcg/kg/min Septic shock, cardiogenic shock
|1, o > D | SVR 10-20 mcg/kg/min Septic shock, cardiogenic shock
Dobutamine |1 > |2 | CO, + SVR 2-50 mcg/kg/min Cardiogenic shock
Isoproterenol |1, |2 | CO (chronotrope), +
SVR
0.015-0.5
mcg/kg/min
Only cardiogenic shock due to
bradycardia
Vasopressin ADH analogue | SVR 0.04 U/min Septic shock (useful as second
agent)
- o vasoconstriction increased SVR, blood pressure
- |1 increased heart rate and contractility increased CO
- |2 vasodilation slight decrease SVR; also bronchodilation
- D vasodilates kidney and viscera increased renal perfusion
27
PARALYTICS
S
u
c
c
i
n
y
l
c
h
o
l
i
n
e (
U
l
t
r
a
s
h
o
r
t
-
i
r
r
e
v
e
r
s
i
b
l
e
)
P
a
n
c
u
r
o
n
i
u
m
(
L
o
n
g
)
R
o
c
u
r
o
n
i
u
m
(
I
n
t
e
r
m
e
d
i
a
t
e
)
C
i
s
a
t
r
a
c
u
i
r
i
u
m
(
I
n
t
e
r
m
e
d
i
a
t
e
)
V
e
c
u
r
o
n
i
u
m
(
I
n
t
e
r
m
e
d
i
a
t
e
)
P
a
r
a
l
y
t
i
c
s
(
n
e
u
r
o
m
u
s
c
u
l
a
r
b
l
o
c
k
i
n
g
a
g
e
n
t
s
)
1
.
0
-
1
.
5
0
.
0
6
-
0
.
1
0
.
6
0
.
1
5
-
0
.
2
0
.
0
8
-
0
.
1
0
I
n
i
t
i
a
t
i
n
g
D
o
s
e
(
m
g
/
k
g
)
0
.
0
4
-
0
.
0
7
0
.
0
1
-
0
.
0
1
5
0
.
1
-
0
.
2
0
.
0
3
0
.
0
1
-
0
.
0
1
5
I
n
t
e
r
m
i
t
t
e
n
t
m
g
/
k
g
M
a
i
n
t
e
n
a
n
c
e
D
o
s
e
N
/
A
1
.
0
m
c
g
1
0
-
1
2
M
c
g
1
.
0
-
3
.
0
m
c
g
0
.
8
-
1
.
0
m
c
g
C
o
n
t
i
n
u
o
u
s
i
n
f
u
s
i
o
n
m
c
g
/
k
g
/
m
i
n
P
l
a
s
m
a
c
h
o
l
i
n
e
s
t
e
r
a
s
e
s
K
i
d
n
e
y
>
>
L
i
v
e
r
L
i
v
e
r
m
e
t
a
b
o
l
i
s
m
a
n
d
c
l
e
a
r
a
n
c
e
;
*
R
e
n
a
l
e
l
i
m
i
n
a
t
i
o
n
L
i
v
e
r
>
>
K
i
d
n
e
y
*
L
i
v
e
r
m
e
t
a
b
o
l
i
s
m
;
R
e
n
a
l
e
l
i
m
i
n
a
t
i
o
n
H
o
f
m
a
n
n
d
e
g
r
a
d
a
t
i
o
n
(
n
o
r
e
n
a
l
/
l
i
v
e
r
e
l
i
m
i
n
a
t
i
o
n
)
L
i
v
e
r
>
>
K
i
d
n
e
y
*
L
i
v
e
r
m
e
t
a
b
o
l
i
s
m
a
n
d
c
l
e
a
r
a
n
c
e
;
r
e
n
a
l
e
l
i
m
i
n
a
t
i
o
n
M
o
d
e
o
f
E
l
i
m
i
n
a
t
i
o
n
M
a
l
i
g
n
a
n
t
h
y
p
e
r
t
h
e
r
m
i
a
,
i
n
c
r
e
a
s
e
d
i
n
t
r
a
c
r
a
n
i
a
l
p
r
e
s
s
u
r
e
,
c
a
r
d
i
a
c
a
r
r
h
y
t
h
m
i
a
s
,
i
n
c
r
e
a
s
e
d
o
c
u
l
a
r
p
r
e
s
s
u
r
e
I
n
c
r
e
a
s
e
d
h
e
a
r
t
r
a
t
e
,
b
l
o
o
d
p
r
e
s
s
u
r
e
a
n
d
p
e
r
i
p
h
e
r
a
l
v
a
s
c
u
l
a
r
r
e
s
i
s
t
a
n
c
e
A
r
r
h
y
t
h
m
i
a
s
,
t
r
a
n
s
i
e
n
t
h
y
p
o
-
a
n
d
h
y
p
e
r
t
e
n
s
i
o
n
,
t
a
c
h
y
c
a
r
d
i
a
,
b
r
o
n
c
h
o
s
p
a
s
m
,
r
a
s
h
B
r
a
d
y
c
a
r
d
i
a
,
h
y
p
o
t
e
n
s
i
o
n
,
f
l
u
s
h
i
n
g
,
b
r
o
n
c
h
o
s
p
a
s
m
,
r
a
s
h
M
a
l
i
g
n
a
n
t
h
y
p
e
r
t
h
e
r
m
i
a
A
d
v
e
r
s
e
E
f
f
e
c
t
s
T
r
a
d
i
t
i
o
n
a
l
l
y
d
r
u
g
o
f
c
h
o
i
c
e
f
o
r
i
n
t
u
b
a
t
i
o
n
.
C
a
n
c
a
u
s
e
c
e
l
l
s
t
o
r
e
l
e
a
s
e
p
o
t
a
s
s
i
u
m
s
o
a
v
o
i
d
i
n
p
a
t
i
e
n
t
s
a
t
r
i
s
k
f
o
r
h
y
p
e
r
k
a
l
e
m
i
a
D
r
u
g
o
f
c
h
o
i
c
e
i
n
p
a
t
i
e
n
t
s
w
i
t
h
n
o
r
m
a
l
r
e
n
a
l
/
h
e
p
a
t
i
c
f
u
n
c
t
i
o
n
a
n
d
s
t
a
b
l
e
h
e
m
o
d
y
n
a
m
i
c
s
.
H
a
s
a
c
t
i
v
e
m
e
t
a
b
o
l
i
t
e
s
.
U
s
e
c
a
u
t
i
o
u
s
l
y
i
n
p
a
t
i
e
n
t
s
w
i
t
h
h
e
p
a
t
i
c
d
i
s
e
a
s
e
,
s
e
v
e
r
e
o
b
e
s
i
t
y
,
e
l
e
c
t
r
o
l
y
t
e
i
m
b
a
l
a
n
c
e
s
,
n
e
u
r
o
m
u
s
c
u
l
a
r
d
i
s
e
a
s
e
,
a
n
d
a
l
t
e
r
e
d
c
i
r
c
u
l
a
t
i
o
n
c
a
u
s
e
d
b
y
C
V
d
i
s
e
a
s
e
,
o
l
d
a
g
e
o
r
e
d
e
m
a
t
o
u
s
s
t
a
t
e
s
.
A
p
p
r
o
p
r
i
a
t
e
f
o
r
r
e
n
a
l
/
h
e
p
a
t
i
c
p
a
t
i
e
n
t
s
.
M
i
n
r
i
s
k
o
f
h
i
s
t
a
m
i
n
e
-
a
s
s
o
c
h
y
p
o
t
e
n
s
i
o
n
L
e
a
s
t
a
d
v
e
r
s
e
C
V
e
f
f
e
c
t
.
D
O
C
i
n
C
V
D
o
r
h
e
m
o
d
y
n
a
m
i
c
i
n
s
t
a
b
i
l
i
t
y
M
i
n
r
i
s
k
o
f
h
i
s
t
a
m
i
n
e
-
a
s
s
o
c
h
y
p
o
t
e
n
s
i
o
n
.
H
a
s
a
c
t
i
v
e
m
e
t
a
b
o
l
i
t
e
s
S
p
e
c
i
a
l
C
o
n
s
i
d
e
r
a
t
i
o
n
s
28
SEDATIVES
F
e
n
t
a
n
y
l
H
y
d
r
o
m
o
r
p
h
o
n
e
M
o
r
p
h
i
n
e
P
r
o
p
o
f
o
l
(
D
i
p
r
i
v
a
n
)

f
o
r
r
a
p
i
d
a
w
a
k
e
n
i
n
g
,
n
o
a
n
a
l
g
e
s
i
c
p
r
o
p
e
r
t
i
e
s
L
o
r
a
z
e
p
a
m
(
A
t
i
v
a
n
)
M
i
d
a
z
o
l
a
m
(
V
e
r
s
e
d
)
S
e
d
a
t
i
v
e
s
5
0
-
2
0
0
m
c
g
I
V
b
o
l
u
s
2
-
4
m
g
I
V
b
o
l
u
s
0
.
0
5

0
.
2
m
g
/
k
g
I
V
o
v
e
r
5
-
1
5
m
i
n
0
.
5
m
g
/
k
g
/
h
r
;
t
i
t
r
a
t
e
i
n
i
n
c
r
e
m
e
n
t
s
o
f
0
.
3
-
0
.
6
m
g
/
k
g
/
h
r
e
v
e
r
y
5
-
1
0
m
i
n
1
-
4
m
g
I
V
b
o
l
u
s
2
-
8
m
g
I
V
b
o
l
u
s
U
s
u
a
l
L
o
a
d
i
n
g
D
o
s
e
5
0
-
3
0
0
m
c
g
/
h
r
1
-
2
m
g
/
h
r
4
-
6
m
g
/
h
r
1
0
m
g
/
h
r
0
.
5
-
3
m
g
/
k
g
/
h
r
2
-
5
m
g
/
h
o
u
r
0
.
5
-
2
-
5
m
g
/
h
o
u
r
U
s
u
a
l
M
a
i
n
t
e
n
a
n
c
e
D
o
s
e
L
i
v
e
r
M
e
t
a
b
o
l
i
s
m
.
R
e
n
a
l
e
l
i
m
i
n
a
t
i
o
n
L
i
v
e
r
M
e
t
a
b
o
l
i
s
m
L
i
v
e
r
M
e
t
a
b
o
l
i
s
m
.
R
e
n
a
l
e
l
i
m
i
n
a
t
i
o
n
L
i
v
e
r
M
e
t
a
b
o
l
i
s
m
.
R
e
n
a
l
e
l
i
m
i
n
a
t
i
o
n
G
l
u
c
o
r
o
n
i
d
a
t
i
o
n
H
e
p
a
t
i
c
C
Y
P
4
5
0
M
o
d
e
o
f
E
l
i
m
i
n
a
t
i
o
n
A
L
L
E
R
G
Y
(
2
/
2
H
i
s
t
a
m
i
n
e
r
e
l
e
a
s
e
)
,
R
e
s
p
i
r
a
t
o
r
y
D
e
p
r
e
s
s
i
o
n
,
M
u
l
t
i
f
o
c
a
l
m
y
o
c
l
o
n
u
s
,
s
e
d
a
t
i
o
n
,
c
o
n
s
t
i
p
a
t
i
o
n
,
N
/
V
H
Y
P
O
T
E
N
S
I
O
N
,
A
n
a
p
h
y
l
a
x
i
s
(
r
a
r
e
)
,
a
p
n
e
a
,
r
e
s
p
i
r
a
t
o
r
y
a
c
i
d
o
s
i
s
,
b
r
a
d
y
c
a
r
d
i
a
R
e
s
p
i
r
a
t
o
r
y
D
e
p
r
e
s
s
i
o
n
,
H
y
p
o
t
e
n
s
i
o
n
A
d
v
e
r
s
e
E
f
f
e
c
t
s
(
d
r
u
g
s
p
e
c
i
f
i
c
)
N
o
h
i
s
t
a
m
i
n
e
r
e
l
e
a
s
e
(
l
e
s
s
s
i
d
e
e
f
f
e
c
t
s
t
h
a
n
m
o
r
p
h
i
n
e
)
L
i
p
i
d
s
o
l
u
b
l
e
,
a
c
c
u
m
u
l
a
t
e
s
i
n
a
d
i
p
o
s
e
t
i
s
s
u
e
>
5
d
a
y
s

p
r
o
l
o
n
g
e
d
s
e
d
a
t
i
o
n
.
N
o
t
a
f
f
e
c
t
e
d
b
y
r
e
n
a
l
f
a
i
l
u
r
e
.
A
c
t
i
v
e
m
e
t
a
b
o
l
i
t
e
a
c
c
u
m
u
l
a
t
e
s
i
n
r
e
n
a
l
f
a
i
l
u
r
e
E
l
i
m
i
n
a
t
i
o
n
n
o
t
i
m
p
a
i
r
e
d
i
n
c
i
r
r
h
o
s
i
s
o
r
r
e
n
a
l
i
n
s
u
f
f
i
c
i
e
n
c
y
.
L
i
p
i
d
e
m
u
l
s
i
o
n
;
g
i
v
e
s
c
a
l
o
r
i
e
s
t
o
p
a
t
i
e
n
t
(
a
d
j
u
s
t
T
P
N
?
)
,
a
n
d
c
a
n
e
l
e
v
a
t
e
t
r
i
g
l
y
c
e
r
i
d
e
s
.
M
o
n
i
t
o
r
t
r
i
g
l
y
c
e
r
i
d
e
s
(
p
a
n
c
r
e
a
t
i
t
i
s
)
.
C
o
n
t
r
a
i
n
d
i
c
a
t
e
d
i
n
p
t
w
/
e
g
g
a
l
l
e
r
g
y
.
S
a
f
e
i
n
l
i
v
e
r
d
y
s
f
u
n
c
t
i
o
n
.
N
o
a
c
c
u
m
u
l
a
t
i
o
n
.
I
n
t
e
r
m
e
d
i
a
t
e
o
n
s
e
t
o
f
a
c
t
i
o
n
R
a
p
i
d
o
n
s
e
t
,
s
h
o
r
t
a
c
t
i
n
g
.
A
c
t
i
v
e
m
e
t
a
b
o
l
i
t
e
a
c
c
u
m
u
l
a
t
e
s
w
i
t
h
c
o
n
t
i
n
u
e
d
u
s
e
i
n
a
l
l
p
t
s
S
p
e
c
i
a
l
C
o
n
s
i
d
e
r
a
t
i
o
n
29
SEVERE METABOLIC ACIDOSIS
The initial therapeutic goal for patients with severe acidemia is to raise the systemic pH above 7.15-
7.2, a level at which dysrhythmias become less likely and cardiac contractility and responsiveness to
catecholamines (pressors) will be restored.
1. Treatment of the Underlying Cause: is usually adequate for correction. (e.g. treat DKA with
fluids and insulin drip and treat sepsis with fluids and antibiotics)
2. Bicarbonate therapy: Consider for pH < 7.15-7.2. The decision to give bicarbonate should
be based upon the pathophysiology of the specific acidosis, the clinical state of the patient,
and the degree of acidosis.
- In hyperchloremic acidosis (RTA and GI losses), the central problem is with the reabsorption
or regeneration of bicarbonate. In these conditions, therapy with bicarbonate makes
physiologic sense in severe acidosis.
- In renal failure, bicarbonate is renally lost, so it makes sense to replace it.
- There is a possible benefit for bicarbonate as a short term therapy for salicylate intoxication
to create an alkalemic environment to enhance toxin elimination.
- There is controversy regarding use of bicarbonate in lactic acidosis. It may only transiently
raise the serum bicarbonate concentration and cause an eventual worsening of metabolic
acidosis. Discuss with ICU Fellow/Attending if bicarbonate therapy is appropriate.
- For all cases of diabetic ketoacidosis, the role of bicarbonate is controversial, regardless of
the pH or bicarbonate level. Glaser et al suggested a possible increased risk of cerebral
edema.
- Complications associated with bicarbonate therapy:
- Hypernatremia, hypokalemia, hypocalcemia
- Fluid overload,
- Post-recovery metabolic alkalosis (as the excess lactate is converted back to
bicarbonate)
- Decreased oxygen availability (due to left-shift of O2-Hb dissociation curve with
increasing pH)
- Paradoxical CNS acidosis (CO2 diffuses through BBB while bicarbonate does not)
- Bicarbonate deficit = (Desired Bicarbonate Measured Bicarbonate) x Weight (kg) x 0.6
- The general recommendation is to replace only half of the total bicarbonate deficit
over several hours, then reassess.
3. Hemodialysis: for metabolic acidosis secondary to ingestions (e.g., salicylate, methanol,
ethylene glycol)
- Preferred treatment for patients with significant metabolic acidosis in the setting of renal
failure. Nephrology should be consulted.
4. Mechanical Ventilation: If the patient is on mechanical ventilation, ventilation can be
increased by increasing respiratory rate (maximum rate 35) or increasing tidal volume (peak
pressure maximum 35- 40). However, increasing the respiratory rate excessively can cause
decreased expiratory time and lead to CO
2
retention acidosis. Note, in ARDS, try not to
greatly exceed plateau pressure of 30.
30
VENT TROUBLESHOOTING
31
PULMONARY FUNCTION TESTING
R(E) = extraparenchymal restrictive disease
R(P) = parenchymal restrictive disease
O = obstructive disease
RV = residual volume
TLC = total lung capacity
32
Spirometry
Value
Obstructive
Disease
Restrictive Disease
Parenchymal Neuromuscular
Chest wall
deformity
TLC
Normal to
RV

Depends on
strength
Depends
on disorder
VC
to normal
FEV1/FVC
Normal to
Depends on
strength
Normal
MIP (max
insp.
Pressure)
Normal Normal Normal
COPD
Global initiative for chronic Obstructive Lung Disease (GOLD) guidelines, 2008
1. Indications for admission:
- Marked increased intensity of symptoms
- Severe background COPD
- Onset of new physical signs (cyanosis, edema, etc.)
- Failure of exacerbation to respond to initial medical management
- Significant comorbidities
- Frequent exacerbations
- New arrhythmias
- Older age
- Insufficient home support
- Diagnostic uncertainty
2. Assess severity:
- ABG
- PaO2 < 60 and/or SaO2 < 90% with/without PaCO2 > 50 on room air respiratory
failure
- pH < 7.36 + PaCO2 > 45-60 in a patient with respiratory failure indication for
intubation
- CXR (PA and lateral): to identify alternative diagnoses
- EKG: look for RVH, arrhythmia, or ischemia
- Lab tests: sputum culture and sensitivities, chem panel, and CBC with diff.
3. Therapy:
- Supplemental oxygen
- Bronchodilators combination therapy probably better than single agent
- albuterol 90mcg 4-8 puffs (MDI) or 2.5mg (neb) Q1-4h PRN [MDI equivalent to nebs]
- Ipratropium 18mcg 2 puffs (MDI) or 500mcg (neb) Q4h PRN
- Steroids: methylprednisolone 60-125mg IV Q6-12h or prednisone 40-60mg po Qday x 7-10
days (no need to taper if treatment less than 3 weeks)
- Antibiotics: indicated if intubated OR if increased dyspnea + increased sputum volume
+ increased sputum purulence
- (-) risk factors for Pseudomonas: moxifloxacin OR ceftriaxone OR cefotaxime
- (+) risk factors for Pseduomonas: cefepime OR ceftazidime OR Zosyn OR
levofloxacin
Risk factors = 4 courses of antibiotics in last year, recent hospitalization 2 days in past 90 days,
past Pseudomonas infection, FEV1 < 50% predicted
33
4. Chronic treatment for stable COPD:
CYSTIC FIBROSIS
Cystic fibrosis is a genetic disorder of the CFTR gene that presents with symptoms from birth.
However, given advances in caring for CF, patients are now living much longer. Adults often present
for acute care secondary to a CF exacerbation; acute, rather than chronic, care is discussed here.
Remember that CF is a multisystem disorder, not just pulmonary.
Most common bacteria isolated in CF patients respiratory cultures: P. aeruginosa (55%), MSSA
(52%), MRSA (19%), H. influenzae (17%), MDR P. aeruginosa (16%), Stenotrophomonas maltophilia
(13%), Burkholdria cepacia (3%)
1. Symptoms of an exacerbation:
- Increased cough, sputum production, or chest congestion
- Decreased exercise tolerance/increased dyspnea with exertion
- Increased fatigue
- Decreased appetite
- Shortness of breath or tachypnea
- Change in sputum appearance
- Fever
- Increased nasal congestion or drainage
2. Antibiotic therapy:
- PO for mild exacerbations: dicloxacillin, Augmentin, cephalexin, TMP-SMX, doxycycline, or
macrolide (may be resistant if receiving azithromycin chronically)
- IV for severe exacerbations, resistance to PO antibiotics, or PO failure
- Usually based on susceptibilities, but studies show in vitro susceptibilities dont matter
(Smith AL et al. Susceptibility Testing of Pseudomonas aeruginosa Isolates and
Clinical Response to Parental Antibiotic Administration: Lack of Association in Cystic
Fibrosis.
- Pseudomonas: TOBRAMYCIN or AMIKACIN or COLISTIN + [piperacillin,
ceftazidime, cefepime, meropenem, doripenem]
- MSSA: cefazolin or nafcillin
- MRSA: vancomycin or linezolid
- Burkholderia: meropenem or per sensitivities
- Usually treat for 2 weeks; PICC line may be helpful if the patient has no long-term
access like a port (but most do have a port).
- Remember to check serum peaks and troughs for aminoglycosides.
- Many patients may be on inhaled tobramycin (tobi) or colistin as an outpatient. This is usually
a one-month-on/one-month-off regimen. If the patient is on tobi/colistin at admission,
continue it.
34
3. Other Affected Systems: continue their outpatient medications
- PULM: oxygen, bronchodilators, mucolytics (DNAse, hypertonic saline, etc), chest
percussion therapy
- GI: enzymes (Pancrease, etc), acid suppressants (H2 blockers, PPI), bowel regimen
- FEN: vitamins (ADEK, etc), diet supplements (Ensure, etc)
- ENDO: insulin if diabetic (pancreatic failure)
- RENAL: may have kidney injury from repeated aminoglycoside use
- ID: antibiotics as above. Remember to check for access site (PICC line, port) infections.
Sinusitis may be present. Patients are at increased risk for allergic bronchopulmonary
aspergillosis.
- PSYCH: treat depression if present
References: Chest 2003; 123(5): 1495-1502.
35
CODE ALGORITHMS
PERFUSING PATIENT
36
ARRESTED PATIENT
37
CARDIOLOGY
CHEST PAIN AND ACUTE CORONARY SYNDROME
1. Cross Cover Clues:
- Differential: MI, dissection, PTX, PE, pericarditis, PNA, GERD, PUD, esophageal spasm,
esophagitis, anxiety.
- Primary goal is determine if it is life-threatening. Find out if CP is new, and if chest pain is
different. If h/o ACS is this the same pain they had before w/ MI?
- Look at the signout: does the patient have chronic/recurrent CP that has been evaluated?
- Recurrent CP still needs to be evaluated, but this may give you a sense of the source initially.
- LOOK AT THE PATIENT: often you get alarming pages that turn out to be nothing.
- Assess for the four major killer causes of chest pain: acute MI, aortic dissection, PE, and
tension pneumothorax.
- Vital Signs are vital: ask the nurse for them with the initial call including SaO2.
- Stratify urgency with vitals and history: if you suspect a cardiac cause, get an urgent EKG
and compare it to previous EKGs for changes.
- Initial workup, if serious cause suspected: EKG, STAT Cardiac Markers and serially, Chem
10, CBC, BNP, Coags, and CXR. If suspect PE, see section: Dyspnea
- Initial interventions: can give trials of sublingual NTG if sbp >90 mm Hg (does not rule out GI
if relieves) or a GI cocktail (Maalox 30 cc, 10 cc viscous lidocaine). NSAIDs if MSK etiology.
- If cardiac is suspected, and SL Nitro only temporarily relieves, move on to Nitro Paste; may
need Nitro gtt (call your resident or ICU/CCU team); assess likelihood of ACS and consider
asa, b-blocker, and heparin drip (call CCU early if you suspect ACS the fellow should be
involved if you are starting heparin gtt for ACS)
- If CXR suspicious for aortic dissection (widened mediastinum) or if there are different BPs in
each arm, get CT chest with contrast and alert CT surgery, NO ANTICOAGULATION.
History Keys: description of pain including length, quality, radiation, duration, relieving factors.
Functional status of patient, can he walk across a parking lot, up a flight of stairs? Increasing severity,
frequency? Prior ischemic disease, CHF, or arrhythmias? Hx of HL, HTN, DM, PVD. Tobacco use?
On aspirin? Allergy to contrast or shellfish? Track down previous EKG/Echo/Cath reports/CABG
reports.
2. Risk Stratification:
- TIMI score can be used to stratify chest pain patients
- Age over 65
- 3 or more risk factors for CAD [Diabetes, cigarette smoking, HTN (BP 140/90 mm Hg or
on antihypertensive medication), dyslipidemia, Family history of premature CAD (CAD in
male first-degree relative 55 or younger, CAD in female first-degree relative 65 or
younger, Age (men 45 years; women 55 years)]
- Established CAD (Angiographic stenosis of 50%)
- 2 or more angina episodes in past 24 hours
- ASA use in the last 7 days
- Elevated cardiac enzymes
- ST depression 0.5 mm
- TIMI Scale
- 0-2 = low risk
- 3-4 = intermediate risk
- 5-7 = high risk
*Negative cardiac markers 12hrs from onset of chest pain makes NSTEMI highly unlikely
3. Physical Exam: JVP, card (murmurs?), pulm (basilar crackles?), pulses in both arms,
peripheral edema, femoral pulses, rectal w/heme if starting heparin
4. EKG: see section below, but for STEMI you are looking for:
- EKG with >1mm ST elevation in >2 contiguous leads, OR
- Left bundle branch block, not known to be old
38
5. Initial Management for ACS: Antiplatelet, Antithrombotic and early revascularization.
Decrease O2 demand and increase coronary perfusion (manage HTN, pain, HR)
- ASA 325mg PO. Immediately in all patients, except for those with active hemorrhage or
allergy.
- Heparin gtt: use in all patients without contraindications (hemorrhage).
- BB: metop 5mg IV q2-5min x3. Goal to decrease myocardial demand. Follow this with PO
metoprolol. Contraindications: cardiogenic shock, hypotension, decompensated heart
failure, symptomatic bradycardia, 2nd or 3rd degree heart block w/o pacer, active wheezing.
- Activate cath lab: contact your fellow who will determine whether or not pt will need
intervention. He will also determine whether to start a GIIb/IIIa inhibitor the preferred
reperfusion therapy in pts presenting <24h after onset of symptoms.
- Control pain with NTG:
- SL NTG (400mcg) q5 min PRN chest pain, repeat PRN up to 3 doses and topical
Nitropaste (1-2 inches to chest wall)
- If pain unrelieved by above, or pain recurs, begin IV NTG drip (20-200mcg/min)
- You may consider morphine, however you need to be able to tell whether or not pt is
having cardiac pain. If you know cath lab is already going to intervene, then it is likely
OK.
6. Orders: ASA/Statin/BB/ACEI, Plavix?, Heparin/Lovenox (attn. dep), IIb/IIIa inhibitor (call
fellow); NPO p MN, ECHO next day, Labs CBC, CHEM10, coags, AM lipids, TSH, LFTs,
BNP
- Have all EKGs from admission (and old one if attainable); also CP patients need qAM EKG
- Patient must be NPO and receive no Lovenox on day of cath. A copy of the H&P MUST be in
the chart for the cath lab.
7. Sub-acute Management:
- monitor on tele
- ASA 81-325mg PO daily
- If received stent, Plavix 75mg PO daily continued at least 1 month for Bare Metal Stents, at
least 1 year for Drug-Eluting Stents
- Heparin gtt for 48h or until trops downtrending or pt revascularized (stent placed).
- Beta blocker: titrate up metoprolol to goal heart rate 55-60
- ACE Inhibitor: helps prevent remodeling.
- Statins: have shown benefit in the acute setting. Get a lipid panel immediately on admission,
as in the acute phase LDL levels will be lower by 25-50% for up to 6 weeks. Goal LDL<70.
- Smoking Cessation
- Exercise: goal 30-60 min of activity 5-7x/wk
- Diet: cardiac diet
8. Consents: get release of info for all pts, then copy and fax to multiple hospitals, we need cath
reports, EKGs, previous echos.
9. Device Interrogation: call rep phone# on signout sheet
EKGS
- Be sure to check out ECG Wave-Maven at http://ecg.bidmc.harvard.edu/maven/mavenmain.asp.
- Finding old EKGs at Hillcrest/Thornton
- Login to webcharts. Click on any patients chart. Click on EKG from the top menu.
Username: ad/emergency
Password: Emer!gency
1. Determine rate and rhythm: P-wave before QRS, upright P-wave lead II, III and aVF
2. Evaluate intervals:
- PR < 0.12-0.20sec <5 boxes
- QRS < 0.06-0.120sec <3 boxes
- QTc < 0.48sec, less than the R-R distance)
39
3. Determine the Axis:
- Normal: QRS upright in I and aVF
- LAD: QRS up in I, down in aVF, II
- RAD: QRS down in I, up in aVF
4. Evaluate for evidence of atrial and ventricular abnormalities:
- LVH: HTN, HOCM, AS/AI, coarctation of aorta
- R in aVL > 9mm (female); 11mm (male) - most specific
- R in aVL + S in V3 > 20mm (female); 28mm (male) - most sensitive
- S in V1 or V2 + R in V5 or V6 >35mm
- RVH: cor pulmonale, congenital, MS, TR
- RAD
- R:S in V1 > 1
- R in V1 > 7mm
- R:S in V5 or V6 < 1
- Combined Right and Left Ventricular Hypertrophy:
- Voltage Criteria for LVH + RAD
- LAE:
- Notched P-wave in II (
- Biphasic P-wave in V1 (terminal P-wave >1 small box)
- RAE:
- Peaked P-wave (>2.5mm) in lead II
- P-wave > 1.5mm in V1 or V2
5. Evaluate for evidence of bundle branch block:
- QRS > 0.12sec (incomplete block QRS 0.10-0.12sec)
- Look @ V1 & V6 draw vertical line down middle of QRS; portion to right of the vertical
line = LATE component of QRS.
- LBBB:
- Late V1 = downward; Late V6 = upward
- Broad, slurred, monophasic R wave in I, V5 & V6. Absence of septal Qs in I, V5 &
V6
- Displacement of ST and T wave opposite to major deflection of QRS +/- PRWP, LAD,
Qs in inferior leads
- LAFB:
- LAD
- Small q (qR) in I, aVL, small r (rS) in II, III, aVF
- LPFB:
- RAD
- Small r (rS) in I, aVL, small q (qR) in II, III, aVF
- No evidence for RVH
- RBBB:
- Late V1 = upward; Late V6 = downward
- RsR pattern in RIGHT precordial leads
- Wide broad S wave in I, V5 & V6
6. Evaluate for evidence of ischemia/infarction:
- ST Elevation/Depression
- TWI
- Change from baseline
- Dominant R wave V1/V2 suggestive posterior MI (IMI)
7. Evaluate For Vascular Distribution:
Territory ECG Distribution Vessel
Anterior wall of LV V1-V4 LAD
Inferior wall of LV II, III, aVF RCA/PD, LCx
Lateral wall of LV I, aVL, V5, V6 LCx
Posterior wall of LV V1-V2 (large R) RCA/PD, LCx
Septal Wall V1, V2 LAD/septal
40
8. Diagnosis of MI in presence of bundle branch block:
- No change in criteria in setting of RBBB, RBBB+LAFB and RBBB+LPFB
- In LBBB, normally ST-T wave changes are in opposite direction of QRS
Diagnosis of acute MI in LBBB:
>1mm concordant (in same direction of QRS) ST elevation
>1mm ST depression in V1-3
>5mm discordant (opposite direction of QRS) ST elevation
Q waves in I, avL, V5-6 suggestive of Q wave infarction
9. Differential of EKG Findings:
- Q Waves:
- LBBB
- WPW
- HOCM
- COPD
- Pulmonary Embolism
- ST Elevation:
- Hypothermia (Osborne/J waves)
- Acute Pericarditis (diffuse without anatomical distribution)
- Myocardial Infarction (follows anatomical distribution)
- LV aneurysm (persistent STE)
- Early repolarization
- Coronary vasospasm
- QT Prolongation:
- Congenital Long QT
- Electrolytes (hypocalcemia, hypomagnesemia, hypokalemia)
- Antipsychotics (e.g. haldol)
- Antiarrhythmics
- Bradycardia
CHF
3. History: orthopnea, PND, dyspnea on exertion, ask about functional status, weight gain,
decreased exercise tolerance, medication/diet non-compliance
4. Etiology: CAD, HTN, idiopathic dilated cardiomyopathy, valvular heart disease, substances
(Alcohol, cocaine, meth), infiltrative (amyloid, sarcoid), hemachromatosis, HIV, viral
5. Acute exacerbation: consider new ischemic event, arrhythmia, infection, hyperthyroidism,
PE, medication and dietary non-compliance, recent increase in beta-blocker dose, renal
failure, HTN
- evaluate severity of exacerbation: warm/dry -> warm/wet -> cold/wet -> cold/dry
6. Physical Exam:
- signs of left-sided failure: rales, tachypnea, S3
- signs of right-sided failure: elevated JVP, ascites, peripheral edema, enlarged/tender liver
- signs of low cardiac output: cachexia, muscle loss, cool extremities, tachycardia
5. Relevant Labs:
- chem7, CBC, TSH, LFTs, BNP, lipid panel, cardiac enzymes
- BNP: secreted by ventricles as a response to ventricular volume expansion/pressure
overload
- Lower than expected BNP: obese, acute MR/MS, flash pulmonary edema
- Higher than expected BNP: AKI, acute MI, lung dz with RHF, acute PE, cirrhosis,age
- BNP levels <100 pg/ml essentially excludes heart failure as a cause of a patients symptoms
or physical exam (sens: 90%, spec: 73%, NPP: 90%)
- not influenced by mild renal insufficiency, but may be increased in dialysis patients
41
6. Treatment:
- Aspirin, ACE-I, BB: shown to decrease mortality. If EF<40%, use carvedilol. Use BB when
pts are clinically stable, not necessarily during an exacerbation
- digoxin: symptomatic improvement in pts with systolic failure, decreases morbidity
- spironolactone: benefits pts NYHA Class III-IV (EF<40%), ischemic etiology
- Diuresis: often starts with Lasix. Lasix has a threshold before it works, so if a pt is not
responding to his diuretics, the dose will need to be increased. 2:1 PO to IV conversion.
Variable bioavaility.
- Bumex: 40x more potent than lasix with higher bioavailability (90%)
- may need to add Metolazone (30 min before giving lasix) as a primer to make diuresis more
effective
- if not responding to metolazone and lasix, may use Diurel (IV) instead
- In everyone, monitor strict I/Os and daily weights. Always have I/Os for rounds. Place pts on
2g Na diet, 2L fluid restriction, while diuresing check lytes BID (K, Mg).
- If bicarb is increasing rapidly, may consider acetazolamide
7. B-type Natriuretic Peptide (BNP):
- A hormone that is synthesized de novo by cardiac ventricles in response to increased wall
stress and/or ischemia. (Note: there are other natriuretic peptides A-type and C-type but are
not as specific and are currently not being used for diagnostic purposes)
- released primarily by LV during CHF exacerbation to relieve ventricular wall stress and thus
improve cardiac function ("wet BNP")
Pro-BNP
BNP = C-terminal proBNP NT-proBNP = N-terminal proBNP
Potent Neurohormone Biologically Inactive peptide
Renally cleared (partially) Not renally cleared
Short half-life (21 min) Short half-life
Physiologic Actions of BNP:
- Hemodynamic: - Renal:
- Vasodilation (venous > arterial) - Diuresis
- Neurohumoral: - Natriueresis
- Aldosterone - Cardiac:
- Norepinephrine - Fibrosis
- Endothelin - Remodeling
When to order a BNP level:
Dyspnea - (Physical exam and BNP)
BNP < 100 BNP 100 - 400 BNP > 400
CHF very
unlikely (2%)
Known LV Dysfunction, Cor Pulmonale or
Acute PE
CHF very
likely (95%)
Yes No
CHF exacerbation (25%) CHF exacerbation
(75%)
Dry BNP Wet BNP
BNP level once euvolemia is reached Anything significant over the dry BNP
Falls slowly with treatment Falls rapidly with treatment
May be 20-1000 pg/ml depending on
severity of Heart Failure
Can be 2 4 fold the dry BNP thus
worsening severity of HF to class III-IV
Correlates with NYHA functional class
42
- Heart failure patients
whose BNP level fails
to fall over their hospital
stay have a higher
incidence of
rehospitalization or
cardiac death.
-The closer the HF
patient gets to his/her
euvolemic weight, dry
BNP level, the less
likely to be
rehospitalized early.
CARDIAC BIOMARKERS
Troponin
- serial testing at presentation, then 6-12hr after onset symptoms. Detectable 4-6hr after myocardial
injury
- peaks at 24hr, may remain elevated 7-10 days after myocardial infarction
- false positive renal failure, demand ischemia
CKMB
- useful for evaluating re-infarction, post CABG and PCI.
- >3x UNL post-PCI suggests reinfarction, >5x UNL post-CABG suggests reinfarction
TACHYCARDIA AND TACHYARRHYTHMIAS
Initial Assessment: is pt symptomatic or hypotensive (SBP<80)?
- If so, place pt in Trendelenburg. Give fluid bolus, begin ACLS if necessary.
- place pt on cardiac monitor, get a 12 lead EKG/rhythm strip
Narrow Complex Tachycardia
Regular (QRS<120msec)
1. Sinus Tachycardia:
- could be caused by hypoxia, hypovolemia, fever, myocardial dysfunction, PE, pain, anxiety,
EtOH withdrawal, hyperthyroid
- usually has upright P waves in II, III, aVF followed by a QRS
- treat underlying cause
2. AVNRT: AV nodal re-entrant tachycardia, usually rate 150-250
- inverted P (or pseudo S) in II, III, aVF. P waves may be buried in QRS
- if pt unstable, synchronized DC cardioversion
- if no history of CVA/TIA/carotid bruit, try carotid massage or valsalva
- if no history of asthma, can try adenosine to break it
- put pt on rhythm strip, have atropine ready in case pt has a long pause
- Adenosine 6mg IVP, then 12mg IVP, then repeat 12mg IVP
- if doesnt break with adenosine, can try Metop 5mg IV q5min x3 (pushed over 2 minutes,
then flush) or Dilt 15mg IV q5min
3. Atrial Tachycardia:
- P waves before each QRS, usually with a P wave different from pts baseline P (atrial
activation does not start at sinus node)
- treat with BB, CCB, or Class III antiarrhythmic (amiodarone, sotalol)
4. Atrial Flutter (with regular block):
- flutter waves in II, III, aVF. Ventricular rate is some division of 300 (150, 100, 75).
- can push adenosine 6/12/12 to reveal flutter waves
- If stable, treat with BB, CCB, or digoxin
- If unstable, synchronized DC cardioversion
- may need anticoagulation
43
Irregular (QRS<120msec)
1. Atrial Fibrillation (see management below):
- caused by hypertension, pulmonary disease, idiopathic, ischemia, valvular heart disease,
thyrotoxicosis, ethanol, sepsis
- NO p waves on EKG
- If stable, treat with Metoprolol 5mg IV q5min x3 (if no COPD or low EF) or Diltiazem 10-15mg
IV q5min x3 (or dilt gtt)
- If unstable, synchronized DC cardioversion or amiodarone 150mg IV over 10 min followed by
1mg/min x6h, then 0.5mg/min x18h
2. Frequent PACs
3. Multifocal atrial tachycardia: caused by multiple ectopic pacemakers
- EKG will have 3 distinct P wave morphologies with 3 different PR intervals
- check magnesium, potassium
Wide Complex Tachycardia
Regular (QRS>120msec)
1. Ventricular Tachycardia (monomorphic):
- assume VT in all pts with heart disease
- ACLS protocol synchronized shock for perfusing VT and defibrillation for VF
- check electrolytes
2. SVT with Aberrancy:
- diagnosis of exclusion
- check if pt has history of BBB, although sometimes this is not brought out until pt is
tachycardic
- treat as SVT
- check lytes
- in any case, call your resident STAT
Irregular (QRS>120msec)
1. VTach (polymorphic): DEFIBRILLATE
2. Torsades de pointes (causes):
- Meds: antiarrhythmic drugs that prolong QT, TCAs, phenothiazines, H1 blockers
(astemizole, terfinadine), pentamidine, erythromycin, antifungal agents
- Electrolyte abnormalities: hypokalemia, hypomagnesemia
- Subarachnoid hemorrhage
- Congenital long QT syndrome
- EKG: twisting QRS complexes, prior EKGs may have long QT (QTc>450)
- Treatment: defibrillate, Mg Sulfate 2mg IV over 1 min
3. Atrial fibrillation with Aberrancy or Accessory pathway:
- diagnosis of exclusion. Treat as VT until proven otherwise
4. Frequent PVCs:
- Frequently due to electrolyte abnormalities, hypoxemia, acidosis.
- Check 12-lead EKG, CMs, Mag, chem 10 (replace Potassium & Magnesium), CBC, digoxin
level (if taking), consider ABG to r/o acidosis.
ATRIAL FIBRILLATION
Atrial Fibrillation w/ Rapid Ventricular Response (RVR)
1. OBTAIN VITALS and assess patient.
- Even if patient appears stable, patient cannot be left w/ HR>100 as rhythm can degenerate
and vitals can deteriorate. Also, sustained tachycardia is a significant stress and if underlying
CAD, pt can develop demand ischemia.
2. UNSTABLE Patient:
- DC CARDIOVERSION notify resident and prepare for it stat (follow ACLS).
3. Stable Patient:
- EKG to see if truly Afib/flutter vs. SVT +/- abberency
- Chem 10, Mag to ensure no electrolyte abnormalities.
4. Medical Interventions:
- Diltiazem: Push q15 min x 3(can start at 5mg and increase to 0.35 mg/kg in increments of
5mg) until rate control. Will have some effect on BP. Can give 1-2amps of Ca Gluconate prior
to dilt and with help negate effect on pressures. Once rate controlled give as gtt (5-15 mg/h)
IV for up to 24 h. Often considered first-line in patients without a h/o CHF; Contraindicated in
severe CHF, sick sinus syndrome, second- or third-degree AV block.
44
- Metoprolol: Push 5-15 mg IV over 5-15 min (5-mg increments). Preferred in patients with
CAD, thyrotoxicosis and increased sympathetic tone. Contraindicated in decompensated
CHF, bradycardia, asthma, cardiogenic shock, and AV conduction abnormalities.
- Digoxin: Loading regimen: 500mcg iv, f/b 250mcg 6h later, and another 250mcg 6h later
(total 1gram); decrease doses by half in CKD. Ensure K is replaced as hypokalemia limits
effect. Obtain level 6h after last loading dose. Effect often not seen for several hours, but
useful especially in low output heart failure. Contraindicated in wet beriberi, idiopathic
hypertrophic subaortic stenosis, constrictive pericarditis, carotid sinus syndrome.
- Other Agents (these are typically only used on the Cards service with fellow/attending
input): see below
ANTIARRHYTHMIC THERAPY
Antiarrhythmic Dosage Indications
Adenosine
6 mg IVP 12 mg IVP if needed
12 mg IVP if needed
1. Stable narrow-complex AV nodal or sinus nodal re-entry
tachycardia
2. Unstable re-entry SVT prior to cardioversion
3. Undefined stable narrow-complex SVT
Amiodarone
150 mg IVP over 10 min 1 mg/min
x 6 hr 0.5 mg/min x 18 hr
1. Narrow-complex re-entry tachycardia
2. Stable VT, polymorphic VT, wide-complex tachycardia
3. Rate control of due to accessory pathway
Calcium-channel blockers
1. Stable narrow-complex re-entry tachycardia
2. Stable narrow-complex automaticity tachycardia
(junctional, ectopic, multifocal)
3. Rate control of atrial fibrillation or atrial flutter (do not
give if WPW present)
Verapamil
1. 2.5-5 mg IV over 2 min 5-
10 mg Q15-30 min (20 mg total)
2. 5 mg IV bolus Q15 min (30
mg total)
Diltiazem 0.25 mg/kg IV 0.35 mg/kg IV
|-blockers
1. Narrow-complex re-entry tachycardia
2. Narrow-complex automaticity tachycardia (junctional,
ectopic, multifocal)
3. Rate control of atrial fibrillation or atrial flutter
(ventricular function preserved)
Atenolol
5 mg IV over 5 min can give
another dose 10 min later
Metoprolol 5 mg slow IVP Q5 min (15 mg total)
Propranolol
0.1 mg/kg slow IVP divided into 3
equal doses at 2-3 min intervals (no
faster than 1 mg/min)
Esmolol
500 mcg/kg IVP 50 mcg/kg/min x 4
min (200 mcg/kg total) if needed,
500 mcg/kg IV 100 mcg/kg/min IV
(max 300 mcg/kg/min)
Sotalol 1-1.5 mg/kg IVP 10 mg/min IV
1. Rhythm control of atrial fibrillation or atrial flutter with
WPW
2. Monomorphic VT
Others
1. Cardioversion of atrial fibrillation or atrial flutter with or
without WPW (<48 hr)
2. Control rate in atrial fibrillation or atrial flutter
Ibutilide
1 mg IV (0.01 mg/kg if < 60 kg) over
10 can repeat 10 min later if
needed
Lidocaine
0.5-0.75 mg/kg IVP (up to 1-1.5
mg/kg) 0.5-0.75 mg/kg IVP Q5-10
min (max 3 mg/kg) 30-50
mcg/kg/min IV
1. Stable monomorphic VT with preserved ventricular
function
2. Polymorphic VT with normal baseline QT interval
Magnesium
1-2 g IV over 5-60 min (stable
slower, unstable faster)
Torsades de pointes
Procainamide
20 mg/min IV until suppression of
arrhythmia, hypotension, or QRS
prolongation by 50% (max 17 mg/kg)
1-4 mg/min IV maintenance
1. Stable monomorphic VT with preserved ventricular
function
2. Rate control of atrial fibrillation or atrial flutter
3. Rhythm control of atrial fibrillation or atrial flutter with
WPW
4. Narrow-complex AV re-entry tachycardia
Reference: American Heart Association. 2005 American Heart Association Guidelines for Cardiopulmonary Resuscitation and Emergency
Cardiovascular Care Part 7.3: Management of Symptomatic Bradycardia and Tachycardia. Circulation 2005; 112(24 Suppl): IV-67-IV-77.
HYPERTENSION
1. Evaluation:
- Assess initially if Hypertensive Emergency (end organ damage) or Hypertensive Urgency.
Both with BP>180/110.
- Important to double check validity of reading, ask for recheck and you should CHECK
MANUALLY at bedside BP in both arms. Check CUFF SIZE.
- PE: Fundoscopic exam (papilledema), neuro exam, lungs for edema, BP in both arms for
possible aortic dissection.
2. Differential:
- OD (cocaine, meth)
- Withdrawal (EtOH, cocaine, meth, narcs, anti-HTN)
- Pain or anxiety
- Increased intracranial pressure (Cushings reflex associated w/ bradycardia)
- Renal Artery Stenosis
- Coarctation of the aorta
- Endocrine (thyrotoxicosis, pheo, Cushings syndrome)
45
** Do not lower BP too abruptly. Goal is 25% reduction in MAP or to reduce DBP to no lower than
100-110mmHg in first 12-24 hrs.
3. Hypertensive URGENCY: HTN without evidence of end-organ damage
- Labetalol: 10mg ivp, followed by 200-400mg po BID
- Clonidine: 0.2mg orally, re-check BP in 30-60 minutes. May follow with 0.1mg q hour to total
of 0.8mg. Watch for sedation, bradycardia with AV nodal blockers. Rebound hypertension if
abruptly stopped.
- Captopril 12.5-25mg orally works in 15-30 minutes.
- Metoprolol 12.5-100mg PO BID (ok to start titrating 25 mg po q6hrs x 48 hrs -> then convert
to BID). Try to avoid IV MTP as it lasts a short time. (2.5 mg PO MTP = 1 mg IV MTP)
4. Hypertensive EMERGENCY: HTN associated w/ end-organ damage
- End-Organ Damage:
- CNS: headache, mental status changes, seizure, stroke
- Eyes: blurred vision, papilledema, exudates, flame hemorrhages
- Cardiac: chest pain, EKG strain or ischemic changes, pulmonary edema
- Vascular: aortic dissection
- Renal: low urine output, edema, hematuria, azotemia
3. Management: admission to ICU
- IV Nipride 0.2-5 mcg/kg/min: 1st line in most patients. Arteriolar and venous dilation.
Toxicity with thiocyanate and cyanide metabolite must be considered with several days of
therapy and with renal and hepatic insufficiency
- IV Nitroglycerin 10-400mcg/min: First choice agent for patients with myocardial ischemia.
Venous dilation
- IV Labetalol (1, 1, 2 antagonist) 20mg IV bolus, then 2mg/min: First choice agent for
patients with myocardial ischemia or acute aortic dissection
4. Special cases:
- Cocaine or Pheochromocytoma, make sure you have alpha blockade (phenoxybenzamine
10mg BID) PRIOR to beta blockade or will have unopposed alpha tone.
- Pulmonary edema/MI, nitroprusside is good choice, but if using for Aortic Dissection,
make sure you have -blocker on board first.
HYPOTENSION
1. Initial Assessment:
- Make sure reading is accurate, repeat manually in both arms. Check CUFF SIZE, may need
arterial line. Assess if patient is mentating well. Remember to treat the patient, not the
numbers. BP 90/50 may be normal, ex: CHF or end stage liver disease.
- Palpate Pulse: Radial ~SBP>80. Carotid/fem ~SBP>60, No pulse - start Chest
Compressions, then call CODE BLUE
- Check for signs if hypoperfusion defects:
- Brain: AMS
- CV: MI, ischemia, arrhythmia
- Renal: decreased uop
- GI: LFT/lipase leak, can check lactate for ischemia
- Check baseline BPs (low in ESLD, ESRD, cardiomyopathy), also check med list.
- Workup to order once patient is stable: CBC with diff, Chem 10, EKG (rule out Afib, SVT
Vtach), cardiac markers, consider lactate, ABG, blood cultures.
2. Differential:
- Hypovolemia: hemorrhage, dissection, GI loss, dieresis, third spacing
- Cardiogenic: MI, arrhythmia, valvular dysfunction
- Obstructive: PE, tamponade, tension pneumo
- Sepsis
- Anaphylaxis
- Medications (BP meds)
- Adrenal Insufficiency (primary or relative)
3. Management: will depend on the etiology, but some general considerations:
- place pt in Trendelenburg position to increase perfusion to the brain
- check for anti-hypertensives like nitropaste, clonidine patch
- if no evidence for CHF or renal failure, bolus with 1L NS and re-evaluate
- in CHF/Volume Overload diuresis often will increase pressure
- in Afib with RVR, rate control will often improve blood pressure
46
PACEMAKERS
1. Nomenclature:
- Position I chamber paced (A atrium, V ventricle)
- Position II chamber sensed
- Position III response to event sensed (I inhibitory, D dual, T triggered, O none)
- Position IV rate modulation (R rate response to activity, O none)
- Position V multisite pacing (usually omitted)
2. Common Pacing Modes:
- AAI/AAIR for sinus node dysfunction. Need an intact AV node
- VVI/VVIR for bradycardias, afib w/ slow ventricular response
- DDD/DDDR for bradycardias, AV node dysfunction, physiology pacing
3. Complications after placement:
- Pneumothorax (check CXR)
- Hematoma formation (especially if patient on anticoagulation)
- Lead perforation of RA or RV (suspect if hemodynamically unstable)
- Lead dislodgment or pacemaker failure (suspect if failure to sense or capture)
STRESS TESTS
1. Exercise treadmill test:
- Excellent if pt has good exercise capacity; 85% sensitive.
- Do not use if pt unable to walk, is from jail (may not give maximal effort), has LBBB, WPW,
paced rhythm, or other EKG abnormalities that would make it uninterpretable.
- Before ETT, HOLD beta blockers and nodal CCB (verapamil, diltiazem).
2. Adenosine sestamibi:
- Causes direct coronary artery vasodilation, which is attenuated in diseased coronary
arteriesthey have reduced reserve and cant dilate in response to adenosine.
- Can cause bronchospasm. Use if BBB.
- Hold caffeine, which is a competitive inhibitor of adenosine.
3. Dobutamine stress echo:
- stimulates beta-1 and beta-2.
- Use in pts with RAD/asthma/COPD.
- Before dobutamine echo, HOLD BB, dilt/verapamil. Nifedipine/amlodipine OK
*Many attending prefer patients to be NPO at midnight before stress test.
THE POST-CATH CHECK
- Pt should be on bedrest with leg straight 6 hours afterwards
- At night, check the groin. Make sure there is no hematoma and distal pulse is palpable.
Listen to ensure no bruit.
- CBC and cardiac markers q6h (follow CKMB for pericath MI, significant if >3x upper limit of
nl)
- If there is an acute bleed, hold pressure ABOVE the puncture site, since the opening from
the femoral artery will be higher than the external puncture site
- Summarize cath report results on S/O sheet
- Post cath complications to watch for: MI >3x ULN CKMB, Renal failure, Stent thrombosis,
47
HEMODYNAMICS AND SWAN-GANZ CATHETERS
Site Normal Pressure (mmHg)
Right Atrium
Right Ventricle
Pulmonary Artery
Pulmonary Capillary Wedge Pressure
Left Atrium
Left Ventricle
Aorta
Peripheral (measured blood pressure)
< 5 (3-5)
< 25/5
< 25/10
< 12 (6-12) ~ PA diastolic pressure
< 12 (~ PCWP)
< 150/10
< 150/90
< 120/80
- Cardiac output (CO) = (stroke volume) x (heart rate) normal 3-7 L/min
- Cardiac index (CI) normal 2.4-4 L/min/m2
- Systemic vascular resistance (SVR) normal 800-1200 dynes-s/cm5
- Mixed venous O2 saturation (SvO2) normal 70-75%
Condition
Right Atrial
Pressure
PCWP SVR CO
Septic shock + + +
Hypovolemic shock + + +
Cardiogenic Shock +
Neurogenic Shock + + + Normal to +
Pulmonary Embolus Normal to Normal +
Tamponade Normal to + +
RV Infarct + +
48
HYPERLIPIDEMIA MANAGEMENT
1. Lipid Lowering Medications:
Drug class* Serum LDL Serum HDL Serum TGs
Bile acid sequestrants 15 to 30 percent 0 to slight increase No change*
Nicotinic acid 10 to 25 percent 15 to 35 percent 25 to 30 percent
HMG CoA reductase
inhibitors (statins)
20 to 60 percent 5 to 10 percent 10 to 33 percent
Gemfibrozil 10 to 15 percent 15 to 25 percent 35 to 50 percent
Fenofibrate
(micronized form)
6 to 20 percent 18 to 33 percent 41 to 53 percent
Cholesterol absorption
inhibitors
17 percent No change No change
Neomycin 20 to 25 percent No change No change
2. Lipid Target Levels:
Risk category LDL goal
LDL level at which to
initiate lifestyle
changes
LDL level at which to consider
drug therapy*
High risk:
Coronary heart
disease (CHD) or
CHD risk
equivalent (10-
year risk >20
percent)*
<100
mg/dL;
optional
goal <70
mg/dL in
very high
risk
100 mg/dL
100 mg/dL; <100 mg/dL
consider drug options
Moderately high
risk: 2 or more risk
factors (10-year
risk 10 to 20
percent)
<130
mg/dL
130 mg/dL
130 mg/dL; 100 to 129
mg/dL consider drug options
Moderate risk: 2 or
more risk factors
(10-year risk <10
percent)
<130
mg/dL
130 mg/dL 160 mg/dL
Lower risk: 0 to 1
risk factor
<160
mg/dL
160 mg/dL
190 mg/dL; 160 to 189
mg/dL consider drug options
Risk category LDL goal
LDL level at which to
initiate lifestyle
changes
LDL level at which to consider
drug therapy*
High risk:
Coronary heart
disease (CHD) or
CHD risk
equivalent (10-
year risk >20
percent)*
<100
mg/dL;
optional
goal <70
mg/dL in
very high
risk
100 mg/dL 100 mg/dL; <100 mg/dL
consider drug options
Moderately high
risk: 2 or more risk
factors (10-year
risk 10 to 20
percent)
<130
mg/dL
130 mg/dL 130 mg/dL; 100 to 129 mg/dL
consider drug options
Moderate risk: 2 or
more risk factors
(10-year risk <10
percent)
<130
mg/dL
130 mg/dL 160 mg/dL
Lower risk: 0 to 1
risk factor
<160
mg/dL
160 mg/dL 190 mg/dL; 160 to 189 mg/dL
consider drug options
* CHD equivalent: AAA, symptomatic carotid artery disease, DM, peripheral arterial disease
*Adapted from Third report of the National Cholesterol Education Program (NCEP) Expert Panel on detection, evaluation,
and treatment of high blood cholesterol in adults (Adult Treatment Panel III). Circulation 2002; 106:3143; with
modifications from Grundy, SM, Cleeman, JI, Merz, CN, et al, Circulation 2004; 110:227.
*Statins use should be maximized if possible before addition of other lipid lowering agents.
49
PERIOPERATIVE CARDIAC RISK ASSESSMENT
1. General concepts:
- The perioperative period can put patients with vulnerable coronary artery lesions at risk for MI
or death due to stresses of surgery, anesthesia, and a prothrombotic postoperative state.
- The ACC/AHA guidelines have been recently updated based on new evidence, which is
summarized below
2. Who is at risk: Need to assess 3 components: functional status, surgical risk, and clinical
risk
- Functional status
- Surgical stress = approximately 4 METS on a treadmill test
- 4 METS = climbing 2 flights of stairs, brisk walking on level ground for 4 blocks
- If a patient can achieve 4 METS without angina equivalent, he likely can withstand the
surgical stress
- Surgical risk
- Certain surgeries are inherently high risk (>5% risk of cardiac complications)
- Aortic or other major vascular surgery
- Peripheral vascular surgery (not including CEA)
- Anticipated major blood loss or prolonged surgery
- Clinical predictors of risk (use Revised Cardiac Risk Index [RCRI] Lee TH et al.
Circulation. 1999; 100: 1043-9 ).
- Assign 1 point for each:
- High risk surgery
- CAD
- CHF
- Cerebrovascular disease
- IDDM
- Creatinine > 2.0
- For < 3 points, risk of cardiac event < 1%
- For 3 or more points, risk > 6%
50
3. How to risk stratify:
4. Estimated Energy Requirement:
1 MET Self care
Eating, dressing, or using the toilet
Walking indoors and around the house
Walking one to two blocks on level ground 2-3 mph
4 METs Light housework (e.g., dusting, washing dishes)
Climbing a flight of stairs or walking up a hill
Walking on level ground at 4 mph
Running a short distance
Heavy housework (scrubbing floor, moving furniture)
Moderate recreational activities (e.g., golf, dancing, doubles tennis,
throwing a baseball or football)
>10 METs Strenuous sports (e.g., swimming, singles tennis, football, basketball,
skiing)
5. Four Active Cardiac Conditions: For Which the Patient Should Undergo Evaluation and
Treatment Before Noncardiac Surgery (Class I, Level of Evidence: B)
- Unstable Coronary Syndromes Unstable or Severe Angina,
Recent MI
- Decompensated HF NYHA Class IV,
Worsening or New-onset HF
- Significant Arrhythmias High-grade AV block,
Mobitz II AV block
Third-degree AV heart block
Ventricular arrhythmias*
Supraventricular Arrhythmias*,
Symptomatic bradycardia,
Ventricular tachycardia
- Severe Valvular Disease Severe Aortic Stenosis**,
Symptomatic Mitral Stenosis
*HR greater than 100 or symptomatic, **gradient >40 mmHg, AoV <1.0 cm2
51
6. Who needs preoperative testing:
- Short answer: almost NO ONE who is an inpatient awaiting urgent surgery
- Short answer #2: ask yourself If I were seeing this patient for an office visit instead of for a
pre-op consult, would I recommend testing? If the answer is no, you should probably not
recommend a test
- Neither stress testing nor revascularization has been shown to improve outcomes
postoperatively, and several high quality trials have been done. (McFalls, et al. NEJM 2004
v 351, Poldermans D, et al. JACC 2006; 48: 964-9.)
- However, in cases for which patients with significant ischemia on testing could reasonably
defer or delay the surgery, it is reasonable to test
- Remember, patients with stents need antiplatelet agents continued for 6-12 months
minimum, so perioperative PCI will delay surgery unless it can safely be performed on
antiplatelet agents
7. Who needs preoperative beta-blockers
- Short answer: DO NOT STOP beta-blockers preoperatively in patients who are taking them
already, DO NOT START perioperative beta-blockers in naive patients.
- Data from the POISE trial (Lancet 2008; 371: 1839-1847) suggest that starting beta-blockers
in nave patients reduces cardiac events at a cost of increased mortality; there remains
controversy about the design but we await results of DECREASE V to shed more light on the
issue.
- IF you are using perioperative beta-blockers, adjust them regularly to a target HR of <75
8. Final thoughts
- Avoid writing cleared for surgery
- Better: patient is at low risk for cardiac events, no further pre-op testing recommended, or
the patients cardiac risk factors are optimized
- Address other issues (e.g. management of diabetes, COPD, adrenal insufficiency, VTE
prophylaxis)
From ACC/AHA guidelines: Fleisher, L. A. et al. Circulation 2007;116:e418-e499
CARDIOLOGY LITERATURE
CHF Trials:
US Carvedilol Heart Failure Study Group
NEJM 1996; 334 (21): 1349-55
Design: Double-blind, placebo-controlled; 1094 pts with chronic HF randomized to placebo or
stratified to carvedilol with initial dosing of 6.25mg, 12.5mg, or 25mg BID based on exercise capacity.
Pts assigned to mild-mod HF based on 6-min walk test; mean follow-up 6.5mo.
Endpoint: Death or hospitalization for CV events.
Results: Early termination; 65% RRR overall mortality (p <0.001); 27% reduction in hospitalization
from CV causes (p = 0.036); 38% reduction in combined risk of hospitalization
or death (p <0.001).
Conclusion: Carvedilol reduces the risk of death and hospitalization from CV causes in pts w/ chronic
HF (mostly pts in class II/III) with concurrent therapy of ACE-I, diuretics, and digoxin.
Carvedilol Prospective Randomized Cumulative Survival Trial (COPERNICUS)
NEJM 2001; 344: 1651-8
Design: Double blind RCT 2289 pts with NYHA III and IV pts w/ LV EF < 25%, but clinically stable (i.e.
euvolemic and not on pressors) with chronic HF from ischemic or non-ischemic CM, but not from
valvular disease. Pts on treatment group received carvedilol 3.125mg bid to target dose 25 mg BID
for 11 months vs. placebo.
Endpoint: Death from HF, cardiac causes and/or re-hospitalization.
Conclusion: The Carvedilol group had a 35% RRR in death. A combined end point of death &
hospitalization was signif for a 24% RRR compared to placebo. Prior studies show that not all b-
blockers have this favorable mortality effect, bucindolol was tested and showed no effect.
Metoprolol CR/XL Randomized Intervention Trial in Congestive Heart Failure (MERIT-HF)
Lancet 1999; 353: 2001-7
Design: Double-blind, placebo-controlled; 3994 pts w/ chronic HF in NYHA II-IV randomized to
Metoprolol XL 12.5mg III-IV) or 25mg (II) daily vs placebo. Target dose 200mg once daily.
Results: Early termination; lower all-cause mortality w/ metoprolol XL (7.2% pt-yr of f/u vs 11%, RR
0.66); fewer sudden deaths (79 vs 132, RR 0.59); deaths worsening HF (RR .51).
Conclusion: Metoprolol CR/XL daily, w/standard HF therapy, improves survival in pts w/ mild-severe
chronic ischemic or non-ischemic heart failure (>95 % of pts in class II-III).
52
Randomized Aldactone Evaluation Study Investigators (RALES)
NEJM 1999; 341: 709-17
Design: Double-blind, placebo-controlled; 1663 pts with severe chronic HF (NYHA III-VI) from
ischemic/non-ischemic causes randomized to 25mg spironolactone qd vs placebo.
Results: Early termination; 30% RRR in mortality in the spironolactone group- attributed to lower risks
of sudden cardiac death and death from progressive HF; 35% RRR in hospitalization for worsening
HF; improvement in NYHA class HF sxs; all with p <0.001.
*Beware of gynecomastia/breast pain and serious hyperkalemia (K >6)
Conclusion: Addition of spironolactone to standard therapy reduces morbidity & mortality in pts with
severe HF. Of note, only 10% pts on a beta-blocker @ baseline during the trial (COPERNICUS
published in 01).
Digitalis Investigation Group (DIG Trial)
NEJM 1997; 336: 525-33
Design: Double-blind, placebo-controlled; in the main trial, pts w/ EF <45% (70% due to ischemia;
>80% in NYHA Class II-III) randomized to digoxin (3397 pts) vs placebo (3403 pts) in
addition to diuretics and ACE-I; median dose digoxin 0.25mg daily. In the ancillary trial,
pts w/ EF >45%; 492 pts assigned to digoxin and 496 to placebo.
Conclusion: Digoxin reduced hospitalization rates both overall and for worsening heart failure;
however, digoxin did NOT reduce overall mortality.
NSTEMI Trials TACTICS-TIMI 18
NEJM. 2001;344:1879-87.
Design: Blinded, randomized controlled trial; 2220 pts w/ ECG e/o ischemia, elevated troponin or
documented CAD were randomized to cath < 48hrs after randomization vs medical tx and stress test.
All pts received ASA, heparin, and tirofiban.
Conclusion: Early invasive strategy was more effective with RRR of 30% at 30 days and 18% at 6
mos. 19% mortality in conservative group vs. 14% mortality in early invasive group. Further
subgroup analysis found that this benefit was only found in pts w/ a TIMI risk score >3, there was no
benefit in pts w/ a low TIMI risk score (0-2).
Clopidogrel in Unstable Angina to Prevent Recurrent Events (CURE)
NEJM August 16, 2001
Design: 12562 patients randomized within 24 hours after onset of non-ST elevation ACS to receive
either ASA (75mg to 320mg) alone or with clopidogrel (300mg LD, then 75 mg daily).
Results: Primary outcome occurred in 9.3% of the clopidogrel group & 11.4% of the ASA alone group
(RR 0.8, p<0.001). Refractory ischemia occurred 16.5% in the clopidogrel group and 18.8% in the
placebo group (RR 0.86, p<0.001). Significantly more bleeding in the clopidogrel group (3.7% vs
2.7%) but no difference in life threatening bleeding or hemorrhagic strokes.
Conclusion: Clopidogrel + ASA has beneficial effects in ACS vs ASA alone. Major
bleeding and possible delay of CABG are the main drawbacks of the study.
The Platelet Receptor Inhibition in Ischemic Syndrome Management in Patients Limited by
Unstable Signs and Symptoms (PRISM-PLUS)
NEJM May 21, 1998
Design: 1915 patients with unstable angina or NSTEMI randomized in a double-blind study
to receive tirofiban, heparin or both. Everyone received ASA.
Results: Study was stopped prematurely for the group receiving tirofiban alone, who had excess
mortality within the first 7 days. Primary endpoint was reached at 7 days for 12.9% in the group
receiving tirofiban + heparin, and 17.9% in the heparin alone group (p=0.004). There was no
significant difference in major bleeding between the two groups.
Conclusion: When administered with ASA and heparin, tirofiban decreased incidence of ischemic
events in pts with ACS compared to those receiving ASA and heparin alone.
Platelet Glycoprotein IIb/IIIa in Unstable Angina: Receptor Suppression Using Integrilin
Therapy (Pursuit)
NEJM August 13, 1998
Design: 10,948 patients with rest angina lasting >10 minutes in the previous 24 hours with ischemic
EKG changes or CKMB elevation were randomized to receive either eptifibatide plus standard
therapy, or placebo.
Results: Primary endpoint reached for 15.7% of placebo group and 14.2% of study group (p=0.04).
Bleeding was more common in the study group, but there was no significant increase in hemorrhagic
CVA.
Conclusion: Eptifibatide reduced death & non-fatal MI in pts with ACS without persistent STE
53
Electrophysiology Trials Madit II (Multicenter Automatic Defibrillator Implantation Trial II)
NEJM March 21, 2002
Design: 1232 pts with prior MI (@ least >30dy prior), EF<30% randomized to receive prophylactic
ICD or conventional medical therapy.
Results: Study terminated prematurely after f/u 20mo because ICD group had significantly reduced
all-cause mortality (14.2 vs 19.8%) all due to reduction of sudden death.
Conclusion: In pts with prior MI & LV dysfx (EF<30%) prophylactic implantation of a defibrillator
improves survival.
SCD-HEFT (Sudden Cardiac Death in Heart Failure Trial)
NEJM January 20, 2005
Design: 2521 pts w/ NYHA class II/III CHF (ischemic/non ischemic) and an EF < 35% were
randomized to conventional drug therapy (CDT) + placebo vs. CDT + amio vs. CDT + ICD.
Endpoint: Death from any cause.
Results: At 45 months there were 244 deaths, 29% in the placebo group, 28% in the amio group, and
22% in the ICD group. ICD had a 23% relative risk reduction (p<0.007) compared to CDT + placebo.
This benefit did not differ between non ischemic and ischemic CHF.
Conclusion: In pts w/ NYHA class II/III CHF and an EF < 35%, ICD therapy has improved survival
benefit compared to amiodarone/placebo.
COMPANION (Comparison of Medical Therapy, Pacing and Defibrillation in Heart Failure)
NEJM May 20, 2004
Design: 1520 pts w/ NYHA class III/IV CHF were randomized to medical therapy vs. medical therapy
+ cardiac resynchronization vs. medical therapy+ cardiac resynch + defibrillator.
Results: For all cause death/hospitalizations, compared to med tx alone, med tx + resynch or med tx
+esynch/defib had a 20% relative risk reduction. For CHF related death/hospitalization, compared to
med tx alone, med tx + resynch had RRR of 34% and med tx + resynch/defib had RRR of 40%. All p-
values <0.05.
Conclusion: Medical tx plus cardiac resynch/defib significantly decreases all cause
mortality/hospitalization. The use of resynch has been further validated by the CARE-HF trial (NEJM
March 7, 2005).
DEFINITE (Defibrillators in Nonischemic Cardiomyopathy Treatment Evaluation)
NEJM May 20, 2004
Design: 458 pts w/ nonischemic cardiomyopathy, NYHA I III, EF < 35% were randomized
to conventional drug therapy (CDT) vs. CDT + ICD
Endpoint: Death from any cause.
Results: ICDs demonstrated a statistically significant reduction of 80% in arrhythmic
mortality and a trend (P = 0.08) toward a reduction in all-cause mortality.
Conclusion: In pts w/ nonischemic CM, NYHA class I-III Heart Failure, and EF < 35%
ICDs reduce arrhythmic mortality and show a trend toward reducing all-cause mortality.
AFFIRM
NEJM 2002;347:1825-33
Design: Randomized, multicenter. rhythm vs. rate control in 4060 pts w/ A-fib & high
risk of stroke or death.
Rhythm-control: Sole or combination of: amiodarone, disopyramide, flecainide, moricizine,
procainamide, propafenone, quinidine and sotalol.
Rate-control: Target resting HR: 80 bpm or while walking: 100 bpm. Drugs: b-blockers, CCB, digoxin
and combination of these drugs. INR of 2 3 maintained.
Results: Trend towards increased mortality with the rhythm control strategy (p = 0.8). Torsades,
bradycardic arrest, and cardiac related hospitalizations occurred more frequent in rhythm control
group.
Conclusion: Rhythm controlled atrial fibrillation had more co-morbidities than rate control strategy.
Therapeutic anticoagulation should be maintained to avoid risk of CVA.
Lipid Trials: Heart Protection Study (HPS)
Lancet 2002 360:7
Design: Randomized placebo controlled, 20,536 pts to either simvastatin 40mg qd or placebo.
Endpoint: Mortality, fatal or non-fatal vascular events, average follow 5.5 yrs
Results: Statin group: 13% reduction all-cause mortality, 24% reduction major CV events, 25%
reduction In first stroke. 3982 diabetics without known CAD 28% reduction MI/Stroke. Reduction
seen even with LDL<100.
Conclusion: For a wide range of high risk populations, 40mg simvastatin reduces rates MI, stroke by
about one quarter irrespective of initial cholesterol concentrations.
54
Reversal of Atherosclerosis with Aggressive Lipid Lowering (REVERSAL)
JAMA March 3, 2004
Design: 654 patients randomized in a double-blind study to receive either 40mg of pravastatin or
80mg of atorvastatin for 18 months.
Results: Significantly lower progression rate in atorvastatin group (p=0.02), mean LDL of 150 reduced
to 110 in the pravastatin group and 79 in the atorvastatin group (p<0.001), and CRP decrease 5.2%
with pravastatin and 36.4% with atorvastatin.
Conclusion: For patients with CAD, intensive lipid-lowering treatment with atorvastatin reduced
progression of coronary atherosclerosis compared with pravastatin.
Pravastatin or Atorvastatin Evaluation and Infection Therapy-Thrombolysis in Myocardial
Infarction 22 Investigators (PROVE IT-TIMI 22)
NEJM April 8, 2004
Design: 4162 pts hospitalized for ACS within the previous 10 days were randomized in a double-blind
study to receive either 40mg of pravastatin or 80mg of atorvastatin.
Endpoint: Death from any cause, MI, unstable angina requiring hospitalization, revascularization
(performed at least 30 days after randomization), CVA.
Results: Median LDL achieved was 95 in pravastatin group and 62 in atorvastatin group (p<0.001).
Primary endpoint reached at 2 yrs in 26.3% of pravastatin group and 22.4% of atorvastatin group;
16% reduction in hazard ratio (p=0.005, 95% CI 5-26%).
Conclusions: Intensive lipid-lowering therapy (reduction of LDL to levels substantially below target
levels at time of study) for patients with recent ACS provides greater protection against death or
major CV events compared to a standard regimen. Intensive Lipid Lowering with Atorvastatin in
Patients with Stable Coronary Disease (TNT)
NEJM April 7, 2005
Design: 10,001 patients with CHD and LDL less than 130, randomly assigned, double-blind therapy,
either 10mg or 80mg of atorvastatin per day, average follow up of 5 years.
Results: Mean LDL=77 with 80 mg of atorvastatin , LDL=101 with 10 mg of atorvastatin. Primary
event in 8.7% of pts receiving 80 mg, as compared with 10.9 % pts receiving 10 mg. ARR of major
CV events of 2.2% and a 22% relative reduction in risk (P<0.001). No difference between the two
groups in mortality. Persistent elevations in liver aminotransferase levels was 0.2 % in the group
given 10 mg and 1.2% in the group given 80 mg (P<0.001)
Conclusion: Intensive lipid-lowering therapy with 80 mg of atorvastatin per day in patients
with stable CHD provides significant clinical benefit beyond that afforded by treatment with 10 mg of
atorvastatin per day. Greater incidence of elevated aminotransferase levels.
Other CV/HTN Studies: COURAGE
NEJM 2007 Apr 12;356(15):1503-16.
Design: 2287 pts with objective myocardial ischemia and stable CAD comparing PCI + optimal
medical therapy vs. optimal medical therapy.
Results: Primary endpoint of all cause mortality and nonfatal MI, median 4.6 years. PCI group 19% vs
medical therapy group 18.5% (NS). Composite secondary endpoint of death, MI, and stroke 20% vs
19.5 % (NS)..Conclusions: As an initial management strategy in patients with stable coronary artery
disease, PCI did not reduce the risk of death, myocardial infarction, or other major cardiovascular
events when added to optimal medical therapy.
ALLHAT
JAMA 2002;288:2981-97
Design: Double-blinded randomized controlled trial; patients with HTN and 1 other CAD
risk factor treated with CCB or ACE-I vs. diuretic (chlorthalidone).
Conclusion: There was no significant difference in primary outcomes in the two groups; all cause
mortality similar between the two groups. However, thiazide diuretics had significantly lower SBPs
after 5yrs. Thiazide diuretics were superior in preventing CAD & should be the FIRST LINE
medication used.
55
HOPE
NEJM. 2000;342(3):145-53
Design: Randomized controlled trial; 9297 high risk patients NOT known to have a low EF or CHF
were randomized to ramipril vs. placebo.
Conclusion: Ramipril reduced the rate of death, MI and stroke in high-risk patients. RR=0.74 for MI;
0.68 for stroke; 0.85 for revascularization procedures, 0.63 for cardiac arrest.
SAVE study showed captopril reduced all-cause mortality (RRR 17%) and CV morbidity when started
3-16 days after MI. RRR in cardiovascular death was 20%. (NEJM. 1992;327:669-77.)
SOLVD study showed enalapril given to pts w/ low EF was associated w/ improved survival and
slowed progression to decompensated heart failure. HOWEVER, the difference was NOT
significant. (NEJM. 1991;325:293-302.)
CONCENSUS II
NEJM. 1987;316:1429-35.
Trial showed that enalapril given within 1 day of MI was NOT associated with a significant survival
benefit.
56
Procedures
CENTRAL VENOUS CATHETERS: AN INTRODUCTION
1. Indications:
- continuous hemodynamic monitoring/transvenous pacing
- continuous infusions of medications that could induce phlebitis
- plasmapheresis/hemodialysis
- lack of peripheral IV access
2. General Rules:
- visualize anatomy use portable u/s if possible.
- be sterile gown, gloves, mask/shield, sterile drapes, chlorhexidine.
- position the patient be comfortable.
- have everything ready before hand (dont forget flushes).
- small amounts of sedation are OK, local anesthesia is essential
- get consent (unless emergent)
- if the patient is on the UCSD CCU SERVICE, YOU MUST HAVE FELLOW
- SUPERVISION, even if you are signed off.
3. Types of central lines:
- Triple Lumen Catheter (TLC) 3 lumens for multiple drugs.
- Cordis this is a large lumen through which swan-ganz catheters, transvenous pacers, triple
lumens and large volume fluid resuscitation can be passed.
- Vascath these are large bore and for dialysis, they require fellow supervision.
4. Where to go:
- Internal Jugular cleaner, easy to access, low risk of PTX, easy to compress carotid artery if
punctured, but uncomfortable for patient. Excellent for swans and pacers.
- Subclavian cleanest, easy to access, most comfortable for patient but higher risk of PTX
and no ability to compress bleeding. Excellent for swans and pacers. Contraindicated if
coagulopathic/thrombocytopenic.
- Femoral Vein easy, especially during CPR, but increased risk of infection, retroperitoneal
bleed and thrombus formation.
5. Seldinger Technique:
After identifying landmarks on a sterile field, the vein is cannulated with a needle and a guide
wire is inserted. NEVER let go of the guide wire. The tract is then dilated and a catheter
inserted over the guidewire. The wire is removed and the catheter stitched in place.
57
INTERNAL JUGULAR VENOUS CATHETER
The IJ vein is internal and anterior to the carotid artery (except in 5-10% of patients therefore, U/S
is a good idea).
1. Positioning: The IJ vein is found at the apex of the triangle formed by the sternal and
clavicular branches of the SCM with the clavicle. The carotid is medial to the vein. Tilt the
patient at 15-30% trendelenburg (less chance of air embolism) and turn the head 45 degrees
away from puncture site.
2. Preparation: Clean the site extensively with chlorhexedine. Open the sterile kit and fill the
wells with sterile saline for flushes. Gown up and organize your kit by arranging materials,
flushing lumens, drawing up lidocaine, etc. Drape your pt.
3. The needle:
- Find and maintain continuous palpation of the carotid artery. Liberally inject local anesthetic
at the apex of the triangle. ALWAYS aspirate before injecting to avoid injecting
anesthetic/air into the vessel.
- Using a finder needle, angle at 45-60 degrees, and aim toward ipsilateral nipple, advance
and aspirate until you find a flash w/ good flow. If it pulsates or is bright red, that is a bad
sign.
- Advance the introducer needle directly in front of the finder and aspirate. With good flow,
remove the syringe (finger over the needle to prevent air entering) and thread the wire. Dont
force it and NEVER let go. If the wire passes successfully, remove the needle.
- Enlarge the puncture site with the scalpel (blade away from the wire) and apply pressure with
sterile gauze to prevent brisk bleeding. Advance the dilator over the wire, then remove the
dilator.
- Thread the catheter over the wire dont let go. When in place, remove the wire, aspirate
blood from all ports and then flush with sterile saline and suture in place with a 2.0 silk
suture. Clean the site and cover with tegaderm.
4. Check a CXR: to confirm the catheter is in the SVC and there is no PTX. Dont forget to clean
up.
SUBCLAVIAN VENOUS CATHETER
The subclavian vein is below the medial third of the clavicle, coursing just below the bone.
1. Positioning: Tilt the patient at 15-30 degrees Trendelenburg, the vein is found at the junction
of the proximal third with distal two-thirds of the clavicle, aiming toward the sternum. Place a
rolled up towel between the scapula to lower the ipsilateral shoulder. Preparation: liberally
sterilize, local anesthetic aspirate before injecting. Prep the kit as above.
2. The needle:
- Place a finger on the sternal notch, and thumb on the clavicle at the junction between the
medial first third and the outer two-thirds.
- Insert the needle 2 cm lateral to the clavicular curve and keep it parallel to the patients body
with bevel up. Do NOT enter at a steep angle, keep it low to avoid PTX.
- Aspirate and advance by marching down the bone, aiming just above the sternal notch. If no
venous return, withdraw and re-direct.
- Use the Seldinger technique to advance the catheter in place.
3. Check the CXR:
4. NEVER attempt a line on the opposite side: without checking a CXR and ruling out a PTX
on the first side.
5. TIPS: Have an assistant pull down on the ipsilateral arm, this will assist in successfully finding
the vein. Once you have inserted the needle bevel up, rotate the needle 90 degrees caudally
to assist in correct advancement of the wire.
58
59
FEMORAL VENOUS CATHETER
The femoral vein is found medial to the artery. Some use the mnemonic NAVEL (from lateral to
medial Nerve, Artery, Vein, Empty Space, Lymphatics).This is opposite to the IJ vein which is lateral
to the artery.
1. Positioning: The leg should be slightly abducted and externally rotated.
2. Preparation: As above, sterilize well. This is a dirty site.
3. The needle:
- Palpate the artery and advance a finder needle (or an introducer) with the other hand at a 45
degree angle.
- Do not let go of the artery and continually aspirate as you advance. It can be very shallow
(i.e. 2 cm) in thin people.
- If no flash, withdraw the needle slowly, but continue to aspirate you may have compressed
the vein while advancing. *If you hit the artery, hold pressure for 15-20 minutes.
- With a good flash, use the Seldinger technique as described above. Never advance your
wire against resistance.
REMOVAL OF CENTRAL VENOUS CATHETERS
There is a small, but real risk of introducing an air embolus into the circulatory system any time a
central venous catheter is removed. This can be a fatal event, which is why removal of catheter Is
now also a supervised-to-signed-off procedure.
1. Gather supplies:The leg sterile scissors, gauze, sterile ointment (bacitraicin ointment packs
work well), tape, chlorhexadine swabs
2. Positioning: position patient in Trendelenberg position
3. Preparation: remove dressing and stitches and swab area with chlorhexadine swab. Squeeze
ointment onto the sterile gauze.
4. Instruct patient on Valsalva maneuver (hum or bear down)
5. While patient is humming, slowly remove catheter and immediately apply the ointment-laiden
gauze to the exit site and hold pressure
6. Dressing should be left in place for 24 hours.
THORACENTESIS
1. Indications: Effusion (unilateral or bilateral but disparate sizes), or febrile, or having
associated pleurisy. Need >1cm on lateral decubitus to be considered safe to tap. If small
effusion or loculated consider US guided tap. If presents with CHF, b/l effusions, afebrile and
without chest pain, may try diuretics first.
2. Contraindications: severe coagulopathy, mechanical ventilation (relative)
3. Positioning: Patient seated upright, bed raised for your comfort.
4. Preparation: Review the CXR, percuss the patient. Your site should be 1-2 interspaces
below where the dullness ends and about midway between the spine and the posterior axillary
line. Liberally clean the site and drape the patient. You need a 50ml syringe, 22 g needle 1.5
in long and lidocaine for local anesthesia.
5. The needle: After local lidocaine, advance your needle superiorly over the rib (to avoid
damage to the neurovascular bundle) and aspirate.
6. CXR is not required: (only 1 in 488 asymptomatic patients post-thoracentesis had PTX)
although usually obtained regardless of the data.
7. Measurements: (patient needs serum LDH and total protein at the same time)
- Appearance bloody effusions concerning for malignancy
- Cell count and differential - >10000 wbc usually empyema. >10% eosinophils, consider
trauma but also fungal/tb/parasite/malignancy
- Gram stain and culture can help with antibiotic choice
- LDH, protein, pH and glucose to determine exudates vs. transudates
- Cytology (need large volume)- at the very least 100 cc
8. Lights Criteria: (Exudates have one or more of the below (98% sensitive))
- fluid/serum protein ratio >0.5
- fluid/serum LDH ratio > 0.6
- OR fluid LDH >2/3 upper limit of normal
- pH usually <7.2 with infection, indicates need for chest tube
60
PARACENTESIS
1. Indications: new ascites, refractory ascites, rule-out SBP.
2. Positioning: Patient supine in bed, raised for your comfort.
3. Preparation: Abdominal wall is thinnest 2 fingerbreadths cephalad & 2 finger breadths medial
to the left anterior superior iliac spine, avoiding large umbilical veins and the large spleen.
Sterilize the LLQ with chlorprep or betadine and drape the patient. Prepare your kit.
4. The needle: After injecting a few ml of lidocaine (aspirate first), advance your needle through
the dermis using the z-technique. Lift the skin up to prevent your needle from entering the
peritoneum in a straight path more prone to leaks. Withdraw fluid for diagnosis with large
syringe. If doing a large volume tap use the paracentesis catheter in the kit.
- If large volume tap, give IV albumin ~8gm/L of fluid removed as may decrease risk of
hemodynamic compromise and acute renal failure.
5. Measurements:
- Cell count (purple top): an ANC of greater than 250 is diagnostic of infection.
- Chemistries (gold top): Measure albumin (for SAAG) and total protein. LDH, amylase,
and glucose can also be useful. The patient will also need a serum albumin and total
protein from that same day.
- Gram stain and Culture (red top): At UCSD, send in syringe. At the VAMC, inoculate blood
culture bottles at bedside (increase sensitivity from 50%80%).
- Cytology: This requires large volumes (dont forget labels)
LUMBAR PUNCTURE
1. Indications: Two out of the three findings: Altered Mental Status, Fever and/or Headache (or
any HIV patient with any one of the above).
2. Contraindications: Focal neurological defect (check head CT) or Coagulopathy
3. Positioning: Place the patient on their side with legs drawn up and chin to their chest. Raise
the bed for your comfort.
4. Preparation: Find your mark palpate the highest point of the iliac crests and locate the
L3/L4 or L4/L5 vertebral interspace. Liberally sterilize and drape the patient. Prepare your kit
by putting together your manometer and arranging your test tubes.
5. The needle: Carefully inject a few ml of lidocaine (always aspirate first). Using your spinal
needle, bevel up, insert between the interspace angling toward the umbilicus and slightly
cranially until you feel a pop. Transiently remove the stylet to confirm CSF flow.
6. Lay flat for 6-12 hours and encourage fluid intake to prevent headaches.
7. Measurements:
- Opening Pressure: CSF < 15 cm = normal. >20 cm is abnormal.
- Color: Should be clear, colorless
- Lab analysis: (label your tubes)
- TUBE 1: Cell count (<5 cells, 0 PMNs)
- TUBE 2: Glucose and Protein: About 1 ml.
- Normal Glucose: about 2/3 of blood sugar.
- Normal Protein: <40. Blood raises protein concentration by 1/1000 RBCs.
- TUBE 3: Culture and gram stain: about 5 ml of CSF; also consider cryptococcal
antigen, VDRL, cytology (may require larger volumes), AFB, fungal cultures, India ink,
lyme titers, HSV/MTD/EBV/West Nile Virus/JC virus PCR where indicated. Also
oligoclonal bands (requires simultaneous red top tube).
- TUBE 4: repeat cell count, if traumatic, can subtract 1 WBC : 500 RBC
Test Bacterial Viral Fungal Tubercular
Open Pressure Elevated Usually normal Variable Variable
WBC count >=1,000 per mm3 <100 per mm3 Variable Variable
Cell differential PMNs* lymphocytes lymphocytes lymphocytes
Protein Mild to marked elevation Normal to elevated Elevated Elevated
CSF-to-serum
glucose ratio
Normal to marked
decrease
Usually normal Low Low
61
SUBACROMIAL INJECTION
1. Indications: For symptomatic relief of pain from rotator cuff tendonitis, subacromial bursitis, or
impingement syndrome.
2. Contraindications: Absolute: severe coagulopathy, overlying cellulitis, allergy to injectables,
joint replacement, fracture, septic joint
- Relative: bacteremia, out of control diabetes, supratherapeutic INR
3. Informed Consent: Discuss risks including infection (~1:10,000), bleeding, no improvement,
tendon/ligament rupture, steroid flare, skin/soft tissue depigmentation and atrophy. Have
patient sign consent
4. Positioning: Patient seated on exam table with shirt off
5. Preparation: Palpate the spinous process of scapula until you reach the posterolateral corner
of the acromion. Mark your site 1cm below this posterolateral corner with cap of needle. Clean
site with 3 betadine swabs and an alcohol swab. For injection mix ~4 cc of anesthetic (1%
lidocaine without epi or 0.25% marcaine) with 1cc (40mg) kenalog or equivalent steroid.
6. Procedure: After spraying ethyl chloride (optional), advance your needle into the marked
space usually all the way to the hub of the needle. Angle the needle towards the coracoid
process and slightly up, just under the acromion. When injecting you should not encounter
resistance, which could indicate you are in a tendon. Reposition the needle and re-inject.
Remove needle and apply pressure with gauze. Apply bandaid.
7. Post-injection instructions: Inform patient to contact you if develop fevers, chills, erythema at
injection site or severe pain. The cortisone typically takes 3 days to take effect
62
KNEE ASPIRATION/INJECTION
1. Indications: For diagnosis of septic joint, gout or hemarthrosis. For symptomatic relief of pain
from osteoarthritis, gout, bursitis.
2. Contraindications: Absolute: severe coagulopathy, overlying cellulitis, allergy to injectables,
joint replacement, fracture, septic joint (for steroid injection)
- Relative: bacteremia, out of control diabetes, supratherapeutic INR
3. Informed Consent: Discuss risks including infection (~1:10,000), bleeding, no improvement,
tendon/ligament rupture, steroid flare, skin/soft tissue depigmentation and atrophy. Have
patient sign consent
4. Positioning: Patient seated upright with knees dangling (injection), or patient lying supine
(aspiration or injection).
5. Preparation: Mark your site with cap of needle. For seated patient the space between the
femoral condyle, tibial plateau, patellar edge and patellar tendon either medially or laterally.
For supine patient just under superolateral or supermedial patella. Clean site with 3 betadine
swabs and an alcohol swab. For aspiration you need several 20 cc syringes and at least a 22
gauge, 1 inch needle. For injection mix ~4 cc of anesthetic (1% lidocaine without epi or
0.25% marcaine) with 1cc (40mg) kenalog or equivalent steroid.
6. Procedure: After spraying ethyl chloride (optional), advance your needle into the marked
space and aspirate. For sitting patient the needle should be parallel to the floor and usually no
fluid is able to be aspirated. For supine patients remember the angle of the patellar facets
when introducing the needle. If performing injection you should not encounter resistance when
injecting the fluid. Remove needle and apply pressure with gauze. Apply bandaid.
7. Post-injection instructions: Inform patient to contact you if develop fevers, days to take
effect.
8. Measurements:
- Cell count and differential
- Gram stain and culture can help with antibiotic choice
- Crystal analysis
9. Joint Fluid Analysis:
Test Normal Non-
inflam
Inflam Sepsis Crystal Hemo
Clarity Clear Slightly
turbid
Turbid Turbid Turbid Bloody
Color Yellow Yellow Yellow Gray/
green
Yellow-
milky
Red/
Brown
Viscosity High Reduced Low Low Low Reduced
WBC/uL 0 - 200 0 - 2000 2000 -
100,000
>50,000 500 -
200,000
50 -
10,000
%Polys <25 <30 >50 >90 <90 <50
Glucose
difference
0 - 10 0 -10 0-40 20-100 0-80 0-20
Crystals Absent Absent Absent Absent Present Absent
Culture Sterile Sterile Sterile Positive Sterile Sterile
63
INFECTIOUS DISEASE
FEVER
1. Basic Concepts:
- Defined variably as >38.5C (101.3F), unless immunosuppressed or neutropenic then a
single measurement >38.3C (101F) or >38.0C (>100.4F) over 1 hour.
- Assess current and past antibiotic use.
- Identify most likely source (GI vs. pulmonary vs. line vs. GU vs. CNS, etc.)
- Differential is broad: infection (as above also sinus, decub ulcers, etc) vs. drug fever vs.
autoimmune vs. neoplasm vs. PE/DVT.
2. Physical Exam: Vitals for hypotension, tachycardia, tachypnea, lung exam, oropharynx, lymph
nodes, abdominal exam, decubs, check all lines.
3. Workup: CBC with diff, UA (eos, sediment), blood cultures x 2 BEFORE antibiotics (usually
dont need to repeat within 24-48hrs), urine cultures, CXR; consider sputum culture, LP, stool
studies if clinically indicated. Consider upgrading level of care to DOU etc if patient meeting
multiple SIRS criteria or septic
4. Intervention: based on workup, but for unstable patients cover broadly AFTER blood cultures.
Consider nosocomial pathogens if patient has been in house >3 days. If patient on antibiotics,
get cdiff toxin along with stool cultures. If patient has been on broad spectrum antibiotics,
consider fungal infections and get fungal blood cultures.
- Note, for fever spike through antibiotics consider:
- Drug failure (not susceptible)
- Development of resistance in previously susceptible organisms
- Drug fever
-
5. Neutropenic Fever (ANC <500): see Heme/Onc section. Treatment usually 3rd generation
cephalosporin with pseudomonal coverage (eg Ceftaz) +/- Vancomycin (nearly always in our pt
pop) if suspect line infection or colonization with MRSA. May consider adding Gentamicin for
synergy.
CLINICAL PEARLS
1. The way to think about antibiotics is to always ask yourself this question- What organisms I
not covering with this antibiotic regimen?
2. Review: Pathology and films yourself
3. Find the Source: Cornerstone of effective infectious disease management is source control:
remove the foreign body ie infected line, even if its a tunneled catheter, drain the infected fluid,
debride the infected tissue. Staph aureus and candidal line infections usually require removal.
4. Antibiotics are NOT antipyretics
5. Treat the patient: NOT the culture. For example, positive sputum culture does not equal
clinical PNA. Also, in the absence of neutropenia, + UA without pyuria excludes a UTI.
6. Antibiotic Misconceptions: IV vancomycin does not work for C. difficile colitis. Vancomycin is
not the best drug for MSSA and should not be used except in the setting of beta lactam
allergy. Cephalosporins are safe in PCN allergies, unless there is a history of anaphylaxis.
7. Complex Regimen = Complex Change: Never add one drug to failing regimen (eg TB, HIV).
8. Approach to Endocarditis: If you suspect BE get blood Cx before Abx. A negative TTE does
not rule out endocarditis (sensitivity ~50%). Aminoglycosides have no impact on the mortality
of Staph aureus bacteremia/endocarditis but they do decrease the duration of bacteremia so
limit treatment to 14 days.
9. Candida as a Pathogen: Not in the sputum or bronch washes. Usually not in the urine
(exceptions are renal transplant, stents, pyelonephritis). Candidemia requires catheter
removal, evaluation by Ophthalmology for retinal disease/endophthalmitis, and antifungal
therapy.
10. Hospital acquired infections due to gram negative bacteria are bad news. These organisms are
highly efficient at up-regulating or acquiring genes that code for mechanisms of antibiotic drug
resistance, especially in the presence of antibiotic selection pressure. Furthermore, they have
available to them a plethora of resistance mechanisms, often using multiple mechanisms
against the same antibiotic or using a single mechanismto affect multiple antibiotics See
recent review, NEJM May 13, 2010 for details.
11. Community Acquired MRSA is bad news, just like hospital acquired MRSA. See review in
Lancet, May 1, 2010
64
MENINGITIS
If you're worried about meningitis to treat it, attempt to obtain a sample of CSF after CT head if safe.
1. Common Bacterial Pathogens:
- 16-50yo N meningitides, Strep pneumo
- >50yo S pneumo, N meningitides, Listeria monocytogenes, Aerobic gram negative Bacilli
2. Antibiotics:
- vancomycin + 3rd gen cephalosporin for pts 16-50yrs
- add ampicillin for >50yrs or HIV+
3. Decadron: initiate with or prior to first dose of abx (10mg q6) and continue for 4 days.
- Unclear benefit if started after the first dose of abxs; and no clear benefit in fungal meningitis
4. Fungal Meningitis: Crypto, Cocci, Histo in HIV or other immunocompromised patients.
- For Crypto: 2 week induction with 5FC and Amphotericin B( with aggressive hydration and
premedication or liposomal preparation) followed by prolonged course of oral fluconazole.
5. TB Meningitis: adjunctive Rx w/ IV decadron (0.4mg/kg x wk1, 0.3mg/kg x wk2, 0.2mg/kg x
wk3, and 0.1mg/kg x wk4, then po decadron taper) reduced mortality (RR 0.69).
6. Nosocomial bacterial meningitis may result from invasive procedures (e.g., craniotomy,
placement of internal or external ventricular catheters, lumbar puncture, intrathecal infusions of
medications, or spinal anesthesia), complicated head trauma, or in rare cases, metastatic
infection in patients with hospital-acquired bacteremia. January 14, 2010 NEJM
References: NEJM 2006;354:44-53, NEJM 2004;351:1741-1751
PNEUMONIA
1. Basic Concepts:
- Pneumonia is one of the most common conditions you will see on the inpatient medical
wards and in the ICU. Over 1 million patients are hospitalized with pneumonia annually in the
U.S., and it is the most common infectious cause of death.
- Pneumonia is not as simple as it used to be. Antibiotic therapy needs to be tailored to the
patients specific risks/comorbidities. Getting initial empiric antibiotic selection right is
CRITICAL, as inpatient mortality rates more than double when antibiotic selection is not
consistent with guidelines, and 30-day mortality increases by a factor of 5.
- Pneumonia comes in several flavors: community-acquired (CAP), hospital-acquired (HAP),
ventilator-associated (VAP), and healthcare-associated (HCAP). HAP, VAP, and HCAP are
all treated the same, and require broader spectrum antibiotics than CAP. The definitions of
HAP and VAP are fairly self-explanatory, so we will focus on distinguishing CAP from HCAP,
and the special circumstances of CAP treated in the ICU.
Apart from being associated with increased morbidity and mortality, suspected hospital-
acquired pneumonia in the ICU can lead to the inappropriate use of antibiotic drugs,
contributing to bacterial drug resistance and increases in toxic effects and health care costs.
NEJM 5/13/10
- If you are too busy to read the rest of this section, all you really need to know is contained in
the PNEUMONIA ORDERSET on EPIC. If you use this to order antibiotics for all your
pneumonia patients you will get it right every time. (OK, it doesnt work for special
circumstances like TB or PCP) The orderset is intended to speed the process of ordering
treatment for pneumonia patients, and its use is STRONGLY ENCOURAGED.
2. Community acquired pneumonia (CAP): pneumonia occurring the outpatient setting > 90
days after a hospitalization and not meeting the criteria for healthcare associated pneumonia
- Usual etiology: Streptococcus pneumonia, H. influenza, other gram negatives, atypical
(Mycoplasma, Chlamydia, Legionella), viral
- Recommended treatment:
- Non-ICU:
- 3
rd
generation Cephalosporin (e.g. Ceftriaxone) PLUS Macrolide (e.g. Azithromycin
or doxycycline)
OR
- Respiratory Fluoroquinolone (e.g. Moxifloxacin)
- ICU:
- 3
rd
generation Cephalosporin (e.g. Ceftriaxone) PLUS Macrolide (e.g. Azithromycin)
or respiratory fluoroquinolone (e.g. Moxifloxacin)
65
3. Healthcare associated pneumonia (HCAP): pneumonia occurring in any patient:
- Hospitalized in an acute care hospital for > 2 days within 90 days of the infection
- Residing in a SNF or long-term care facility
- Receiving intravenous antibiotic therapy, chemotherapy, or wound care within the past 30
days of the current infection
- Attending a hospital or hemodialysis clinic within the past 30 days
- Family member with multidrug-resistant pathogen
- Usual etiology: Streptococcus pneumonia, enteric gram negatives, MSSA/MRSA, ESBL
producing gram negative rods, Pseudomonas, Acinetobacter
- Recommended treatment: Should cover MRSA and include 2 antipseudomonal agents
- Vancomycin PLUS Piperacillin/tazobactam PLUS Ciprofloxacin
OR
- Vancomycin PLUS Aztreonam PLUS Ciprofloxacin (if Penicillin Allergic)
4. Ventilator associated pneumonia (VAP): pneumonia occurring > 48-72 hr after endotracheal
intubation and mechanical ventilation.
- Recommended Treatment: Same as HCAP
5. Risk factors for multidrug-resistant (MDR) pathogens:
- Antimicrobial therapy in last 90 days
- Current hospitalization 5 days
- High frequency of antibiotic resistance in community or hospital unit
- Any of the HCAP criteria
- Immunosuppressive disease or therapy
6. Consider ICU Admission in CAP if the patient has either:
- One of the major criteria:
- Need for invasive mechanical ventilation
- Septic shock with the need for pressors
- Three of the minor criteria:
- Respiratory rate 30 or PaO2/FiO2 250 or NIPPV
- Multilobar infiltrates on CXR
- Confusion or disorientation
- Uremia (BUN 20 mg/dL)
- Leukopenia (WBC < 4000) as a result of infection alone
- Thrombocytopenia (platelets < 100,000)
- Hypothermia (T < 36C)
- Hypotension requiring aggressive fluid resuscitation
7. Other tips on pneumonia:
- Whenever possible, obtain blood and sputum cultures, especially for HCAP/VAP/HAP. If
positive, it will allow you to narrow antibiotic therapy.
- More commonly cultures will be negative or unobtainable. In that case, it is usually safe to
narrow antibiotic therapy for patients with HCAP who have achieved clinical stability.
CLINICAL JUDGMENT is imperative here as there are no evidence-based guidelines.
- Patients rarely decompensate due to their pneumonia once they achieve clinical stability.
There is no benefit to inpatient observation of a stable patient just to ensure they tolerate a
transition from IV to oral antibiotics.
- Although important, obtaining blood and sputum cultures should not delay antibiotic
administration. In general the yield of cultures is low, and delays in antibiotic administration
increase mortality.
-In patients crashing on the floor with a suspected pneumonia versus volume overload, we
often simultaneously diuresis the patient if their vitals and renal function can handle it
Consider Influenza. Infection with the 2009 H1N1 virus caused a broad spectrum of clinical
syndromes, ranging from afebrile upper respiratory illness to fulminant viral pneumonia.- May
6, 2010 NEJM
66
8. Quality control: National Pneumonia Project: www.nationalpneumonia.org
- Abx given within 4hrs after arrival to ED was associated w/ reduced length of stay, 17%
reduction in mortality
- Blood cultures before Abx.
- Blood cultures within 24 hrs of admission or transfer to ICU.
- Oxygen assessment
- Antibiotic selection: consider hospital-acquired pneumonia in pts w/ recent previous
hospitalization(<90days) and fungal/PCP in immunocompromised.
- Vaccination pneumovax and influenza vaccine prior to discharge.
- Smoking cessation counseling, if appropriate
- If indicated, urine antigen tests for Legionella pneumophila and S. pneumoniae (all common
serotypes). Both tests are specific but not completely sensitive.
- PCP:
- Silver stain (cytology) of induced sputum (sensitivity 50% in HIV patients) or
bronchoalveolar lavage (BAL); must fill out paper requisition.
- TMP-SMX always the first choice. If pt has sulfa or septra allergy, can use clinda +
primaquine or IV pentamidine. If severe PCP, consider desensitization for septra (ask
pharmacist).
- Prednisone if paO2<70 or A-a grad >35 in RA ABG.40mg bid x 5 days, 40mg qday x 5
days, then 20mg qday to complete 21 day course.
- Nasal swab or wash for influenza antigen. Most sensitive <48 hours
- AFB smears and culture of sputum to diagnose Mycobacterial infections. If there is a +
smear, do a PCR assay to rule-out TB. All HIV+ patients should have a sputum PCR sent in
addition to 3 smears. Once positive TB identification, TB cultures can be repeated every 2
weeks until negative.
- At VA: send one sputum q8h x 3 (one must be a morning sample).
- At UCSD: send daily sputum x 3 days.
References: American Thoracic Society and Infectious Diseases Society of America. Guidelines for
the Management of Adults with Hospital-acquired, Ventilator-associated, and Healthcare-associated
Pneumonia. Am J Respir Crit Care Med 2005; 171: 388-416. Infectious Diseases Society of America
and American Thoracic Society. Consensus Guidelines on the Management of Community-Acquired
Pneumonia in Adults. CID 2007; 44(Suppl 2): S27-72. Gilbert DN, et al. The Sanford Guide to
Antimicrobial Therapy 2007. 37th ed. Antimicrobial Therapy Inc. 2007: 85-92. Arch Intern Med 2004;
164:637644 85-92. Arch Intern Med 2004; 164:637644
67
LINE ASSOCIATED INFECTIONS
Non-tunneled central venous catheters
Tunneled central venous catheters and implantable devices
68
UTI
Tips:
- Never Collect urine for culture from a drainage bag. If a patient cannot provide a clean catch
specimen, catheterize the patient to obtain the urine. If a urine specimen is obtained from a
urinary catheter - label it.
- In men there are no guidelines for interpreting colony counts and the isolation of potential
urine pathogens in any number should be regarded with suspicion.
- In women, a voided urine should have <104 CFU/ml; values greater than that are abnormal.
However, you should not diagnose and treat urinary tract infections in patients with normal UA,
as the culture results are either false positives or the patient has asymptomatic bacteriuria. A
catheterized urine should be sterile.
Pathogenic Organisms by Gram Stain
Gram Positive Cocci
in clusters - Staph
in chains - Strep
Staph epi (coagulase negative)
Staph aureus (coagulase positive)
Strep pneumo (ovals in pairs)
o-hemolytic strep
ie, viridans, salivarius
|-hemolytic strep
Group A strep (this is what rapid strep
tests look for)
Group G strep
Groups C, D, E, F, G
Enterococcus species (also Group D)
Faecalis (sensitive to ampicillin)
Faecium (resistant to ampicillin)
Gram Negative Cocci
Neisseria gonorrheae
Moraxella catarrhalis
(both are aerobic diplococci)
Gram Positive Rods
Actinomyces species*
Bacillus anthracis
Clostridium species*
Corynebacterium diphtheriae
Listeria species (|-hemolytic)
Nocardia species (branching)
Diphtheroids species (both aerobic and
anaerobic)
*Anaerobic organisms
Gram Negative Rods
Escherichia coli
Proteus species
Klebsiella species
Enterobacter species
Citrobacter species
Pseudomonas aeruginosa
Salmonella species
Shigella species
Acinetobacter species (coccobacillus)
Haemophilus influenza
Bordetella pertussis
Vibrio cholerae (comma shaped)
Campylobacter species*
Bacteroides species*
Yersinia species
Brucella species*
Pasteurella multocida
Francisella species
Zoonoses
Coliform
69
HIV
Guidelines for treatment of HIV change rapidly; up-to-date information may be obtained from:
http://AIDSinfo.nih.gov
1. Indications for ARV*:
- Any pt. with AIDS-defining illness or HIV nephropathy regardless of CD4 count (AI)
- Asymptomatic patients with CD4 T cells <350 (AII)
- Pregnant women (AI)
- HBV coinfection when treatment for Hepatitis B is indicated (BIII)
- Regimen must include TDF and either FTC or 3TC
- Consider ARV for asymptomatic patients with CD4 T cell counts >350 who also have plasma
HIV RNA >100,000 or rapid decline in CD4 count (>120 cells/mm3/annum)
- Defer ARV for patients with CD4 T cell counts >350 and plasma HIV RNA <100,000.
*Delay initiation of HAART 2-8 weeks in setting of TB (BIII), PCP, MAC, or CM (CIII)
2. NNRTI-Based Regimens (1 NNRTI + 2 NRTIs):
- Preferred NNRTI (AII):Efavirenz (except during 1st trimester of pregnancy or in women
w/high pregnancy potential*)
- Alternative NNRTI (BII Nevirapine may be used as an alternative in adult females w/CD4 T
cell counts</=250 and adult males with CD4 T cell counts </= 400. Increase risk of hepatitis
when Nevirapine is given to women whose CD4>250 and men whose CD4 >400.
- Women with high pregnancy potential are those who are trying to conceive or who are
sexually active with men and not using effective and consistent contraception.
3. PI Based Regimens ( 1 or 2 PIs + 2 NRTI) (alphabetical):
Preferred PIs: Alternative PIs (BII):
atazanavir + ritonavir* (AIII) atazanavir**
fosamprenavir + ritonavir* q 12 hrs(AII) fosamprenavir
lopinavir/Ritonavir (Kaletra) q 12 hrs(AII) fosamprenavir + ritonavir* q day
lopinavir/ritonavir (kaletra) q day saquinavir + ritonavir
- Ritonavir @ daily doses of 100 400 mg used as a pharmacokinetic-booster
- Ritonavir 100 mg q day must be given when tenofovir or efavirenz is used with atazanavir.
4. Dual-Nucleoside Options as Part of Initial Combination Therapy:
- Preferred dual NRTI (AII) (alphabetical)
- Abacavir/lamivudine (Epzicom)*(co-formulated) (in patients negative for HLA-B*5701)
- Tenofovir/emtricitabine (Truvada)* (co-formulated)
- Alternative dual-NRTI (BII) (in order of preference)
- Zidovudine/lamivudine (Combivir)* (co-formulated)
- Didanosine + (lamivudine or emtricitabine)
5. Drug Interactions:
- PI: PIs are metabolized in the liver by the cytochrome P450 CYP 3A4. There are numerous
drugs with significant interactions with PIs. A few of the examples include statins, benzos,
calcium channel blockers, cyclosporine, tacrolimus, dilantin, rifamycins, erectile
dysfunction medications (eg. viagra), azole antifungals, macrolides, oral contraceptives
(OCPs) and methadone.
- NNRTI: NNRTIs are substrates of CYP3A4 and in some cases are also inducers
(nevirapine) or a mixed inducer and inhibitor (efavirenz).
- NRTI: NRTIs do not undergo metabolism through the CYP pathway. Beware of
interactions.
- Zidovudine and ganciclovir or valganciclovir: additive bone marrow suppression
- Zidovudine and Ribavirin: severe anemia may occur
- Zidovudine and stavudine: Do not use together pharmacologic antagonism
- Tenofovir and Didanosine: Increased levels of Didanosine (adjust ddI dose)
- Atazanavir and Tenofovir: Decrease levels of Atazanavir (interaction can be
overcome with addition of ritonavir)
- Cidofovir/foscarnet/Amphotericin B/Aminoglycosides and Tenofovir: watch for renal
toxicity
70
- CCR5 Antagonists: Maraviroc is metabolized by CYP 3A4. Requires decreased dosing
when given with 3A4 inhibitors (eg ritonavir), and increased dosing when given with 3A4
inducers (eg rifampin).
*Web Site for HIV drug interactions: HIVinsite.org and search for drug interactions.
References: Guidelines for the Use of Antiretroviral Agents in HIV-1 Infected Adults and Adolescents,
Oct 10,2006. Benson CA, Kaplan JA, Masur H, Pau A, Holmes KK. Treating Opportunistic Infections
among HIV-Infected Adults and Adolescents: Recommendations from CDC, the National Institutes of
Health, and the HIV Medicine Association/Infectious Diseases Society of America. Clinical Infectious
Diseases 2005;40:S131-235.
OPPORTUNISTIC INFECTION: PRIMARY PROPHYLAXIS
CD4 Count Pathogen Primary Regimen Alternative
Regimen
When to DC
<200/mm3 Pneumocystis
jiroveci
TMP/SMX DS
160 mg trimethoprim/
800 mg
Sulfamethoxazole
1 tablet q 24 hrs
OR
TMP/SMX SS
80 mg trimethoprim/
400 mg
Sulfamethoxazole
1 tablet q 24 hrs
Dapsone
100 mg q 24 hrs
OR
Dapsone 100 mg q 24 hrs AND
Pyrimethamine 50 mg q wk AND
leucovorin 25 mg po q wk
OR
Aerosolized Pentamidine
300 mg q month via Respirgard II
nebulizer
OR
Atovaquone susp 1500 mg po q 24
hrs.
OR TMP/SMX DS 3X/wk
DC prophylaxis
when CD4 >200
for > 12 wks in
response to
HAART
< 100/mm3 Toxoplasma
Gondii
TMP/SMX DS
160 mg trimethoprim/
800 mg
Sulfamethoxazole
1 tablet q 24 hrs
Dapsone 50 mg q 24 hrs AND
Pyrimethamine 50 mg q wk AND
leucovorin 25 mg q wk
OR
Mepron/Atovaquone susp
750 mg q 6-12 hrs AND
Pyrimethamine 25mg q24 hrs AND
leucovorin 10 mg q 24 hrs.
DC prophylaxis
when CD4 >200
for >12 wks in
response to
HAART
< 50/mm3 Mycobacteriu
m Avium
complex
Azithromycin
1200 mg q wk
OR
Clarithromycin
500 mg q 12 hrs
Rifabutin 300 mg q 24 hrs DC prophylaxis
when CD4 >100
for 12 wks in
response to
HAART
< 50/mm3 CMV Valganciclovir 900
mg q 24 hrs
* Ophthalmic exam q
3 mos to r/o CMV
retinitis
* May elect to follow
pts closely without
prophylaxis (this is
what we do at Owen)
* Consider
prophylaxis for high
risk groups: Gay
men: 35% risk, +
plasma CMV PCR:
43% risk
Ganciclovir 1 gm q 8 hr DC prophylaxis
when CD4 >100
150
Any CD4
cell count
Mycobacteriu
m
tuberculosis
Isoniazid 5 mg/kg q
24 hrs (max 300 mg
q 24 hrs)
AND
Pyridoxine (B6) 50
mg po q 24 hrs X 9
months
Rifampin 600 mg q24 hrs for 4
months
OR
Rifabutin 300 mg q 24 hrs for 4
months
*TBTC 26 is a multicenter
randomized study comparing daily
self administered INH 300 mg for 9
months to Isoniazid 900 mg q wk
AND Rifapentine 600 mg q wk for 12
wks. Trial includes HIV+ and HIV-
patients. Call Peach Francisco @
543-7719 to enroll
NA
71
OPPORTUNISTIC INFECTION: SECONDARY PROPHYLAXIS
Pathogen Regimen When to DC
Candida: frequent or
severe recurrences of
oropharyngeal OR
esophageal candidiasis
Fluconazole 100 200 mg q 24 hrs Suppressive therapy should be balanced
with risk of fluconazole resistant
candidiasis, costs and severity/frequency of
candidial infections.
Coccidioidomycosis Fluconazole 400 mg q 24 hrs
OR Itraconazole 200 mg q 12 hrs
(Liquid formulation is preferred)
Lifelong suppression required regardless of
CD4 count
Cytomegalovirus Retinitis
(CMV)
Valganciclovir 900 mg q 24 hrs DC prophylaxis when CD4 >100-150
for 6 months
AND No evidence of CMV disease
AND Ophthalmic exam w/o active disease
Cryptococcus Fluconazole 200 mg q 24 hrs. CD4 cell count > 100-200 X 6 months
AND Pt has completed initial therapy
Histoplasmosis Itraconazole 200 mg q 24h
(Liquid formulation is preferred)
Consider when CD4 >150
AND s/p 12 months of anti-fungal therapy
AND 6 months of ARVs
Mycobacterium avium
complex
(MAC)
Clarithromycin 500 mg q 12 hrs AND
Ethambutol 15 mg/kg q 24 hrs
Plus/minus Rifabutin 300 mg q 24 hrs.
OR
Azithromycin 500 mg q 24 hrs AND
Ethambutol 15 mg/kg q 24 hrs
Plus/minus Rifabutin 300 mg q 24 hrs.
CD4 cell count > 100 for 6 months
AND s/p 12 months of MAC therapy
AND Pt is clinically asymptomatic
Pneumocystis jiroveci
(PCP)
Same as primary prophylaxis CD4 cell count >200 for > 12 wks
Toxoplasma gondii Same as primary prophylaxis CD4 > 200 for 6 months
AND s/p Treatment for Cerebral
toxoplasmosis
OPPORTUNISTIC INFECTION: TREATMENT
Pathogen/Disease Preferred Regimen Comments
Esophageal Candidiasis Fluconazole 100 - 400 mg q24h x 14-21d Nystatin or Clotrimazole for oral candida;
Consider endoscopy if no improvement in
7 10 days
Coccidioidomycosis Itraconazole* 200mg BID OR Fluconazole 400-
800mg;
Ampho B 0.7-1mg/kg/d for disseminated disease
Fluconazole 800mg daily for CNS disease
*Liquid formulation preferred
Ampho B in preganancy or if life-
threatening disease; Intrathecal Ampho B
may be needed in refractory CNS cases;
Itraconazole preferred for non-CNS
disease by some experts
Cytomegalovirus (CMV)* Valganciclovir 900mg PO BID x3 wks then 900
mg daily +/- ocular implant (retinitis);
Ganciclovir 5mg/kg IV q12h PLUS Foscarnet
90mg/kg IV q12h for encephalitis/myelitis
Treatment varies by site and severity of
infection; no RCTs to validate
combination therapy in CNS disease,
typically 3 weeks induction then maint.
Cryptococcus neoformans
meningitis*
Amphotericin B Liposomal complex 4mg/kg/d IV
AND Flucytosine 25 mg/kg PO q6h x 14d; then
Fluconazole 400 mg PO daily x 8 10 weeks;
then 200 mg PO daily
If OP >25 cm H2O, take off CSF until
pressure decreases by 50% and do daily
therapeutic LP until OP <20 cm x2-3d.
Consider following flucytosine levels,
peak <100 g/mL to minimize toxicity.
Steroids not indicated in routine CM.
Histoplasma capsulatum Itraconazole* 200mg TID x3 d then 200mg BID
x12w;
Amphotericin B 0.7 mg/kg IV q24h for severe
disease until stable, then Itraconazole as above
*Liquid formulation preferred
Itraconazole preferred over fluconazole
unless CNS disease, where 800mg/d
fluconazole should follow 12-16 weeks of
Ampho B 0.7mg/kg IV q24h
Mycobacterium avium
complex (MAC)
Clarithromycin 500 mg q 12 hrs (AI) or
Azithromycin 500 mg q 24 hrs (AII) AND
Ethambutol 15-25 mg/kg q 24 hrs
Plus/minus Rifabutin 300 mg q 24 hrs
Fluoroquinolone or amikacin may be
added in refractory cases. Caution
rifabutin interaction with ARVs; dose-
adjustment of rifabutin may be necessary.
Pneumocystis jiroveci (PCP)* TMP/SMX 5mg/kg IV/PO q8 hours x 21 d.
OR Pentamidine 4mg/kg IV daily
OR TMP 5mg/kg q8 plus Dapsone 100mg daily
OR Clindamycin 1.8-2.4 g/d PO/IV plus
primaquine 30 mg base/d PO
TMP/SMX preferred
Add prednisone if paO2<70 or A-a grad
>35 in RA ABG. 40mg bid x 5 days, 40mg
qday x 5 days, then 20mg qday to
complete 21 day course.
Toxoplasma gondii
encephalitis
Pyrimethamine 200mg x1 then 75 mg/d AND
Leucovorin 10-20 mg/d AND Sulfadiazine 1.5g
q6h x 6 weeks then dose x 6 wks
Decadron if significant mass effect;
treatment often empiric, consider biopsy if
no improvement in 1-2 weeks
72
ANTIBIOTICS
Antibiotic Mechanism Static
or
Cidal
Spectrum Toxicities Common Clinical/Empiric
Usages
Vancomycin Inhibits
peptidoglycan
chain formation
in bacterial cell
wall
cidal MRSA, MSSA,
strep,
enterococcus
Fever, chills, rash, nephrotoxicity
(extremely rare), but vanc must be
renally adjusted to avoid such things
as ototoxicity. Redman syndrome
very common with rapid infusion
infuse over 1-2 hrs
IV empiric therapy any
infection requiring IV abx in
which MRSA is suspected.
Empiric oral therapy for
refractory C. dif or empirically
in very old or very ill.
Nafcillin/
Oxacillin
Inhibits
crosslinking of
peptidoglycan
chains, -
lactamase
resistant
cidal MSSA, strep All -lactams may cause:
GI Upset
Hypersensitivity, Anaphylactic shock
(rare), hepatic, hematologic and
neurologic abnormalities .
Nafcillin may cause neutropenia or
interstial nephritis.
IV therapy for MSSA
bacteremia/endocarditis, MSSA
pneumonia, osteomyelitis, or
MSSA skin infections.
Ampicillin/
Amoxicillin
-lactam:
Inhibits
synthesis of
peptidoglycan
layer of
bacterial cell
wall.
cidal Strep,
enterococcus,
Listeria,
Menningococcus,
M. catarrhalis, H.
flu
Hypersensitivity
Anaphylactic shock
GI upset
Candidiasis, TEN, Liver/Hematologic
abnormalities, renal adjustment is
necessary
Ampicillin in meningitis if
listeria is suspected
Amoxicillin in sinusitis, otitis
media, or UTI with sensitive
organisms, (must be taken q8h)
Augmentin
(Amox/clavulana
tic acid)
-lactam:
Inhibits
synthesis of
peptidoglycan
layer of
bacterial cell
wall.
Beta-lactamase
inhibitor:
irreversibly
binds to beta-
lactamases
cidal Strep,
enterococcus,
MSSA, listeria,
many non-
resistant GNRs,
anaerobes
Similar to other -lactams
Extended courses may cause C. Dif
Sinusitis, UTIs, PNA with
sensitive organism
Zosyn
(Piperacillin/tazo
bactam)
-lactam:
Inhibits
synthesis of cell
wall
Beta-lactamase
inhibitor:
irreversibly
binds to beta-
lactamases
cidal Strep, MSSA,
enterococcus,
resistant GNRs,
anaerobes
Diarrhea, GI upset, headache,
candidiasis,
C. dif colitis
Empiric therapy for
hospital/healthcare associated
pneumonia.
Carbapenems
Reuire ID
approval
-lactam:
Inhibits
synthesis of
peptidoglycan
layer of
bacterial cell
wall
cidal Strep, MSSA,
resistant GNRs,
anaerobes
Similar to other -lactams Infections with extended
spectrum -lactamase producing
(ESBL) GNRs, including
meningitis
1
st
Generation
Cephalosporin
Cefazolin
Cephalexin
-lactam:
Inhibits
synthesis of
peptidoglycan
layer of
bacterial cell
wall
Cidal Strep/MSSA (not
generally drug of
choice), some E.
Coli, Klebsiella,
and Proteus
Similar to other -lactams Skin infections with
strep/MSSA (although keflex is
not as effective against MSSA)
3
rd
Generation
Cephalosporin
Ceftriaxone
Ceftazidime
-lactam:
Inhibits
synthesis of
peptidoglycan
layer of
bacterial cell
wall
Cidal CFT- strep,
MSSA, most non-
resistant GNRs ,
oral anaerobes
CTD- expanded
coverage of
pseudomonas,
less effective
against strep and
MSSA
Similar to other -lactams:
Remember people on these drugs for
extended periods need to be
monitored with periodic CBCs and
liver panels.
CFT-
empiric therapy for:
Community acquired
pneumonia (with macrolide),
UTIs/Pyelo requiring IV abx,
Meningitis
SBP
CTD-
Empiric therapy for
Suspected pseudomonas
4
th
Generation
Cephalosporin
-lactam:
Inhibits
synthesis of
peptidoglycan
layer of
bacterial cell
wall
Cidal Strep, MSSA,
many resistant
GNRs, oral
anaerobes
Similar to other -lactams Infections with extended
spectrum -lactamase producing
(ESBL) GNRs, including
meningitis
Aztreonam Inhibits
mucopeptide
synthesis in
bacterial cell
wall
cidal GNRs Elevated LFTs, C. dif colitis, GI
upset, hypersensitivity
Use for resistant GNR
infections when there is a true
-lactam allergy
Ciprofloxacin Inhibits DNA
gyrase
cidal Non-resistant
GNRs
Most frequent:
CNS toxicity, GI upset.
Good choice for UTIs (although
>20% E. Coli now resistant),
often paired with metronidazole
as empiric therapy of enteric
infections
Moxifloxacin Inhibits DNA
gyrase and
topoisomerases
cidal Strep, non-
resistant GNRs,
oral anaerobes
Most frequent:
CNS toxicity, GI upset. Remember
quinolones can also cause QT
prolongation.
Empiric therapy for CAP
(penetrates excellently into
lungs and urine; but not FDA
appeoved indication
Antibiotic Mechanism Static
or
Cidal
Spectrum Toxicities Common Clinical/Empiric
Usages
73
Aminoglycocide
Gentamicin
Amikacin
Tobramycin
Blocks protein
synthesis by
binding to 30S
subunit of
bacterial
ribosome
cidal Primarily for most
GNRs including
pseudomonas;
adjunctive therapy
for complicated
MSSA infections
and used for
enterococcal
infections
combined with a
cell wall inhibitor
Most frequent: Auditory
Neurotoxicity, CNS Toxicity,
Nephrotoxicity (cumulative effect, In
general, one dose of AG does not do
much harm to kidneys), Ototoxicity,
Renal Disease
Empiric therapy for in which
pseudomonas is possible
pathogen, GU infections if there
is allergy to alternative.
Also used synergistically for
gram-positive bloodstream
infections
Macrolides
Azithromycin
Clarithromycin
Erythromycin
Binds reversibly
to 50S subunit
of bacterial
ribosome
preventing
peptide
elongation
static Strep, legionella,
mycoplasma,
chlamydophilia
Oral anaerobes
GI upset
Hepatotoxicity
Pruritis
Hypersensitivity
Prolonged QT
Community acquired/atypical
pneumonia
Tetracyclines
Doxycycline
Bind to 30S
subunit to
prevent peptid
elongation
static MRSA > MSSA,
atypical
pneumona
organisms
GI upset, CNS Toxicity, Dental
Discoloration Photosensitivity
Atypical pneumonia, COPD
exacerbation, sensitive
community acquired MRSA
skin/soft tissue infections,
suppressive therapy for chronic
osteo.
TMP-SMX TMP- inhibits
folic acid
biosynthetic
pathway
through
dihydrofolate
reductase
SMX- inhibits
folic acid
nucleic acid
synthesis
cidal MRSA>MSSA,
50% of GNRs that
cause UTIs are
now resistant to
tmp-smx. DOC
for PCP
Most frequent: GI upset, allergy/rash,
photosensitivity, headache/dizziness
Less frequent: AIN, hepatitis,
Stevens-Johnson/TEN, a wide variety
of hematologic disorders
Skin and soft tissue infections
with MRSA (95 % of MRSA in
community are sensitive), but
does not cover strep well, which
is most common cause of
cellulitis.
Rifampin Inhibits RNA
polymerase
Both MSSA, MRSA,
never used as
monotherapy
GI upset, discoloration of bodily
fluids, allergy/rash,flu like symptoms,
hepatotoxicity,
Use in RIPE therapy for TB; its
use as an adjunt to treat MRSA
infections is controversial
Metronidazole Disrupts DNA
synthesis?
cidal Anaerobes
amoebas
Nausea/Vomiting, xerostomia, CNS
effects, hypersensitivity, disulfiram-
like effect if taken with EtOH
C. difficile colitis, Empiric
therapy in which anaerobes are
suspected, often used in
combination with cipro or CFT
for empiric coverage of enteric
infections.
Clindamycin Binds to 50S
subunit, similar
to macrolides
static Oral anaerobes,
strep, MSSA,
MRSA
Abdominal pain, nausea/vomiting,
esophagitis
C. dif colitis, hypersensitivity
Aspiration pneumonia, oral
infections, infections with
sensitive community acquired
MRSA.
Antibiograms:
- UCSD: EPIC -> Web References -> InfectionControl/Epidemiology.
- VAMC: CPRS -> Services -> Infection Control -> Antibiograms.
74
GASTROENTEROLOGY
CALLING A CONSULT
1. When you call, have a question. For example, instead of calling the Fellow and saying I
have a patient with pancreatitis you might say I have a patient with pancreatitis and Im not
sure if he/she needs an ERCP
2. Call the consult after you examine the patient yourself (including rectal exam). In regards
to the rectal exam, stool color is most important black, red or brown.
3. Understand the limitations of endoscopy. In the GI bleeder, resuscitation is most important
and endoscopy will be delayed until the pt. is stable enough to undergo endoscopy. Also, if
you have a very unstable bleeder, consider calling GI and Surgery/IR to evaluate them.
4. Anemia. You wont likely get GI to scope an inpatient for anemia if there is no evidence of
blood loss (melena, hematochezia or hematemesis).
5. PEGs. GI wont do a PEG on an emergent basis. Also, consider previous abdominal surgery
when considering a PEG (contraindication).
ACUTE PANCREATITIS
1. Treatment Goals: Rest the pancreas/gut and provide supportive care
2. General Considerations:
- Ransons criteria: although these have largely fallen out of favor, they can still help identify
patients with high risk pancreatitis.
- Amylase vs. Lipase: Around 25% of patients with acute pancreatitis have a normal amylase.
Lipase is more sensitive and specific than amylase.
- Early Nutrition: Has been demonstrated to improve outcomes. Earyl oral feeding is a
reasonable approach in cases of mild-to-moderate pancreatitis. Whereas PPN/TPN may be
necessary until jejunal feedings are initiated in severe cases.
3. Etiologies:
- Alcohol and gallstones (most common); Trauma - blunt and iatrogenic (2%-5% after ERCP).
- Others: hypertriglyceridemia (when greater than 1000 mg/dl); hypercalcemia; medications
such as pentamidine, antiretrovirals, thiazides, diuretics, and sulfa antibiotics; pancreatic
divisum; infections such as mumps, CMV, HIV, and E. coli; and the dreaded scorpion sting.
4. Signs: Quiet, distended and painful abdomen with rebound and guarding.
- Cullens sign: periumbilical discoloration
- Turners sign: flank ecchymoses, from retroperitoneal tracking of blood
5. Therapeutics:
- NG tube decompression if significant nausea and vomiting
- NPO while in pain. If your patient is critically ill for an extended period of time, consider
feeding distal to the ligament of Treitz (jejunum) youll need to call IR to place this tube.
- Aggressive IVF; approximately 250 to 300 cc of intravenous fluids per hour are required for
48 hours if the cardiac status permits. One of the biggest mistakes in pancreatitis is
underhydration. Follow K
+
, Mg
++
, and replete as necessary.
- Pain Managment: Morphine is commonly used for analgesia, although in theory it can cause
spasm of the sphincter of Oddi. Consider a PCA. Meperidine may cause seizures and
hypotension in large amounts, although it is the 'textbook' favorite. Also, if you believe that
cause of the pancreatitis is ETOH, remember withdrawal and consider librium taper / PRN
ativan. Watch the patients mental status and vital signs. Take care not to overmedicate,
because it is easy to send people over the edge with benzos.
- Trending Laboratories: QD: CBC, lytes, BUN, creatinine, glucose, calcium, LDH, LFTs. It is
not necessary to follow pancreatitic enzymes.
- Further Studies: ABG and CXR if any evidence of respiratory compromise. Abdominal CTs
should include IV and oral contrast with a rapid bolus or dynamic or pancreatic protocol
(with thin cuts through the pancreas) to delineate necrosis from edema. Non-necrotizing
disease generally has an excellent prognosis, whereas necrotizing pancreatitis markedly
increases the risk of complications (see below).
75
6. Complications:
- In general, with clinical deterioration do the following: pan culture, CT the abdomen
(pancreatic protocol), and cover for bowel flora. Use of empiric antibiotics should be limited to
patients with severe, necrotizing disease, and even in this setting, is still somewhat
controversial. Imipenem has the best pancreatic penetration and tissue levels. Look for
pancreatic necrosis (that may be sterile or infected) and abscesses. To rule out infected
necrosis, especially with continued fevers, consider a FNA for organisms on gram stain. If
FNA is positive, this represents infected necrosis and is generally managed with surgical
debridement/necrosectomy.
- Pulmonary: atelectasis, effusion, ARDS.
- Renal failure: from severe intravascular volume depletion.
- Sepsis/multisystem organ failure.
7. Ranson's Criteria: Severe acute pancreatitis if > 3 Ranson criteria or if any of the following:
shock, renal insufficiency, or respiratory distress.
- At Admission: Glucose >200, AST >250, LDH >350, Age >55yo, WBC >16000
- At 48h: Ca <8, Hct drop >10%, PaO2 <60mm, BUN rise >5, Base deficit >4, IVF >6L
Reference: Whitecomb D. NEJM 2006; 354: 2142.
Admission 48 hrs
Nongallstone pancreatitis
Age > 55 Decrease in Hct > 10%
WBC > 16K Increase in BUN > 5 mg/dl
Glucose > 200 mg/dl Ca
++
< 8 mg/dl
LDH > 350 U/L PO
2
< 60 mmHg
AST > 250 u/L Base deficit > 4 mM
Fluid deficit > 6 L
Gallstone pancreatitis
Age > 70 Decrease in Hct > 10%
WBC > 18K Increase in BUN > 2 mg/dl
Glucose > 220 mg/dl Ca
++
< 8 mg/dl
LDH >400 U/L Base deficit > 5 mM
AST >250 u/L Fluid deficit > 4 L
Risk factors Mortality
12 < 1%
34 15%
67 100%
ACUTE GI BLEED
1. Treatment Goals: Stabilize a bleeding patient and determine the site of the bleed as best you
can.
2. Etiologies: The H&P is key. Remember melena is an UGIB until proven otherwise and brisk
upper GI bleeding can cause BRBPR/hypotension. Conversely a slow, right-sided colonic
bleed can supposedly cause melena. However, the presumed location of bleed will help guide
appropriate studies: EGD, colonoscopy, sigmoidoscopy, angiography, radionuclide scan, etc.
3. Dont forget to ask about prior bleeds, prior abdominal surgery, trauma, ETOH use, liver
disease, coagulopathy, anticoagulant meds, NSAID use.
76
4. Predisposing Risk Factors for GI Bleed:
Risk factors Symptoms/signs
Upper GI
Bleed
NSAID use
Previous ulcer disease
GERD/esophagitis
Retching/emesis
Known liver disease
EtOH abuse
Epigastric pain
Hematemasis
Melena
Lower GI
Bleed
Diverticulosis
Angiodysplasia
Colon cancer
Bowel ischemia
IBD
Hematochezia
(BRBPR)
*Clinical judgment always comes first. Use this to help determine need for ICU care, and need for GI
to see the patient.
5. Patient Risk Stratification prior to Endoscopy:
Low risk Moderate risk High risk
Age <60 Age >60 Age > 60
Initial SBP >100; vital signs
now normal
SBP <100 on admission and/or
mild ongoing tachycardia
Current SBP <100 and/or
severe ongoing tachycardia
Transfusion <2 units Transfusion >2 units Transfusion >5 units
No active major co-morbid
disease*
Stable major co-morbid
disease*
Unstable major co-morbid
disease*
No liver disease Liver disease without
coagulopathy or
encephalopathy
Decompensated liver disease
No moderate or high-risk
clinical features
No high-risk clinical features
* Major comorbid disease defined as CAD, CHF, acute renal failure, sepsis, disseminated
malignancy, altered mental status, pneumonia, COPD, asthma
6. Clinical Evaluation and Stabilization:
Low risk Moderate risk High risk
Vital signs q30 min
Single IV okay
T&C 2 units PRBC
Continuous ECG
Vital signs q15 min with BP
and O
2
sat
Two IVs with isotonic fluid
T&C 2-4 PRBC
Continuous ECG
Vital signs q15 min with BP and
O
2
sat
Two IVs with isotonic fluid
T&C 2-4 units PRBC
Foley catheter
Transfuse if no response to
crystalloid,
Consider airway protection
and surgery consult
*Remember that most deaths occur from respiratory, cardiovascular, and renal complications
associated with bleeding, not exsanguination.
7. Endoscopic risk stratification:
Low risk Moderate risk High risk
Gastritis
Esophagitis
Malloy-Weiss tear
Clean-based ulcer
Ulcer with clot
Dieulafoys ulcer
Varices
Actively bleeding ulcer
Ulcer with visible vessel
77
8. Labs/Studies: Check serial CBC (q4-6h at least until it is clearly stable), lytes, coags, LFTs,
and a CXR. Get a KUB for abdominal distention, pain, or peritoneal signs; get an ECG for
history of coronary artery disease or age >45. In an upper GIB, you will see high BUN/Cr ratio
2/2 blood absorbed in small intestine. NGT can help identify if bleeding is upper. Using a stiff
cold NGT with lidocaine jelly is kinder to the patient, and be sure to keep the neck against the
chest. Placing an NGT in a patient with suspected variceal bleeding is probably not
recommended, although somewhat controversial. If no blood is visible and content is clear,
bleeding is not likely to be from stomach (duodenal bleeding cannot be assessed). If content is
bilious, there is likely no active duodenal bleed but the visual assessment of the presence of
bile is inaccurate. No role for guiaic of stomach contents.
9. Therapeutics:
- Volume resuscitate with IV NS to correct vital signs. Patient should have two large-bore IVs,
and be typed and crossed.
- Transfusion of PRBCs to maintain hematocrit of >25% (>30% if CAD) and platelets >50K.
- FFP and vitamin K should be considered if INR >1.5, DDAVP (0.3 mcg/kg IV q12h x 2) if
uremic bleeding.
- Octreotide drip for patients with acute bleeding and evidence of advanced liver disease or
portal HTN. Dosing: 50 mcg bolus then 50 mcg/hr x 72h.
- Ceftriaxone (1gm IV q24h) for SBP prophylaxis in patients with cirrhosis and ascites
- NPO until after endoscopy
- IV PPI drip in patients with presumed nonvariceal UGI bleeding. Avoid NSAIDs, ASA,
anticoagulants.
- Propranolol or Nadolol should be initiated in patients with variceal bleeds from portal
hypertension after the acute bleed has resolved. Titrate to a dose that lowers the baseline
heart rate 25%.
- If pt. is unstable, call the ICU, transfuse, call GI and surgery. Can consider tagged RBC scan
or IR eval for coiling (if active blood loss continues without an identified source).
10. Post-endoscopy treatment:
- Ask GI when it is okay for the patient to eat. They are guided by endoscopic risk factors, the
matter simply being that GI wants an empty stomach for a re-scope, if needed.
- Generally if patients are admitted they are often observed for re-bleeding after endoscopy.
Length of time depends on endoscopic and clinical risk criteria. Rapid post-endoscopy
discharge is reasonable for some patients with low-risk endoscopic findings; others, with
high-risk stigmata, will generally be observed for up to 72 hours.
11. Lower GI bleeding: follow the same guidelines above for ER evaluation and stabilization as
well as resuscitation. For brisk lower GI bleeds, consult GI and consider proceeding to tagged
RBC scan (nuclear medicine) which can detect bleeding at a rate of 0.1 - 0.5 cc/minute. Once
bleed is localized, consult interventional radiology for angiography and embolization/coiling to
achieve hemostasis. Often, the bleed has stopped by the time the patient has finished the
bowel prep and GI recommends a push enteroscopy or pill endoscopy. Some studies have
shown that the pill is better at picking up occult small intestinal bleeding.
References: Baradarian, R, et al. Am J Gastroenterol 2004; 99: 619. Barkun, A, et al. Ann Intern Med 2003; 139: 843.Fernandez, J, et al.
Gastroenterology 2006; 131: 1049.Leontiadis, GI, et al. BMJ 2005; 330: 568.
78
CLOSTRIDIUM DIFFICILE COLITIS
1. Treatment Goals: Treating diarrhea.
2. Etiologies: The focus is usually on recent antibiotic use, but the biggest risk is being exposed
to C. diff spores, which usually means proximity to another infected patient or MD/RN vectors.
PPI and antibiotics are also linked to C. diff. IDSA guidelines discourage performing repeat
testing, however, sensitivity of current tests is not perfect and some still advocate for sending 3
samples. Think about C. diff in hospitalized patients with new nausea, vomiting, leukocytosis
and (obviously) diarrhea.
3. Definitions: To help guide treatment
- Mild/Moderate Infection: WBC <15K AND Cr < 1.5x baseline
- Severe Infection: WBC > 15K OR Cr > 1.5x baseline
- Severe w/ Complications: WBC > 15K OR Cr > 1.5x baseline with hypotension/shock,
ileus, megacolon
4. Therapeutics: First stop the offending antibiotic.
- Mild to moderate infection: Metronidazole 500mg PO tid x 10-14d with 1st and 2nd episode.
- Severe infection: Vancomycin 125mg QID x10-14d for the 1
st
and 2
nd
episode.
- Severe Infection w/ Complications: Vancomycin 500mg PO QID AND Metronidazole
500mg IV q8h. If NPO, vanco can be given in a rectal solution, but it is in a large volume (1-
2Liters) and if the patient is awake, is miserable for all involved.
- 3
rd
episodes: Vancomycin 250-500mg PO qid x 10d, then 250mg po daily x 14-21d or
Vancomycin 125mg po qid + Rifampin 600mg po bid x 7-10d
- Cholestyramine: can be used to bind the toxin but cannot be given at the same time as
antibiotics (i.e. 2 hours after).
- Probiotics: Once therapy is completed can try adding probiotics, but remember the antibiotics
will kill the probiotics too.
5. Lactobacillus for Antibiotic-Associated Diarrhea Prophylaxis:
Lactobacillus has been studied in several trials for its theoretical efficacy in reducing the risk of
antibiotic-associated diarrhea by maintaining the gut microflora. The data appear to be
conflicting, with several studies (including a double-blinded randomized controlled trial)
showing benefit but at least one randomized, controlled double blinded study by the Mayo
Clinic showing none in a group of patients with previous episodes of pseudomembranous
colitis. However, Lactobacillus is by and large without adverse effects other than cost. Patient
characteristics and flaws varied from study to study, and clear guidelines are few. Decisions
on whether or not to administer lactobacillus as diarrheal prophylaxis may be made on a case-
by-case basis. If given, the VA pharmacy suggested dose is two capsules orally three times
daily with meals.
References: Surawicz CM. Best Pract Res Clin Gastroenterol 2003;17:775-83. Schroeder MS. Am Fam Physician. 2005 Mar 1;71(5):921-8. Infection
Control and Hospital Epidemiology 2010; 31:431-455
GERD AND PEPTIC ULCER DISEASE
1. GERD:
- In patients with moderate to severe GERD who are NPO, it is reasonable to use IV
lansoprazole 30 mg daily until such time as the patient can have oral or enteral medication.
- Oral lansoprazole at 30 mg daily should be given to patients who are taking a PPI chronically
for moderate to severe GERD.
- For inpatients with mild to moderate GERD, oral H2 blockers (or IV H2 blockers in patients
who are NPO) should be the first line agents instead of lansoprazole.
2. Peptic Ulcer Disease:
- For inpatients that are taking a PPI for the chronic treatment of peptic ulcer disease, oral
lansoprazole 30 mg daily should be given. In the case where patients are NPO, IV
pantoprazole 40mg daily should be given until such time as the patient can have oral or
enteral medication.
79
3. H. Pylori Treatment:
- First Line:
P PPI bid
A Amoxicillin 1g bid
C Clarithromycin 500mg bid
- First Line, PCN-allergic:
P PPI bid
M Metronidazole 500mg bid
C Clarithromycin 500mg bid
- Salvage Quadruple Therapy:
P PPI bid
B Bismuth subsalicyclate 525mg qid
M Metronidazole 250mg qid
T Tetracycline 500mg qid
- Eradication: Generally should be checked if there was a significant complication from the
ulcer (GI bleed, stricture, perforation, etc) or if patient is high risk. If you want to prove
eradication after treatment, dont recheck serology (which will stay positive) or repeat
endoscopy (which has low sensitivity). You need to either order an H. pylori breath test (an
outpatient procedure) or an H. pylori stool antigen.
Note: All regimens should be continued for 10-14d to achieve 80-95% efficacy, Metronidazole
resistance >> clarithromycin >> amoxicillin.
ULCER PROPHYLAXIS
- The development of stress ulcers is due to local tissue hypoperfusion in the gastric lining, which is
usually present only in critically ill patients. Despite this, the popularity of PPIs has led to substantial
overuse of this class of medications for the purpose of GI prophylaxis.
- New stress ulcer prophylaxis guidelines have been developed by a multidisciplinary team at UCSD
(see below). One should note that the vast majority of the patients who would benefit from
prophylaxis are in the intensive care unit.
1. Risk Factors for the Development of Stress Ulcers: In a review by Cook et al, over 2800
critically ill inpatients were studied to determine risk factors for stress ulcer bleeding.
- Only two risk factors were found to be significant:
- mechanical ventilation for >48 hours (odds ratio 15.6)
- coagulopathy not due to warfarin or heparin (odds ratio 4.3)
- one or both of these risk factors had a bleeding rate of 3.7% vs 0.1%.
2. Therapeutics:
- IV H2 blockers (ranitidine 50mg iv q8h)
- enteral PPIs (lansoprazole 30mg PO/NGT daily)
- IV PPIs (pantoprazole 40mg iv daily) reserved for patients whom are NPO
3. Patients requiring stress ulcer prophylaxis:
1. ICU patients with at least one of the following risk factors:
- Mechanical ventilation > 48 hours
- Coagulopathy not due to anticoagulant medication (INR > 1.5, PTT > 2x normal,
platelet count < 50K)
- Glasgow Coma Score < 10
- Burn injury with >35% body surface area involved
- Acute spinal cord injury
- Transplant patients in the perioperative period
- Multiple trauma (Injury Severity Score > 15)
- Hepatic failure
2. History of PUD/UGIB plus at least two of the following risk factors:
- Sepsis
- ICU stay > 7 days
- Occult bleeding lasting > 6 days
- High-dose corticosteroids (>250 mg hydrocortisone/day or the equivalent)
80
4. Discontinuation of stress ulcer prophylaxis:
- Stress ulcer prophylaxis may be discontinued once risk factors have resolved.
- Stress ulcer prophylaxis is rarely warranted in adult patients in non-ICU settings, with the
exception of those patients who meet the criteria listed above.
References: Geerts et. Al, Chest 2001; 119:132-175. Ching, Drug Therapy Topics 2004, 33(1):1-8. Anderson et. al, Ann Intern Med 1991; 115:591-595.
Alikhan et. al, Arch Intern Med 2004; 164:963-968 Cade, Crit Care Med 1982; 10(7):448-450. Gardlund et. al, Lancet 1996; 347:1357-1361.Gallus et. al,
NEJM 1973;288(11)545-551. Belch et. al, Scott Med J 1981;26:115-117. Samama et. al, NEJM 1999;341(11):793-800. Cook et al, JAMA 1996 275 (4),
308-314. Kantorova et al, Hepatogastroenterology 2004 51(57), 757-761. Cook et al, NEJM 1994 330(6), 377-381
SMALL BOWEL OBSTRUCTION
- Although generally considered a surgical problem, SBO is something we see on the wards. It is
important to be able to recognize SBO and to be able to initiate treatment.
- The surgeons say: Never let the sun rise or set on a small bowel obstruction
1. Risk Factors:
-Crohns disease
-Previous abdominal surgery or radiation
-Intra-abdominal malignancy
-Hernias
2. Clinical features:
-Nausea/vomiting (bilious vomiting)
-Decreased bowel sounds or high pitched tinkling bowel sounds
-Abdominal distention
-Abdominal pain
-Constipation
3. Diagnosis:
-Acute abdominal series Look for air fluid levels, volvulus or free air under the diaphragm
(could indicate perforation)
-Labs:
-CBC: Marked leukocytosis could indicate ischemia
-Chem panel: Electrolyte abnormalities 2/2 dehydration and N/V
-Consider CT scan of the abdomen
-Consider other diagnoses including ileus or pseudo-obstruction
4. Initial management:
-NG tube to intermittent suction
-NPO
-Stat surgical consult
-Correct the hypovolemia/electrolyte disturbances
-Treat the pain and nausea/vomiting
81
HEPATOLOGY
END STAGE LIVER DISEASE & CIRRHOSIS
1. Treatment Goals: These patients often present with numerous complications of their
advanced disease, including AMS, infection, and metabolic derangement. Initially, the goal in
these patients is to control massive ascites, GI bleeding, bacterial pathogens translocating
from the gut and hepatic encephalopathy.
2. Etiologies: The number one cause of end stage liver disease in the US is HCV. ETOH and
HBV follow after HCV, and rare causes of liver disease including AIH, hemochromatosis, PBC,
PSC, and Budd-Chiari syndrome. NAFLD / NASH are emerging causes of chronic liver
disease. Use the Child-Pugh classification (below) to evaluate patients survival. MELD score
was developed to assess patients surgical risk when undergoing TIPS, but is now used in
UNOS organ allotment.
3. Child-Pugh Classification:
Points scored 1 2 3
Encephalopathy grade None 12 (mild confusion/lethargy) 34 (marked
confusion/coma)
Bilirubin (mg/dl)* < 1.5 1.52.3 > 2.3
Ascites None Easily controlled Poorly controlled
Albumin (mg/dl) > 3.5 2.83.5 < 2.8
PT (sec >control) < 4 46 > 6
* Does not apply to primary cholestatic liver disease (e.g. primary biliary cirrhosis)
Total points Classification 1 & 2 year survival
56 Class A 100%, 85%
79 Class B 80%, 60%
1015 Class C 45%, 35%
4. MELD Score:
3.8 Ln (bilirubin mg/dL) + 11.2 Ln (INR) + 9.6 Ln (creatinine mg/dL) + 6.4 (etiology 0 for
Alcohol / Cholestasis)

- Scoring ranges from 6 (least ill) 40 (most ill)
- 4 special cases in which higher MELD score assigned includes HCC,
Hepatopulmonary Syndrome, Familial Amyloidosis, and Primary Oxaluria (pediatric
metabolic disorder)
- Average MELD for pt underoing liver transplantation is 20.
ASCITES
1. Treatment Goals: Usually fluid management and determining if the patient has SBP.
2. Etiologies: Most commonly ESLD with portal HTN, but other causes include cardiac (right
sided heart failure, constrictive pericarditis); noncardiac (Budd-Chiari, cirrhosis, etc.); lymphatic
obstruction (from TB or peritoneal carcinomatosis); pancreatitis; nephrotic syndrome.
3. Paracentesis: Tap the belly if the etiology is not clear, or if you have any suspicion of SBP, as
SBP can often be present without symptoms. Many patients present simply with an
uncomfortable and swollen belly, but best to rule SBP ASAP. Coagulopathies are high in this
patient population and prior to performing any procedure check coagulation studies. Use an
ultrasound to decide if there is enough to tap. The fluid appears dark on ultrasound, and
usually the right or left LQ are popular locations. If you doubt whether the fluid can be tapped
IR can help, but you may miss the diagnosis by waiting. Send a cell count, and cultures to
evaluate for infection. Send albumin to calculate the SAAG and lipase and amylase if you
think pancreatitis is involved.
4. Fluid Analysis: Calculate the SAAG (serum ascites-albumin gradient) by subtracting the
ascites albumin from the serum albumin. A high gradient (> 1.1) means there is portal
hypertension. The total protein concentration of the ascites, a proxy for complement levels,
predicts risk of SBP (< 1 g/dl is high risk). A high total protein (> 2.5 g/dl) with portal
hypertension can be seen in congestive heart failure.
82
High gradient (> 1.1 g/dl) Low gradient (< 1.1 g/dl)
Cirrhosis
Alcoholic hepatitis
Cardiac failure
Massive liver metastases
Fulminant hepatic failure
BuddChiari syndrome
Portal vein thrombosis
Venoocclusive disease
Fatty liver of pregnancy
Myxedema
Mixed ascites
Peritoneal carcinomatosis
Peritoneal TB
Pancreatic ascites
Biliary ascites
Nephrotic syndrome
Serositis
Bowel obstruction/infarction
5. Treatment of Ascites due to Cirrhosis: If the patient is drinking, cessation of ETOH is
helpful. Liver usually starts patients on lasix 40mg and spironolactone 100mg daily, and
adjusts depending on the patients response. Sodium restriction 2 grams/day (without Na
restriction diuretics rarely work). Large volume paraecentesis are reserved for refractory
ascites (meaning unresponsive to diuretics). TIPS is often performed in these patients as well,
but it often results in worsening or new encephalopathy.
References: Moore, K.P., Aithal G.P. Guidelines on the management of ascities in cirrhosis. Gut 2006 55vi1-vi12.
SPONTANEOUS BACTERIAL PERITONITIS
1. Treatment Goals: Treating and preventing peritonitis in cirrhotics.
2. Etiologies: In patients with cirrhosis & ascites SBP is thought to be secondary to translocation
of gut flora. Debatable on whether it goes directly into the peritoneum or blood first.
Classically, SBP presents as a patient with pre-existing ascites who develops abdominal pain,
fever, decreased bowel sounds, worsened hepatic encephalopathy, and hypotension.
However, fever is present only 70% of the time, abdominal pain 60%, and encephalopathy only
50%. One in ten patients with SBP will have no symptoms at all.
3. Fluid Analysis: PMN > 250/mm3: The sensitivity and specificity of this value is above 90%,
but only in cirrhotics (cant use same diagnostic method in patients with peritoneal cancer or
ascites due to cardiac or other causes). Traumatic taps generate 1 WBC per 250 RBCs, and 1
lymph per 750 RBCs. WBC greater than 10,000 is suggestive of secondary peritonitis. PMN
<250 and other signs/symptoms of infection should be treated empirically while awaiting
culture results. A positive Gram stain occurs 5-20% of the time, and is not only diagnostic, but
can quickly rule-in bowel perforation when polymicrobial. A positive culture is diagnostic of
SBP and will narrow therapy. Most common organisms are E. coli, Strep species, and
klebsiella pneumoniae.
4. Therapeutics: Treat with third generation cephalosporin (e.g. ,cefotaxime or ceftriaxone). Give
albumin on day 1 (1.5 g/kg of 25% albumin) and day 3 (1.0 g/kg of 25% albumin). In high-risk
patients (those with total protein in ascites < 1.0, previous SBP, or s/p recent GI bleed),
consider initiating SBP prophylaxis with ciprofloxacin 500mg po daily.
5. SBP Prophylaxis:
- During GI bleed, Ceftriaxone x 5-7 days for prevention of SBP.
- Indications for SBP Prophylaxis: Previous SBP, ascites Total Protein <1.5gm/dL and with at
least one of the following: a) Child-Pugh 9 points and serum bilirubin 3 mg/dL or b) serum
creatinine 1.2 mg/dL or blood urea nitrogen 25 mg/dL or serum sodium 130 mEq/L .
- Above patients should receive daily fluoroquinolone indefinitely (theoretical risk of selection
for resistance with intermittent FQ).
References: Runyon B. Hepatology 2009, 49; 2087. Sort P, et al. NEJM 1999; 341: 403-409. Runyon BA, et al. Gastroenterology 1991; 100:1737-1742.
83
HEPATIC ENCEPHALOPATHY
1. Treatment Goals: Improving mental status in cirrhotics
2. Etiology/Precipitating Causes: Most commonly, it is an exacerbation in a patient who is not
taking lactulose. But can also be worsened by GI bleeding, increased protein intake, infection,
hypovolemia, sedatives/tranquilizers (narcotics also constipate), hypokalemia, and
hypoglycemia.
- Checking Ammonia levels rarely useful but can help to determine if there is a hepatic etiology
for delirium / encephalopathy or to monitor the efficacy of ammonia-lowering treatment (e.g.
rifaximin, lactulose).
3. Grading:
Grade Level of
Consciousness
Intellectual
Function
Neuro Findings
1 Day/night reversal
Lack of awareness
Short attention Incoordination
2 Lethargic &
inappropriate
behavior
Disoriented Asterixis & abnl
reflexes
3 Asleep but arousable Loss of
meaningful
communication
Asterixis & abnl
reflexes
4 Unarousable Comatose Decerebrate
4. Therapeutics: ID and treatment of precipitating event. Lower blood ammonia: Give 20ml of
lactulose q2-4h to tirate to about 4 BM/day. If the pt is NPO for some reason, you can also do
lactulose enemas, but it wont make you friends with the nursing staff. Starting rifaxamin
400mg TID will also help.
References: Ferenci P, et al., 1998. Hepatology. 2002; 35:721. Garcia-Tsao, Wongcharatrawee. Leevy CB, et al.. Am J Gastroenterol 2005; 100
(suppl)t S134. Mas AR, et al. J Hepatol 2003; 38:51-5
ESOPHAGEAL VARICES
1. Treatment Goals: Controlling active variceal hemorrhage
2. Etiology: From portal HTN.
3. Therapeutics:
- Hemodynamic Resuscitation:
- 2 large bore IVs
- Type and Cross
- IVF: goal SBP>70, HR<100 and Hct 27-30; Permissive anemia better; over-
aggressive transfusion leads to increased Portal Pressure leading to rebleeding
- Octreotide: 50mcg bolus IV, then 50 mcg/hour x 2-5 days
- Endoscopic Therapy: Sclerotherapy or Esophageal Variceal Ligation (EVL)
- SBP Prophylaxis: Ceftriaxone x 5-7 days preferred over Fluoroquinolone (IV or PO) BID
4. Options when endoscopy fails (10-20% of patients):
- Repeat Endoscopy
- Balloon Tamponade (initial control of bleeding observed in 30-90% of pts)
- TIPS (early on can provide hemostasis and survival benefit)
- Surgical consultation for surgical shunt placement or other nonshunt options.
5. Primary Prevention of Variceal Hemorrhage:
- Nonselective |cto blockers decrease risk of 1st variceal hemorrhage by 50%: It has been
recently shown that BB were not effective in preventing development of varices
- Prophylactic EVL c/w Bcto blockers decreased risk of 1st bleed, but no difference in
bleeding-related mortality or all cause mortality (2 RCTs comparing EVL and BB are
conflicting). Therefore, data exist only to support EVL for 1 prophylaxis only
in pts intolerant to BB.
84
6. Secondary Prevention of Variceal Hemorrhage:
- Nonselective Bcto blockers
- Block adrenergic dilatory tone in mesenteric arterioles causing unopposed adrenergic
mediated vasoconstriction leading to decreased portal inflow causing decreased portal
pressure
- Decrease risk of rebleeding by 40% (1 study showed survival benefit).
- EVL plus Bcto blockers reduce rebleeding much more than banding alone.
- Start with propranolol 20 mg BID (or nadolol 40 mg QD) and titrate dose to achieve
25% reduction in HR or resting HR of 50-60.
7. TIPS: decompresses portal vn. without risks of general anesthesia. TIPS has been shown to
rebleeding c/w sclerotherapy (but assoc with survival & cost). Considered salvage
therapy for pts who rebleed despite adequate endoscopic and medical therapy.
- Adverse effects: encephalopathy, TIPS stenosis portal pressure.
References: Garcia-Tsao, et. al. Hepatology 2007;46:922-938. Sung JY, et al. The Lancet 1993, 342: 637-641.Bernard B, et al. Hepatology
1999; 29: 1655-1661. Bildoza M, et al.. Scand J Gastroenterol 2000; 35:419.Loannou G, et al. Cochrane Database Syst Rev 2003; CD002147 NEJM
1992; 326: 1527.Chojkier M, et al. Gastroenterology 1996; 111: 138. Laine L, et al.. Ann Intern Med 1995; 123:280.Pascal JP, et al. NEJM 1987;
317:856.
ABNORMAL LFTS
Remember the normal range for a lab test = mean value in a healthy group 2 SD. Therefore 5% of
the results from normal pts fall outside the defined range of normal.
1. Sources of Individual Liver Tests:
- AST: liver, cardiac muscle, skeletal muscle, kidneys, brain, pancreas, lungs
- ALT: mostly liver and is therefore a more specific marker.
- Alk phos: liver, bone, placenta. Levels inc with age (65yo ~150% seen in 30 yo).
- GGT: hepatocytes and biliary epithelium. Sens for detecting ETOH ingestion: 52-94%.
- 5NT: 5 Nucleotidase produced by the liver and specific for hepatobiliary etiology for
elevated alk phos.
CAUSES OF CHRONICALLY ELEVATED AMINOTRANSFERASE LEVELS
Hepatic Causes Nonhepatic Causes
Alcohol Abuse Celiac sprue
Medications Muscle disorders (polymyositis, rhabdo)
Chronic HBV and HCV Strenuous activity
Steatosis and NASH
Autoimmune Hepatitis
Hereditary Hemochromatosis
Wilsons disease
Alpha1- antitrypsin deficiency
HEPATIC CAUSES OF CHRONICALLY ELEVATED ALK PHOS LEVELS
Obstructed bile ducts
Drug-induced Cholestasis
Primary Biliary Cirrhosis
Primary Sclerosing Cholangitis
Infiltrative disease (sarcoid)
Bile Ductopenia of adulthood
2. Helpful Patterns of LFT Abnormalities:
- AST:ALT>2 is >90% sens for ETOH liver
- AST:ALT> 3 is >96% sens for ETOH liver
- GGT 2x normal and AST:ALT > 2 very suggestive of Alcoholic Liver Disease
- AST or ALT>1000 usually seen only in Acute Viral Hepatitis, Ischemic Injury, Fulminant
Failure, or after Chemo-Embolization (TACE)
- In acute biliary obstruction, elevation of alk phos and GGT often lags symptoms or rise in
ALT (~24 hours).
- Elevated Alk Phos and GGT in non-dilated biliary tree suggests cholestatis or hepatic
infiltration (sensing). If GGT normal, suggestive of non-hepatic source (bone or placenta).
85
3. Work up of chronic LFT abnormalities:
- RUQ Ultrasound
- Hepatitis Panel
- Screen for the below conditions based on clinical scenario
LESS COMMON LIVER DISEASE
Most LFT abnormalities and liver disease encountered in our hospitals is secondary to Alcohol, Viral
Hepatitis, and Medications. Below are a few well characterized, but less common liver diseases that
you may encounter.
1. Auto-Immune Hepatitis: SPEP is a useful screening test as >80% of patients with AIH have
hypergammaglobulinemia. A finding of >2x nl polyclonal Igs is most suggestive of AIH. (While
ANA, anti-smooth muscle and anti-liver-kidney microsomal Ab are commonly ordered, these
are all relatively insensitive and routine use is discouraged).
2. Non-alcoholic Fatty Liver Disease NAFLD: Ranges from simple Hepatic steatosis to Non-
Alcoholic SteatoHepatitis (NASH) and Cirrhosis: See AST/ALT <1. US or CT demonstrates
fatty infiltration. The diagnosis of NASH is a histologic one and requires liver biopsy
(demonstrating pericentral fibrosis, inflammation or hepatocyte necrosis).
3. Hereditary Hemochromatosis: Transferrin saturation (serum Iron/TIBC) >45% is highly
suggestive (94% sens and 94% specific). Confirmation should include Ferritin and genetic
testing for HFE gene.
- If C282Y homozygote patient has HH
- If C282Y/H63D heterozygote HH most likely
- If H63D homozygote HH likely, but must r/o other causes of elevated LFTs.
4. Primary Biliary Cirrhosis: Pathognomic finding is anti-mitochondrial antibody positive with a
florid duct lesion on biopsy. Clinically, these patients are women more than 90% of the time
with an insidious onset of their disease usually associated with fatigue and other autoimmune
disease. Ursodiol has been shown to improve transplant free survival, and vitamin D has been
associated with an improvement in fatigue.
5. Primary Sclerosing Cholangitis: The mans version of PBC, usually diagnosed by ERCP or
MRCP. MRCP is preferable for diagnosis to save the extrahepatic ducts from further damage.
Associated with cholangiocarcinomas, ulcerative colitis and ANCA positivity. Walter Payton
died of this.
6. Wilsons Disease: consider in pts <40 yo. Screen with serum cerruloplasmin (decreased in
85% of cases) & ophtho exam looking for Kayser Fleischer rings. If highly suspicious,
measure urine copper excretion (>100mcg per day is suggestive). Copper stain of liver biopsy
is gold standard.
7. Alpha1-Antitrypsin Deficiency is diagnosed by either:
- Direct measurement of alpha1-antitrypsin (careful: its an acute phase reactant).
- Lack of peak in alpha globulin bands on SPEP.
8. Nonhepatic causes of elevated LFTs:
- Celiac Sprue: 10% of cases of asymptomatic elevated LFTs (check anti-endomysial and
antigliadin Ab)
- Polymyositis: check CK and aldolase levels
- Strenuous exercise: check CK and aldolase levels
86
Kings College Criteria for Liver Transplantaion in Fulminant Hepatic Failure
Acetaminophen-Induced Disease: Other Causes of Fulminant Liver Failure:
Arterial pH < 7.3
or
The three following:
1. Grade III/IV Encephalopathy
2. PT > 100 sec
3. Cr > 3.4 mg/dL
PT > 100 sec
or
Any three of:
1. Age 10 or > 40 yrs
2. Non-A, Non-B hepatitis, drug toxicity
3. Jaundiced > 7 days before HE onset
4. PT > 50
5. Serum Bilirubin > 18 mg / dL
References: Pratt DS, Kaplan MM, NEJM 2000; 342: 1266-1271. Tavill AS, Hepatology 2001, 33: 1321-1328.
87
RENAL
CALLING A CONSULT
1. Have a question beyond elevated Cr. Think about what your differential is and how you
would like nephrology to help. For example, instead of saying I have a patient with a Cr of 3.0
you might say I have a cirrhotic patient with AKI who hasnt responded to a volume challenge
and Im concerned about HRS
2. Know the time course of Cr rise and history of urine output.
3. Realize that setting up dialysis takes time. Calling sooner rather than later on AKI patients
who are not turning the corner in terms of urine output, acid base or electrolyte regulation is
never a bad idea. Getting dialysis going requires renal seeing the patient, consent for both
access and dialysis, placing access (which may require reversing coagulopathy or platelet
dysfunction), dialysis prescription, lining up nursing and equipment and finally putting the
patient on.
4. If admitting a chronic dialysis patient call Nephrology. Schedules for the next days
inpatient dialysis are made the night prior so get the basic info and call ASAP. Ask the patient
what schedule they are on (MWF, TThSat), when they were last dialyzed, where they are
usually dialyzed, what their access is (fistula, graft, permacath) and current/dry weights to
asses degree of overload.
5. Renal transplant patients. Call Nephrology for renal related admits and FYI for unrelated (ie
pneumonia). Remember that transplant meds interact with lots of other medications always
talk with pharmacy or the renal fellow if unsure. Ask when immunosuppressant levels should
be drawn, random levels are rarely useful.
ACUTE KIDNEY INJURY
When you see a patient with renal failure of any kind (any elevation in creatinine or anuria),
follow the following steps, in order.
A.) Rule Out Killers
Determine if there is any related problem that will quickly kill the patient. The following disorders
are potentially lethal and often times require hemodialysis. Remember them with AEIOU:
Acidosis - Renal failure oftentimes causes a nongap acidosis in its early stage but with accumulation
of organic ions a gap acidosis later predominates.
Obtain ABG to determine degree of acidosis/compensation and calculate gap as described below to
rule out other acid-base disturbances that may be present.
Electolytes- specifically potassium- if >6.0, obtain EKG, treat as below. If patient does not respond to
treatment then should prepare for dialysis.
Intoxication- Lithium is classic example of toxic med that is dialyzable
Overload (of fluid)-
If respiratory rate is >30, O2 sat <90%, then goal is to remove fluid fast.
Try furosemide (patients with renal failure will only respond to high doses- try 80mg IV),
if not responding then patient will require urgent hemodialysis. Note If patient Is anuric
this is less likely to work.
Uremia- Uremic syndrome includes confusion, pericarditis, and cardiomyopathy and should be
treated with urgent dialysis.
For uremic bleeding expected or ongoing- Trial DDAVP followed by estrogens and
cryoprecipitate. If not reversing, may need dialysis.
B) Workup:
1. History and Physical exam: Attention to urine output/color, recent procedures and medications,
vital signs, volume status, signs of obstruction/vascular disease/systemic disease. If urine
output has slowed/stopped, what was happening when it stopped? (new medications-
especially NSAIDS, hypovolemia, fever, etc)
2. Check the creatinine.
- What is it today? What was it yesterday? A few months ago?
- The trend in the creatinine is the ONLY true way to know if the failure is acute or chronic.
3. Urine evaluation: Output, urinalysis, sediment, electrolytes (Na, Cr, UN), osmolality
88
4. Fractional Excretion of sodium/urea: (UNa/PNa)/(Ucr/PCr): If FeNa is <1% then likely prerenal,
contrast-induced, or glomerulonephritis. If >2% then ATN
- In setting of diuretics, then check FeUN: <35% is prerenal.
5. Additional workup: If indicated renal ultrasound, serologies, urine eos.
C) Treat the patient: In order to treat the patient, you should try to determine the diagnosis. Much of
this can be determined by history and physical.
If it is determined that the renal failure is acute, the next question to ask what kind of renal failure is it.
A simple algorithm is to determine if the renal failure is pre-renal, intra-renal, or post-renal. This
allows you to form a more narrow differential and focus therapy better.
1. Is this Pre-renal?
- This is the second most common cause of acute renal failure and generally an easy one to fix,
so it is good to think about it first.
- Prerenal is caused by:
1) Decreased intravascular volume (dehydration, blood loss, sepsis) - patients look dehydrated
2) Renal Vasoconstriction (NSAIDs, ACE/ARB, hepatorenal syndrome, CHF) - patients may
look euvolemic or even overloaded
Diagnosis:
- Check history and physical: Is the patient dehydrated? Low blood pressure? Elevated heart
rate? Did they have a reason to lose volume, ie vomiting, diarrhea or blood loss? Did they
take NSAIDs or ACE? Do they have cirrhosis or CHF?
- Check course of creatinine rise: It should have been over only a few days (a few weeks at
most) if this is pre-renal.
- Check urine: Patients should have a very concentrated urine (look at the specific gravity-
concentrated is >1.020. Pre-renal patients should be making only small amounts of urine or no
urine. Urine osmolality frequently >450 mosm/kg and urine sodium <25 meq/l
- Compare the urine creatinine to the serum creatinine. This ratio can help determine the type
of acute renal failure
If it is >40, that is consistent with pre-renal, if it is less than 20 then it is more consistent with
tubular necrosis
- If not volume overloaded, provide patient with fluid challenge. Give a few liters of fluid over one
day, and then recheck the creatinine and urine output the next day. If there is improvement
this is most likely prerenal.
- Consider renal ultrasound if diagnosis of renal failure unclear.
- Note if the patient appears volume overloaded and urine studies are consistent with prerenal
azotemia, then the effective arterial blood volume is low such as in CHF or Hepatorenal
Syndrome (HRS discussed in separate section).
Treatment:
- If fluid depleted, hydrate with normal saline and then recheck creatinine and urine output over
the next day. If they are improving, continue hydration until creatinine normalizes or reaches a
nadir.
- If blood loss caused the renal insufficiency, then give blood.
- If due to CHF, treatment involves improving cardiac output
- If due to hepatorenal syndrome then treatment is with midodrine/ocreotide/albumin (MOA)
cocktail.
- If sepsis (can be pre-renal early in the course), use fluid and antibiotics
- Treat underlying problems (ie those causing the vomiting or diarrhea)
- Stop offending medications like NSAIDs and ACE-I.
- Renally dose all medications
89
2. Is this post-renal?
Diagnosis:
- Requires either bilateral obstruction (or unilateral obstruction with a single functioning kidney)
and is most commonly due to prostatic disease (hyperplasia or cancer) or metastatic cancer.
Can be acute or chronic.
- Possible causes of acute urinary retention to consider:
BPH (53%), constipation, prostate cancer, postoperative, urethral stricture, neurologic
disorder, medications/ drugs (anticholinergics/sympathomimetics), UTI, urolithiasis, bladder
neoplasm, spinal cord injury/compression, phimosis, pelvic masses, malpositioned foley
catheter, acute prostatitis.
- Important to diagnose early since most causes of obstructive uropathy are reversible but can
lead to irreversible kidney damage if treatment is delayed
- Feel suprapubic area for distended bladder, DRE for enlarged prostate. Perform neurologic
exam.
- Realize that many patients with urinary obstruction can still make normal urine volumes!
Anuria 2/2 obstruction is rare and only occurs with acute obstruction.
- If suspicious, check post void residual (have patient urinate, then place foley or obtain u/s
immediately after urinating). If >200ml remaining in the bladder after urination, this indicates
obstruction.
- Order renal ultrasound to look for hydronephrosis, (acute obstruction or obstruction in volume
depleted patients may not show hydronephrosis)
Treatment:
- Place foley: serves 2 purposes
o Relieves possible bladder neck obstructions
o Helps with UOP measurement
o If decompressing high post void residual clamp foley ever 500cc for an hr or two to
reduce risk of hemorrhagic cystitis from decompressing too quickly
o Also be aware of post-obstructive diuresis- if patient is urinating large volumes (>5L
day), they will need fluid replacement to prevent dehydration. Aim to replace about
80% of what he is urinating each day so that the urine volume will eventually
decrease
- Treat underlying cause of obstruction (ie BPH)
3. Is this intrinsic renal disease?
Diagnosis:
- First rule out pre-renal and post-renal etiologies as above.
- Causes of intrinsic renal problems to consider:
Vascular:
o Acute
o Malignant Hypertension
o HUS/TTP
o Vasculitis (Wegners)
o Scleroderma
o Cholesterol emboli (can be acute or chronic, but usually occurs post
procedure)
o Chronic
o Unilateral or Bilateral renal artery stenosis
o Nephrosclerosis
Glomerular
o Nephritic: UA with RBCs, WBCs, casts (of rbc's or wbc's), variable proteinuria
o Nephrotic: Urine protein >3g/ 24h, few RBCs, WBCs The urine protein can be
measured for 24 hours or in a single sample of urine (spot protein/creatinine ratio):
o Urine protein in 24 hrs ~ urine protein in single sample / urine creatinine in
single sample x 8.8 for women OR x 13.2 for men)
o There are many different diseases that cause glomerular disease; they usually do not
cause renal failure but still can. There will be other times where a patient with an
underlying glomerular disease presents with acute renal failure from another cause
and the urine studies will reflect both problems.
90
Tubular
o Acute
ATN: UCr/SerumCr <20; FeNa>2%; muddy brown casts on UA
(many underlying causes: sepsis that persists and the kidneys become ischemic,
rhabdomyolysis, meds,
contrast, progression of pre-renal)
Note: Contrast Induced Nephropathy= 25% increase in serum creatinine
after exposure to contrast; usually transient: Creatinine peaks at 3 days
and returns to normal by day 10. Risk Factors: DM, Cr> 150, HTN,
NSAIDS, proteinuria, periproceduere volume depletion, heart failure,
cirrhosis
AIN (drug induced)
Cast Nephropathy (multiple myeloma)
Acute Phosphate Nephropathy (after bowel prep)
Tumor Lysis Syndrome (with chemotherapy)
o Chronic
Polycystic
Hypercalcemia
Autoimmune (Sarcoid, Sjogren's)
Infectious (note, these can be glomurular, tubular or vascular): HIV, Malaria, Schistosomiasis,
Bacterial , TB
Treatment: Two parts:
0. Follow Guidelines for General Treatment (see below)
1. Treat underlying disorder.
5. General Treatment of all Kidney Failure (prerenal, postrenal, intrinsic)
- Check all meds. Stop those that are nephrotoxic. Adjust doses as necessary using GFR
(remember that equations for GFR often underestimate true GFR in AKI). Common meds to
hold or dose adjust include ace -i, metformin, glimepizide, clonidine, atenolol, digoxin,
allopurinol, colchicine, metoclopromide, antibiotics, lamivudine, pyrazinamide, ethambutol
- Avoid contrast
- Monitor blood pressure and treat as needed (goal <130/80)
- Monitor urine output and volume intake (IV AND ORAL) closely. Treat volume overload if
necessary. Kidney failure requires the use of HIGH doses of furosemide (around 80mg IV or
more) in cases of volume overload. Follow daily weights.
- Monitor electrolytes daily at minimum (K, Phos, Ca). Phos Binders as needed.
- Monitor creatinine daily at minimum
- Monitor for acidosis. May need sodium citrate or sodium bicarbonate.
- Encourage good nutrition. Avoid high potassium and high protein foods.
6. Consider a renal biopsy or dialysis
Consider this if:
Your patient is not improving with treatment outlined above.
Your patient is quickly getting worse (rising creatinine, decreasing urine output, or an AEIOU)
HEPATORENAL SYNDROME
- Development of acute renal failure due to cirrhotic liver disease and portal hypertension
through the generation of nitric oxide with resultant splanchnic vasodilation and steal from
effective arterial volume. Also component of renin-angiotensin mediated systemic
vasoconstriction. Diuretics can cause azotemia but they do not cause HRS.
- Two types of hepatorenal syndrome: Type I HRS is the most severe and is defined as at least
a 50% decline in creatinine clearance over 2 week period; Type II HRS is more insidious and is
often described as ascites resistant to diuretics.
- HRS Type I most commonly precipitated by GI bleeding, infection including SBP, large volume
paracentesis without albumin supplementation.
- Criteria for diagnosis include
o Presence of cirrhosis with ascites
o Serum creatinine > 1.5
91
o No improvement in creatinine after at least two days with diuretic withdrawal and
volume expansion with albumin. Recommended challenge dose is 1 g/kg of body
weight per day up to a maximum of 100 g/day.
o Absence of shock, recent treatment with nephrotoxic drugs.
o Absence of parenchymal kidney disease as indicated by proteinuria >500 mg/day,
microhaematuria (>50 red blood cells per high power field) and/or abnormal renal
ultrasonography.
- Treatment
o Various small studies have shown benefit with combined therapy of
vasopressor/albumin in which roughly 2/3 of patients show improvement in serum
creatinine.
o Current recommendations are as follows:
1. Vasopressor: Include midodrine in combination with octreotide or
norepinephrine
Most commonly use midodrine 7.5 mg tid (increase to 12.5 tid if
needed) in combination with octreotide 100 ug SQ tid (increased to 200
ug tid if needed)
2. Albumin 1 gram/kg on day one followed by 20-40 grams daily
3. Duration of therapy 5-15 days. End point: reduction of serum creatinine
concentration<1.5
o Other treatment modalities include TIPS (although not much data on this) and liver
transplantation.Genes Pere and Shrier. Renal Failure in Cirrhosis: Medical Progress. NEJM 2009;361:1279-90; Genes
Pere MD, Cardenas A, Vincente A, Rodes J. Management of Cirrhosis and Ascites Review. NEJM 2004;350:1646-54.
- Prophylaxis: It is worth noting that up to 30 percent of patients with SBP develop HRS.
Studies have shown that in addition to antibiotics, cotreating them with IV albumin at 1.5 gm/kg
at diagnosis and 1 gram/kg 48 hours later helps prevent HRS and improve probability of
survival. In one study of 126 patients with cirrhosis and SBP who were randomized to
cefotaxime verusus cefotaxime + albumin, the development of renal dysfunction while in house
was reported in 33% of patients in the cefotaxime only group versus 10% in the cefotaxime +
albumin group. Survival at three months was 22% in the cefotaxime group versus 41% in the
cefotaxime + albumin group respectively. Sort P. Navasa et al. Effect of intravenous albumin on renal impairment and
mortality in patients with cirrhosis and SBP. NEJM. 1999; 341: 403-409.
CHRONIC KIDNEY DISEASE
- >3 months of reduced GFR (<60 ml/min).
- Multiple etiologies but diabetes and hypertension the most common
- Five stages of chronic kidney disease based on GFR
o Stages I-III focused on slowing progression and treating complications: controlling
hypertension (treat with ACE/ARB) with goal <130/80, strict diabetic control, modify
cardiac risk factors, avoid contrast administration, monitor/treat anemia (goal Hgb 11-
12), secondary hyperparathyroidism, acidosis, hyperposphatemia/hypocalcemia.
Dietary restriction: Na/K, moderate protein restriction. Smoking cessation
o Stage IV (GFR15-29): as above, prepare for hemodialysis.
o Stage V (GFR<15): as above; dialysis. Avoid contrast as many patients are not
anuric and do have residual kidney function that can be adversely impacted. Patients
should fluid restrict and monitor their dry weight
- Hemodialysis patients can be dialyzed through AV fistula, AV graft, tunneled catheter or
temporary hemodialysis catheter. Patients requiring dialysis need functioning access at all
times.
- AV fistulas are the preferred method of access for long term patency and lower rate of
complications.
Levy, A. Nondiabetic Kidney Disease: Review. NEJM. Vol. 347, No. 19 November 7, 2002
K/DOQI Clinical Practice Guidelines and Clinical Practice Recommendations 2006 Updates Hemodialysis adequacy Peritoneal Dialysis
Adequacy Vascular Access. Am J Kidney Dis 2006; 48(Suppl 1):S1.
92
CONTINUOUS RENAL REPLACEMENT THERAPY
- Indicated in patients with AKI and ESRD in need of hemodialysis who are unable to tolerate
fluid shift of intermittent hemodialyisis
- Requires hemodialysis chatheter (temporary/tunneled)
- Blood is filtered through hemodialysis membrane and filtrate is discarded.
- Based on goal volume status (positive/even/negative) a replacement solution is infused at a
rate (greater/equal/or less than) the filtration rate of blood which is adjustable.
- Citrate is infused prior to the hemodialysis filter as an anticoagulant (binds calcium which is
pro-coagulant) and is monitored by measuring post-filter ionized calcium which is maintained
at an appropriate level via a calcium infusion.
- Typically nephrology team controls the infusate/dialysate/citrate infusion rate directly with the
nursing staff. However, goal volume status is to be determined by both ICU/nephrology teams.
MAINTENANCE INTRAVENOUS FLUIDS
- In normal adults obligatory daily intake includes roughly 500 cc of ingested water, 800 cc of
water from food, and 300 cc of water from oxidation
- Obligatory water output is roughly equal to 500 cc in urine, 500 cc from skin, 400 cc from
respiratory tract and 200 cc in stool.
- Keep the above in mind as patients who are unable to eat, as all obligatory sources need
replacing, not just that of ingested water.
- In the patient who is euvolemic with normal serum sodium then maintenance fluid can be
infused according to the following formula:
o Maintenance IV fluid infusion of 60 ml/hour plus 1 ml/kg per hour for each kilogram
over 20 kg up to a maximum of 120 ml/hour (or 3 liters per day). For example an
individual weighing 60 kg would have a maintenance fluid intake of approximately
1440 ml (60 ml/hour x24 hour) + 960 ml (40 kg x 1 ml x24 hours) or 2400 ml/day.
- Increased water intake required in the setting of sweating, burns, tachypnea, surgical drains,
polyuria, or ongoing GI loss.
- NS or NS with 20 meq KCL/liter are good choices of maintenance fluid in patients with
good renal function and normal sodium concentrations. Adjustments in concentration should
be made if hypo/hypernatremia develops.
- Bicarbonate addition can be beneficial in setting of acidosis. Prior to administration of 1-3
amps (50-150 meq HCO3) to maintenance fluid calculate the bicarbonate deficit to further
guide therapy.
- Be very cautious about providing maintenance fluids in patients who are overloaded and
especially if they are oliguric/anuric!
- For fluid repletion in setting of hypo/hypernatremia see separate section below.
Rose, Burton D. Maintenance and replacement fluid therapy in adults. Uptodate. January 2010.
CONTRAST INDUCED NEPHROPATHY
1. Definition: A fixed (0.5 mg/dl]) or proportionate (25 percent) rise in serum creatinine levels
after exposure to contrast. Usually transient, Cr peak at 3 days and returns to normal by day
10.
2. Risk factors: Diabetes, Cr>1.2-1.5 or GFR<50-60, age>75, periprocedure volume depletion,
heart failure, cirrhosis, nephrosis, HTN, proteinuria, NSAIDS, intraarticular injection of contrast,
and high dose of contrast
3. Intervention: Guidelines. Patients with normal kidney function do not require prophylactic
intervention.
- Patients with risk factors and GFR <50-60:
- If the GFR is less than 50, consider alternative imaging approaches.
- If contrast must be given, a low-osmolar agent should be used at the minimal dose
necessary.
- Measurement of the serum Cr should be repeated 24 to 48 hours after the contrast.
- NSAIDS and diuretics should be withheld for at least 24 hours before and after, if possible.
- Metformin should be withheld for 48 hours before and until it is certain that CIN has not
occurred.
- Hydration is key.
93
- Normal saline 1ml/kg/hr for up to 12 hours pre and post is studied best.
- Consider the IV bicarb (3cc/kg/hr x 1 hr, then 1cc/kg/hr x 6 hours) after (appears to be
better than saline) especially if the test is urgent.
- No definitive data to support mucomyst, but It is cheap, harmless, and may work.
Barrett, B. J. et al. N Engl J Med 2006;354:379-386)
NEPHROGENIC SYSTEMIC FIBROSIS

1. General concepts:
- Originally case reports focused on skin findings and was first known as Nephrogenic
Fibrosing Dermopathy subsequent reports have shown involvement of multiple organs.
- Caused by Gadollinium (Gd) induced activation of fibroblasts, has been reported with all five
marketed forms of Gd.
- Unfortunately no widely accepted treatment though improvements have been seen in
patients in which renal function improved after disease development.
2. Clinical Presentation:
- History of burning, itching, myalgias, arthralgias and skin hardening
- Woody indurated skin, SC nodules, hyperpig plaques, waxy erythematous papules
- Symmetric LE involvement most common, UE common as well but usually with LE first, trunk
late, face spared, can lead to severe contractures with pts becoming wheelchair bound and
death 2/2 complications
- Yellow scleral papules
- Can involve skeletal muscle, dura, rete testes, renal tubules, myocardium, pericardium,
pleura, lungs
- Onset days to months post Gd exposure
3. Patients at Risk:
- AKI or CKD with GFR <30
- Any degree of renal dysfunction attributed to HRS
Any degree of renal dysfunction post liver transplant
4. Diagnosis: Skin biopsy in appropriate clinic setting
5. Prevention/Risk Management:
- Does the pt need Gadolinium? Think hard about whether the study will change management
in a meaningful way.
- If test is performed in patient with impaired GFR either you/nephrology will have to write a
note documenting that benefit clearly outweighs risks. If the patient is hemodialyzed, then
consider three sequential dialysis sessions over three hours to remove 78%, 96%, and 99%
of Gd respectively (although currently no studies show that this modifies risk of NSF)
- Watch for signs and sxs and make sure to document in discharge summary events so that
future symptoms can be interpreted appropriately
ELECTROLYTE REPLACEMENT

1. Calcium:
- 1 amp Ca-gluconate raises ionized Calcium by 0.03
2. Magnesium:
- For every 0.2 mg/dL below normal give 1g MgSO4 IV
3. Phosphate:
- If replacing phosphate preferentially use oral phosphate, as IV formulations are potentially
dangerous as they can precipitate calcium. However they are preferred for severe
/symptomatic hypophosphatemia.
-Note: 10meq = 6.8mmol = 212.5mg
*for phosphate serum concentration below 0.5 mg/dl give 0.5 mmol/kg over 4-6 hours IV
and recheck/redose
*for phosphate serum concentration between 0.5-1 mg/dl give 0.25 mmol/kg IV over 4-6
hours and recheck/redose
*for serum concentrations between 1-1.5 mg/dl supplement with 15 mmol (or 0.2
mmol/kg) of phosphate (IV or oral=2 packets) and recheck/redose
*for levels 1.5-2.5 give 6.8 mmol (1 packet) of phosphate
*If patient is hypercalcemic dosage should be decreased by 25-50%
- Oral replacement preferable unless patient has symptomatic/severe hyophosphatemia.
94
* Neutra-Phos-K Packet: Phos 250mg; K 14.3meq; Na 7.1meq
* Neutra Phos Packet: Phos 250mg; K 7.1meq; Na 7.1meq
* K-Phos-Neutral tablet: Phos 250mg; K 1.3meq; Na 1.1meq
-In general higher K containing preparations preferred if K<4
4. Potassium:
- For every 0.1meq below normal, replace with 10meq KCl.
- If replacing via PIV then can replace no faster than 10meq/hr.
- If replacing via central line then can replace at 20 meq/hour
HYPERNATREMIA
1. Definition: Na > 145 mEq/L (Its a water not a salt problem)
2. Etiologies: Pt losing hypotonic fluid AND impaired access to free water (intubated, AMS and
so on) rarely infusion of hypertonic fluid or mineralocorticoid excess with ADH suppression.
3. Hypovolemic hypernatremia: (most common situation) divided into extrarenal vs. renal water
loss.
- Extrarenal losses- Diarrhea or insensible loss.
- Renal losses- loop diuretics, osmotic diuresis (Glu, Urea, mannitol)
4. Euvolemic hypernatremia: DI is divided into central vs nephrogenic. Think of it when UOP
>3L/day. If pt is ambulatory then hypernatremia is usually mild as thirst mechanism intact
- Central- Multiple etiologies resulting in ADH deficiency, most commonly seen in patients
post-neurosurgical procedure.
- Nephrogenic (ADH resistance); Li, Ca, postobstructive and recovery from ATN
5. Hypervolemic hypernatremia: Usually iatrogenic from injection of hypertonic saline or
NaHCO3 (post code) also the rare case of mineralocorticoid excess
6. Symptoms: irritability, seizures, lethargy, coma and death.
7. Treatment:
- Correct underlying etiology (address AMS, treat hyperglycemia, etc).
- Calculate free water deficit ([Na]
serum
-140)/140 and then determine rate of sodium change per
liter of infusate:
- ([Na]
serum
-[Na]
infusate
)/TBW+1 where TBW=0.60x IBW (x0.85 if female/elderly).
- In cases of symptomatic hypernatremia, correct serum sodium rapidly with assistance of
nephrology team. Goal is to correct free water deficit by 50% over 12-24 hours. Serial
electrolyte measurements q2 hours should be performed along with neurologic exams. Rate of
correction should be tapered based on improvement in symptoms.
- For chronic hypernatremia with no or mild symptoms, rate of correction should not exceed 0.5
meq/L/hour.
- If a volume deficit and hypernatremia are present. Intravascular volume should be restored
with normal saline prior to free water administration.
- If having trouble after above measures or problem is chronic consider asking pharmacy to
change IV abx and other IVs from NS to D5W
- Remember to never give sterile water IV
95
HYPONATREMIA
1. Definition: Symptomatic <115 mEq/L or rapid rate of change, Asymptomatic typically 120-135
mEq/L or slow change
2. Etiologies: Can be hypertonic (excess of another effective osmole, glu, mannitol draws H2O
out of cells and dilutes serum Na), isotonic (rare lab artifact 2/2 hyperlipidemia or
hyperproteinemia) or hypotonic which is the most common scenario and further subdivided by
volume status
o Hypovolemic Hyponatremia: Renal (thiazide diuretics) and Extrarenal (diarrhea, poor
PO, insensible loss, third spacing as in pancreatitis)
o Euvolemic Hyponatremia: SIADH, Hypothyroidism, AI, Tea and Toast, Beer
Potomania, massive polydipsia (>12L free water/d), Reset osmostat
o Hypervolemic Hyponatremia: CHF, Cirrhosis, Nephrotic Syndrome, Advanced CKD
6. Workup: Think about possibility of hypertonic or isotonic states (examples are hyperglycemic
pt or pt with large protein gap) if you have clinical suspicion measuring serum osm helps
confirm, in most cases the above are essentially ruled out by Hx/PE and basic labs, the next
and trickiest question is whats the pts volume status
- Use Hx/PE and vital signs (CVP, JVD, Orthostats, Skin turgor, mucus membranes) as first
line of investigation followed by
- Labs and imaging typically obtained in ED (Spec Grav on UA, BNP, BUN/Cr,
hemoconcentration, acid/base, CXR)
- Check urine osmolality- A low value is seen in euvolemic or volume overloaded patients
whereas a high value is seen in hypovolemic patients or in those with SIADH.
- Check TSH and Cortisol in patients who you are considering SIADH
7. Symptoms: Lethargy, altered mental status, seizures, and coma.
8. Treatment: Isotonic hyponatremia is a lab artifact; hypertonic hyponatremia is treated my
correcting underlying etiology, hypotonic hyponatremia as below:
- Severe Symptomatic Hyponatremia:
- Symptomatic (AMS, Sz) hyponatremia regardless of etiology is a medical emergency
requiring ICU level care with Na q2h
- Use hypertonic 3% saline for initial rapid correction at a rate of 2 meq/L/hr for the first
2-3 hours or until symptoms resolve. Remember that the max correction per 24 hour
period is 10-12 mEq.
- Use equation 3 to estimate rate of Na increase
- Hypovolemic:
- Volume repletion with normal saline to correct at a rate no greater than 0.5
meq/L/hour; 10-12 meq/L/day to avoid osmotic demyelination.
- Euvolemic:
- Low solute, feed pt regular diet
- SIADH
- Free H20 restrict ~ can initially try placing 1 L/day fluid restriction and tightening
this if needed.
- If fluid restriction fails to increase sodium consider hypertonic saline versus
conivaptan gtt (V1a/V2 antagonist) which requires central line.
- Consider Lasix - low dose to poison loop (not as diuretic)
- Hypervolemic:
- Free water restrict
- Diuresis, consider colloid in cirrhosis
96
HYPERKALEMIA
1. Definition: Mild 5.1-6.5 mEq/L, Moderate 6.5-8.0 mEq/L, Severe >8.0 mEq/L
2. Etiologies: Pseudohypokalemia, Transcellular shift, Decreased GFR, Normal GFR with
decreased potassium excretion (ie hypoaldosteronism). Most acute inpatient hyperkalemia
due to trancellular shift or AKI along with predisposing Renin-ATII-Aldo abnormality (DMII,
ACE, NSAID so on)
3. Workup:
- Rule out pseudohyperkalemia (repeat the lab if no good reason for elevated K) and
transcellular shift
- Assess GFR
- If GFR nml Calculate TTKG > 7, suggest nml aldo, if TTKG < 7, suggest hypoaldo state,
TTKG same as FeNa just put K in for Na and Osm in for Cr, think of it as a measure of aldo
activity (should be high in hyperK as body tries to kick K out)
- Check EKG for peaked T waves, flattening of p waves, prolonged PR interval, QRS
widening, ventricular arrythmias and eventually sine wave pattern and PEA.
4. Symptoms: Asx, weakness, cramps, arrhythmias, and sudden death.
5. Treatment:
- Stabilize the Cardiac Myocyte Membrane:
- Any EKG changes start with 1-2 amps Ca gluconate (CaCl though it has more Ca is
typically reserved for codes as extravasation from PIV can cause tissue necrosis)
caution if Dig toxic
- Shift K intracellularly:
- 10U Regular insulin, coadminister with 1 amp D50 to keep pt euglycemic (Dont need
the D50 if the pt is hyperglycemic)
- Consider albuterol 10-20mg neb, has little effect in up to 1/3 of pts, caution in pts who
are already tachycardic or arrythmia prone (h/o afib, SVT so on)
- Consider 1-2amps NaBicarb if serum bicarb <22, if bicarb higher will have little effect
- Can further temporize with insulin/D10 and bicarb ggts
- Decrease Total Body K:
- Kayexalate 30-90mg PO/PR, consider rectal tube if using PO (kayexalate today to
lower K tomorrow)
- Use the kidney, 90% of K excretion in the normal state, if pt eu- to hypovolemic
increase UOP with NS, once hydrated consider Lasix 40mg IV, assess response and
repeat
- Hold meds that impair K excretion (ACE/ARB, NSAID, Septra so on)
- HD: severe hyperkalemia or EKG changes beyond peaked T waves are a medical
emergency, call nephrology sooner rather than later as it takes time to get vascath in
and dialysis nurse with equipment
6. Tips:
- Outpatients on ACE/ARB, K up to 5.5 mEq/L, manage with dose reduction, low dose lasix,
d/c other meds that impair K excretion (NSAIDs), eval dietary habits (salt substitutes with
high K), NaHCO3 tabs in CKD pts with bicarb <22
- Dont go nuts with kayexalate if am labs on an otherwise stable inpt have mildly elevated K,
just give a little lasix if volume status allows, the nurse and pt will thank you
HYPOKALEMIA
1. Definition: K < 3.5 mEq/L.
2. Etiologies: divided into extrarenal vs. renal losses.
- Extrarenal: include diarrhea, vomiting, and increased insensible losses.
- Renal losses: diuretics, RTA, DKA, mineralocorticoid excess states.
3. Workup:
- Calculate TTKG. If TTKG < 3, then extrarenal losses, if TTKG > 7 then renal losses.
- On EKG can see U waves, T wave flattening, and ST changes.
- Should always check Mg since may be difficult to replace K if Mg is low
4. Symptoms: Fatigue, nausea, ileus, weakness, cramps, arrhythmias.
97
5. Treatment:
- Treat underlying etiology
- For every 0.1meq below normal, replace with 10meq KCl.
- Replace Mag to 2.0
HYPERCALCEMIA
1. Definition: Ca > 10.5 mg/dl
2. Etiologies: Can be divided into increased absorption vs. decreased excretion. Mnemonic
CHIMPANSEE (calcium excess, hyperparathyroidism, immobility, mild alkali syndrome/multiple
myeloma, Pagets dz, Addisons, neoplasm, sarcoidosis and granulomatous dz, excess vit D,
excess vit A).
3. Symptoms: moans, stones, groans, psychatric overtones (nausea, anorexia, abdominal pain,
kidney stones, altered mental status, polyuria). EKG: shortened QT.
4. Treatment: Fluids (2-4L NS) since most patients are hypovolemic, consider using diuretics
but need to hydrate prior to use, otherwise will worsen hypercalcemia. Consider calcitonin (4
iu/kg SQ q12h) which decreases Ca by 1-2 mg/dl in 2-3 hrs. Other options include
bisphosphonates and steroids for long term control.
-Treat underlying etiology!
HYPOCALCEMIA
1. Definition: Ca < 8.5 mg/dl. Correct w/albumin since 40% of calcium is bound to albumin and
only ionized Ca is active. Every decrease in alb by 1 g/dl from 4, add 0.8 to measured Ca.
2. Etiologies decreased absorption, increased loss, and sequestration, decreased osteoclastic
activity
- Decreased absorption- etiologies inclue vitamin D deficiency, hypoparathyroidism, calcium
chelators.
- Increased losses usually through diuretic use
- Sequestration-blood transfusion, pancreatitis, tumor lysis
- Decreased bone turnover- hypoparathyroidism, cinacalcet, bisphosphonates
3. Workup: check PTH, Vitamin D-25, Vitamin D-1,25, mag, phos, albumin, alkaline
phosphatase, urine calcium.
4. Symptoms: tetany, muscle spasms, weakness, paresthesia, hypotension, seizures, altered
mental status. On EKG can see prolonged QT.
5. Treatment:
- Always check Mg and replace it, since difficult to replace Ca if low.
- For mild hypocalcemia (Ca > 8.0 mg/dl or ionized Ca2+ > 0.8) give calcium carbonate.
- For mod to severe hypocalcemia (Ca < 7.0 mg/dl or ionized Ca2+ of <0.7 mEq/dl) or
symptomatic, give 2 - 3 amps of calcium gluconate. Elevation in calcium only transient (2-3
hrs) and should recheck Ca in several hours. May consider repeating dose in 6 hr or
consider starting calcium gtt at 0.5-1.5 mg/kg/hr for several hours.
- 15 mg/kg Ca needed to raise Ca by 2-3 mg/dl.
- 1 amp Calcium gluconate has 90 mg Ca
- 1 amp Calcium chloride has 272 mg Ca (although can cause tissue necrosis if extravasates).
- 1000 mg Calcium carbonate has 400 mg Ca but keep in mind that this is slower acting.
98
HYPERPHOSPHATEMIA
1. Definition: Phosphate > 4.5 mg/dl
2. Etiologies: divided into decreased excretion, increased intake, and shift from intracellular to
extracellular space
- Decreased excretion usually due to renal failure and hypoparathyroidism
- Increased intake usually from vit D intoxication
- Shift occurs in rhabdo, trauma, tumor lysis syndrome, insulin deficiency, acidosis
3. Symptoms: Usually asx but can result in calcium deposition and acute hypocalcemia.
4. Treatment: treat the underlying cause for chronic hyperphosphatemia, also can use oral
phosphate binding drugs such as calcium carbonate, calcium acetate, and sevelamer.
HYPOPHOSPHATEMIA
1. Definition: phosphate < 2.6 mg/dl
2. Etiology: increased excretion, decreased intake, and EC to IC shifts
Increased excretion occurs with hyperparathyroidism, thiazides, acetazolamide, and
Fanconis syndrome
Decreased intake w vit D def., antacids, poor nutrition
Shifts can occur w/ refeeding (increase use of phosphate), tx of DKA or HNK, alkalosis
3. Symptoms: usually symptomatic when < 1.0 and include tremor, paresthesia, weakness,
AMS, rhabdomyolysis
4. Treatment:
- Remember that only phosphate checked while fasting is reliable.
- If replacing phosphate preferentially use oral phosphate as IV formulations are potentially
dangerous as they can precipitate calcium but preferred for severe /symptomatic
hypophosphatemia.
-Note: 10meq = 6.8mmol = 212.5mg
*for phosphate serum concentration below 0.5 mg/dl give 0.5 mmol/kg over 4-6 hours IV
and recheck/redose
*for phosphate serum concentration between 0.5-1 mg/dl give 0.25 mmol/kg IV over 4-6
hours and recheck/redose
*for serum concentrations between 1-1.5 mg/dl supplement with 15 mmol (or 0.2
mmol/kg) of phosphate (IV or oral=2 packets) and recheck/redose
*for levels 1.5-2.5 give 6.8 mmol (1 packet) of phosphate
*If patient is hypercalcemic dosage should be decreased by 25-50%
- Oral replacement preferable unless patient has symptomatic/severe hyophosphatemia.
* Neutra-Phos-K Packet: Phos 250mg; K 14.3meq; Na 7.1meq
* Neutra Phos Packet: Phos 250mg; K 7.1meq; Na 7.1meq
* K-Phos-Neutral tablet: Phos 250mg; K 1.3meq; Na 1.1meq
-In general higher K containing preparations preferred if K<4
HYPOMAGNESEMIA
1. Definition: Mg < 1.8 mg/dl
2. Etiologies: include GI losses from diarrhea, malabsorption, vit D def, renal losses due to
diuretics, hyperparathyroidism
3. Treatment:
- Low magnesium can make correcting hypocalcemia and hypokalemia more difficult
- For every 0.2 mg/dL below normal, give 1g MgSO4 IV. Should raise to goal above 2 mg/dl in
cardiac patients.
- Oral mag used for dietary supplementation and mild hypomagnesemia and should be given
TID.
- IV preparation is MgSO4 1000 mg (98 mg elemental Mg)
99
- Oral replacements at University Hospital include Mg Oxide 250 mg (150 mg elemental Mg) or
500 mg (300 mg elemental Mg), and Slow Mag (64 mg elemental Mg). At the VA, can give
Mg Oxide 400 mg (242 mg elemental Mg).
EQUATIONS
- TBWater = WT(kg) x 0.6 (if male) or 0.5 (if female)
- Free water deficit = TBW x (Serum
Na
140) 140
- Change in serum Na with 1L of IVFs = (Na in IVFs in mEq - Serum
Na
) (1+TBW)
- 24 creatinine clearance = [U
Cr
x Volume (ml)] (P
Cr
x 1440)
- TTKG = (U
K
x P
osm
) (P
K
x U
osm
)
- Serum osmolarity = 2 x Na + BUN/2.8 + Glu/18
- Electrolyte Free Water Clearance = 24 Hr Urine Volume [1- (Urine
Na
Urine
K
/Serum
Na
)]
- A-a gradient = [FiO2(713) PaCO2/0.8] PaO2
- Expected A-a gradient = 0.21 x age + 2.5
ACID-BASE DISTURBANCES
Normal Values: pH 7.35-7.45
pCO
2
35-45
pO
2
80-100
HCO
3
22-26
To determine if the ABG is accurate, calculate H
+
=24 x pCO
2
/HCO
3
.
H
+ 70 60 50 40 32 25
pH 7.16 7.22 7.30 7.40 7.50 7.60
Step 1: Look at the pH to determine the primary disturbance. Use pCO
2
and HCO
3
to determine if it is respiratory or metabolic and if appropriate compensation
is present (see below).
- pH < 7.35 = primary acidosis
- pH > 7.45 = primary alkalosis
Step 2: Calculate the anion gap to determine if a gap metabolic acidosis is present.
(Normal AG = 10 in young adults; 12 in older adults)
Anion Gap = Na
+
(Cl
--
+ HCO3
--
)
Step 3: If an anion gap is present, calculate the excess gap to determine if a
concomitant non-gap metabolic acidosis or a concomitant metabolic
alkalosis is present.
Excess AG = (Calculated AG Normal AG) + Measured HCO
3
Excess Anion Gap > 30 = metabolic alkalosis
Excess Anion Gap < 22 = non-gap metabolic acidosis
Numbers to Remember:
Acute Respiratory Process: pH changes 0.08 per 10mmHg increase in pCO2
Chronic Respiratory Process: pH changes 0.03 per 10mmHg increase in pCO2
A-a gradient = [FiO2(760-47) PaCO2/0.8] PaO2
Expected A-a gradient ~ Patients Age/4 + 4
Example: 7.10/50/94/15, Na 145, Cl 100
Step 1: Acidosis is present. Since pCO
2
is elevated, there is a primary respiratory acidosis.
However, an acute respiratory acidosis only accounts for a 0.08 drop in the pH.
Step 2: The AG = 145 (100+15) = 30. Therefore, there is an anion gap metabolic
acidosis also present.
Step 3: Excess AG = (30-12)+15=33. Therefore, a concomitant metabolic alkalosis is
also present.
References: Haber, RJ. A practical approach to Acid-Base Disorders. Western J Med, Aug 1991;155(2):146-151

100
METABOLIC ACIDOSIS
Gap Acidosis Non-Gap Acidosis
Methanol ingestion
Uremia
Diabetic Ketoacidosis
Paraldehyde
Intoxication (EtOH)
Lactic acidosis
Ethylene glycol
Salicylate ingestion
Diarrhea (GI losses)
RTA
Early renal failure
Mineralocorticoids
Alkali intake
Potassium depletion
1. Usually see pH < 7.35 and HCO3 < 22 mEq/L.
2. Determine if Respiratory Compensation is adequate or if there is a secondary respiratory
disorder.
Winters formula: pCO2 = 1.5 x HCO3 + 8.
3. If pCO2 is higher than expected, there is a secondary respiratory acidosis. If pCO2 is lower
than expected, there is a secondary respiratory alkalosis.
4. Determine whether there is an anion gap. Anion gap = Na (HCO3 + Cl). Normal anion gap
is <12. If pt has low albumin, then need to adjust anion gap. This can be done by adding 1.5
to the anion gap for every 1 g/dl decrease from 4 in albumin.
5. If anion gap is present, then check for serum ketones and/or lactate based on clinical
presentation. If concerned about alcohol poisoning, calculate the serum osmolar gap. If
osmolar gap is > 10, then consider methanol and ethylene glycol.
6. There may be a concomitant anion gap and nonanion gap acidosis or metabolic
alkalosis. First calculate Delta gap (anion gap 12). Delta + HCO3 > 30 suggests metabolic
alkalosis. Delta + HCO3 < 23 suggests additional nonanion gap acidosis. Another method is
to calculate delta gap/delta HCO3. If ratio > 1, then addl metabolic alkalosis. If ratio < 1,
then addl nonanion gap metabolic acidosis.
7. For nonanion gap acidosis, etiology is divided between renal and extrarenal. Renal etiologies
include RTA. Extrarenal include diarrhea, pancreatic fistula, ureteral diversion. To
differentiate between renal and extrarenal, calculate the urine anion gap = UNa + Uk UCl. If
extrarenal losses, then UAG will be negative since kidneys will be compensating by producing
more NH4+. If renal losses, then UAG will be positive.
METABOLIC ALKALOSIS
Low Urine Chloride NL/High Urine Chloride
Vomiting
Diuretics
Post-hypercapnea
Hypercortisolism
Steroid use, Cushing
Diuretics
Increased renin, RAS
1. Usually see pH > 7.45 and HCO3 > 26 mEq/L.
2. Determine if respiratory compensation is adequate or if there is a secondary respiratory
disorder. Formula is pCO2 = 0.7 x HCO3 + 20. If pCO2 is higher than expected, then there is
a secondary respiratory acidosis. If pCO2 is lower than expected then there is a secondary
respiratory alkalosis.
3. UCl< 20 indicates GI loss while UCl>20 indicates early diuretic use or hyperaldosteronism.
101
RESPIRATORY ACIDOSIS
CNS/Respiratory Drive (Drug overdose, OSA)
Lung Disease (ie, Acute COPD Exacerbation)
Chest Cavity (neuromuscular, PTX)
1. Usually see pH <7.35 and HCO3 > 26 mEq/L for respiratory acidosis.
2. Can estimate if pH is appropriate for given pCO2. pH decreases 0.08 per 10mmHg increase
in pCO2 for acute process. pH decreases 0.03 per 10mmHg increase in pCO2 for chronic
process. Partial pressure of O2 decreases as pCO2 increases, resulting in severe hypoxemia
as a consequence of respiratory acidosis.
3. Determine if metabolic compensation is appropriate. For every 10 increase in pCO2,
increase in HCO3 by 1 mEq/L for acute vs. 4 mEq/L for chronic. If HCO3 is higher than
expected, then there is an underlying metabolic alkalosis. If HCO3 lower than expected, then
there is an underlying metabolic acidosis.
RESPIRATORY ALKALOSIS
Anxiety/Pain/Fever
Hypoxia/Lung Disease (PE,
early bronchospasm)
CNS Disease
Mechanical Ventilation
Central hyperventilation
Drugs (Aspirin, progesterone)
Pregnancy
Hepatic Encephalopathy/ESLD
Hyperthyroidism
Sepsis
1. Usually see pH >7.45 and HCO3 < 22 mEq/L for respiratory alkalosis.
2. Determine if metabolic compensation is appropriate. For every 10 decrease in pCO2,
decrease HCO3 by 2 mEq/L for acute vs. 4 mEq/L for chronic. If HCO3 higher than
expected, then also underlying metabolic alkalosis. If HCO3 lower than expected, then
underlying metabolic acidosis.
3. Sepsis and salicylate toxicity are the only single disorders that cause both gap metabolic
acidosis and respiratory alkalosis.
4. Cannot predict pH change with change in pCO2 for respiratory alkasosis. Only acid/base
disorder in which compensation can return pH to normal.
GENERAL CONCEPTS
- Insensible losses are about 500-1000 cc/day but increased in patients with fever, diarrhea,
vomiting, intubated, etc. For every 1 F increase, insensible losses goes up by about 60-80
cc/day
- Kidneys able to regulate osmolarity of urine between 50 mosm/kg to 1200 mosm/kg (based
upon low or high ADH levels repectively)
- Normal GFR is 100-125 cc/min in men, 85-105 cc/min in women
- Oliguria is < 400 cc/24 hrs, Anuria is < 50 cc/24 hrs, Polyuria is > 3000 cc/24 hrs
- MDRD is a better measure of GFR than Cockroft-Gault equation or 24 hr creatinine clearance
- 24 hr creatinine clearance overestimates GFR with in pts with low GFR
- Use of creatinine as measure of GFR during AKI is inaccurate
- NS has 154 mEq/L of Na, NS has 77 mEq/L, 3%NS has 512 mEq/L
- FENa < 1%suggests prerenal azotemia, however, affected with use of diuretics. In diuretic
use, FEUrea < 35%suggests prerenal azotemia.
102
103
ENDOCRINE
CORRECTING SUGARS
1. Hypoglycemia: If mild, can give orange juice (unless hyperkalemic); if severe D50 (1 amp
iv); if low nl sugars, can hold short acting insulin/oral secretagogues; but if low nl and
nurses ask to hold long acting, consider giving with close monitoring as sugars will be very
high by AM if you hold their basal insulin; hypo is almost always iatrogenic.
2. Hyperglycemia: If mild, can give additional SS dose of SQ short-acting insulin. Find out if
type 1 or 2 (risk for DKA vs. HONK); check chem for gap, serum/urine ketones for DKA,
consider VBG if suspect DKA. If patient is in DKA, aside from w/u of cause, will need
transfer of care to at least IMU and likely ICU for insulin gtt and q 1-2 hour finger sticks.
DIABETES MELLITUS
Medications:
1. Insulin Types and Sliding Scale:
Insulin Action Onset Peak Duration
Lispro/Humalog
Aspart/Novalog
Glulisine /Apidra
Immediate 5-15 1-2hrs 3-5hrs
Regular Short acting 30-60 2-4hrs 4-8hrs
NPH Intermediate 1-3hrs 4-10hrs 10-18hrs
70/30 30-60 to 3-4hrs 2-10hrs 10-18hrs
Glargine/Lantus Long acting 2-3hrs No true peak ~24hrs
- U500: this is regular insulin that is 5x as concentrated and is useful for people who are
very insulin resistant and use more than 100 units of insulin (at regular concentration 1x)
for each injection. U500 is give bid to qid and given before meals (in place of both short
and long acting insulin). For more info or dose conversion, consult endocrine.
2. Injectables:
- Exenatide/Byetta: an incretin mimetic. Indicated for use w/metformin and/or
sulfonylurea. Start 5mcg bid w/in 1h of am/pm meal, may inc to 10mcg bid after 1mo.
Induces some weight loss. Decrease HbA1c by 1%. Nausea can occur. Expensive.
- Pramlintide/Symlin: co-secreted w/ insulin. Suppresses glucagon secretion, delays
gastric emptying, promotes satiety, stabilizes post-prandial glucose, induces wt. loss.
Adjunct use with meal time insulin. Start 30 mcg SC for type II, increase up to 120mcg
qac. For type 1 DM, start 15 mcg SC qac, slowly increase to goal 45 mcg QAC if nausea
manageable. Need to reduce insulin by 50% when Symlin is started. Decreases A1c by
0.5 %.
3. Oral Hypoglycemics:
- Metformin: should be held when Cr >1.4 ( women), Cr > 1.5 (men), GFR < 70,
susceptible to lactic acidosis, decompensated CHF.
- Hold on the day of IV iodine dye studies and restart 2 days after w/ normal Cr.
- Titrate up at 500 mg interval every week to avoid GI upset.
- First drug of choice for new type 2 diabetes. Helps with weight loss. Decrease A1c
by 1-2%.
- Sulfonylurea: glyburide has active metabolites that also cause hypoglycemia. For
liver/renal dysfunction and in the elderly, glipizide is better tolerated. Decrease A1c by 1 to
2%.
- Meglitinides: short acting insulin secretagogue. Useful for postprandial hyperglycemia.
- Thiazolidinediones: not recommended for pt w/ severe liver dysfunction or CHF NYHA
III or IV.
- LFTs before the start and later only if clinically indicated.
104
- Side effects: weight gain, lower extremity edema and some reports of macular
edema. Decrease A1c by 1-2 %.
- Alpha-glucosidase inhibitors: not for pts with GI illness or liver dysfunction. Acarbose
should be titrated at weekly interval. Decreases A1c by 0.5 - 1%. Check LFTs q3mon in
the 1
st
year.
Outpatient Diabetes Management:
1. Screening: Glucose: pt over age 45 q3yr, or earlier if BMI > 25, IFG/IGT*, physical
inactivity, FHx, Non-Caucasian, h/o GDM, components of dysmetabolic syndrome, PCOS,
vascular disease, or insulin resistance.
- Impaired fasting glucose (IFG) = glucose between 100 and 125 after fasting 8 hrs
- Impaired glucose tolerance (IGT) = glucose between 140 to 199 2 hrs post-prandial.
2. Established Diabetes Patient Monitoring and Care:
Outcome
Measures:
Frequency: Goal:
A A1C Q3-6 m < 7%
B BP Every visit goal bp < 130/80 unless MA>30 mcg/g,
then consider ACE I and bp goal<125/75
C Cholesterol Q1y if ctrl goal < 70-100 mg/dL
D Diet Usually 1800, 2000, or 2200 kcal diet
E Eye Exam Annually screen for retinopathy (Q1y in DMII, after 5
y Q1y in DMI)
F Foot Exam Annually monofilament and tuning fork, refer to
vascular surg or podiatry when appropriate
G Glucose As directed fasting fsg 90-130, postprandial < 180, fsg
as directed*
*fsg monitoring:oral agents/Qd insulin: 1-
2x/d, BID insulin: 2-4x/d, TID-QID: 3-6x/d
H Healthy Living self mngmt goal, get active, healthy diet,
smoke cessation, check fsg, daily foot
checks, know meds. *At UCSD, can refer
to Project Dulce education program.
I Immunize Annually Flu shot annually, Pneumovaccine once,
then repeat in five years once > 65 y.o.
J Join Tob
Cess
call 1-800-NO-BUTTS, or begin Chantix
K Kidneys Annually microalbuminuria & eGFR yearly, goal MA
< 30 mcg/gm
3. Diet Recs: If moderately active: 30-35 kcal/kg/day; for weight loss, can reduce to 20-30
kcal/kg/day. Decrease of 500 kcal/day should result in gradual wt. loss of 1 lb/wk. Typical diet
50-60%carbs w/ complex carbs emphasized, 10-20%protein (< 10%/d if overt diabetic
nephropathy), <30%calories from fat/d, w/ < 10%saturated fat, and total cholesterol <
300mg/d. Emphasize high fiber 20-35 gmsol/insoluble. If HTN: Na+ restrict to <2400 mg/d; if
nephropathy, Na+ < 2000 mg/d.
105
INPATIENT DIABETES MANAGEMENT
When in doubt, use the EPIC inpatient subcutaneous insulin order set
1. Target blood glucose range: Goal for in-hospital patients is controversial given recent
report of increase mortality for critically ill pt in MICU. Goal is to keep fsg close to 110
fasting (90-130 according to ADA for non-critical ill patient, 100-150 in critically ill) and <
180 postprandial (Standards of Medical care in Diabetes-2006, ADA, Diabetes care, 2006,
29(S1), p. s29). Should get HbA1c if none in the last 3 months.
2. Stopping oral medications: In acute settings, it is prudent to stop oral hypoglycemics. In
addition, metformin should be discontinued in patients with a serum creatinine >1.5 or in
whom there is a risk of nephrotoxicity (e.g., receiving iv contrast); sulfonylureas should not
be used in the NPO patient; and glitazones should be discontinued in patients with CHF
decompensation or volume overload.
3. For patients eating meals or receiving bolus tube feeds: Glargine insulin is the most
physiologic basal insulin and is recommended in these patients. Lispro insulin is more
appropriate than regular insulin for nutritional doses due to its shorter, more predictable
half-life and correspondence with UCSD inpatient meal times. If you choose to use NPH
instead of glargine, the distribution of TDD should be modified: Basal insulin: NPH -- 0.5 x
TDD; dose 2/3 of this amount before breakfast and 1/3 of this amount at bedtime.
Nutritional insulin: Lispro -- 0.17 x TDD, dosed with first bite of each meal.
4. For patients receiving continuous enteral or parenteral nutrition:
- Consider using an insulin infusion for optimal control in this setting. Keep insulin separate
from TPN until a stable dose is reached. Glargine insulin is the most physiologic basal
insulin and is recommended in these patients, as it has no serum spike and therefore
has less hypoglycemia than NPH.
- Regular insulin is recommended as the nutritional insulin rather than lispro - because of
its longer half-life, it is better suited to continuous nutritional sources.
- If tube feeds or parenteral nutrition are held, the nutritional regular insulin doses should
also be held.
5. For the NPO patient: NPO patients have fewer episodes of hypoglycemia when given a
low-dose dextrose infusion like D5 NS at 75/hr along with their basal long acting insulin.
Nutritional or scheduled short-acting insulin should not be given to patients without a
nutritional source. Sliding scale insulin should be offered in small doses only once FSG >
200.
- Type I Diabetes: Always require insulin, even when NPO, can become ketotic w/in 12-24h.
- Type II Diabetes: Hold oral meds in general. TZD ok to ctn.
- Type I and II DM: If on intermediate acting insulin (NPH), give on the morning of
procedure. Patients on long acting HS insulin, reasonable to decrease dose by approx 10-
20%.
106
6. Starting/Adjusting Long Acting Insulin:
- Lantus/Glargine: When A1C> 9% and already on 2-3 PO agents: Start 10 U QHS, and
d/c sulfonylurea. Adjust insulin Q3d by 2U HS until FSG < 130. If after 3 mo, A1C >
7%, add pre-meal insulin or change to 70/30.
Intermediate/NPH Insulin
Calculate 0.5 U/kg/d body wt to estimate
TDD. Gi ve 2/3 am, 1/3 pm
AM dose If pre-lunch fsg > 120 Pre-dinner > 150 Inc am dose by 3U
Pre-dinner > 250 Inc am dose by 5U
PM dose If HS > 150 Am > 150 Inc pm dose by 3U
Am > 250 Inc pm dose by 5U
MANAGEMENT OF DKA& HHS
Initial evaluation: plasma glucose, chemistry panel (calculate anion gap), osmolality, serum and
urinary ketones, and UA, as well as initial ABG and a CBC with a differential. An ECG, CXR, and
urine, sputum, or blood cultures should also be obtained based on clinical presentation.
Precipitating factors should be identified and treated (infection, medication non-compliance, MI,
etc).
Diagnostic Criteria:
DKA: Blood glucose >250, pH < 7.3, bicarbonate < 15, ketonuria and ketonemia.
HHS: Blood glucose > 600, pH > 7.3, bicarbonate >15, osmolality >320.
Fluid Management:
Replacement varies based on age, wt, hemodynamics and comorbidities.
Replace intravascular volume: Give 1L 0.9% NS over 30-60min. Give additional NS until
hemodynamically stable and urine output increased.
Replacement of total body water deficit: Calculate corrected Na. Low Na: Give 0.9% NS 250-
500ml/hr*. Normal/High Na: Give 0.45% NS 250-500ml/hr*. When BG <200 (DKA) or <300
(HHS) add D5 and decrease rate to 100-200ml/hr.
*Rate dependent on volume status. Fluid replaced over 12-24hr. Generally patients in DKA
depleted 3-6L and in HHS 8-12L. Care taken in patients with CHF and renal disease.
Insulin Management:
Bolus: 0.1-0.15U/Kg of regular insulin IV and then start continuous infusion (DKA: Start at
0.1U/Kg/hr HHS:start 0.05U/Kg/hr)*. Check BG q1hr until 3 consecutive in target range of 50-
75/hr.
Decrease insulin by 50% if BG decreases >100 in any 1hr period.**
Increase insulin by 50%if BG decreases <50 in any 1hr period.
Decrease infusion by 50% once at target range (DKA 150-200, HHS 250-300). May need to
adjust again after adding D5 to fluid replacement.
Start SC insulin once patient able to eat AND AG closed and Bicarb >15 (DKA) or Mental
Status improved and normal osmolality (HHS).
Stop insulin gtt 1-2 hours after ensuring the patient has a meal, given twice the hourly rate of
infusion in the form of short acting insulin SC and Long acting insulin at 0.2-0.3U/kg or home
insulin dose.
*Alternative to bolus regimen is to start the insulin gtt at 0.14U/kg/hr then giving 0.14U/Kg bolus if
not achieving target decrease/hr.
**Lowering glucose >100mg/dL/hr may cause osmotic encephalopathy.
Electrolyte Management: Repeat chem panel q2-4h based on imbalance
Potassium:
K >5.5: No supplementation
K 4-5.4: add 20mEq of KCl/L to IVF
K 3-3.9: add 40mEq of KCl/L to IVF
K <3: add 60mEq of KCl/L to IVF
In DKA, do not give insulin if initial K <3.3 until it has been supplemented (Risk of severe
hypoKalemia).
Bicarb replacement: Generally not needed.
Reasonable to give when pH <7, HCO3 <5-10, Cardiac instability related to acidosis, severe
hyperKalemia.
Kitabchi, AE et. al. Diabetes Care. 2009; 32: 1335.
107
ADRENAL INSUFFICIENCY
Primary Adrenal Insufficiency
Etiology:
1. Autoimmune (Addisons Disease)
2. Metastatic malignancy (lung, GI, breast, renal) or lymphoma
3. Adrenal hemorrhage
4. Infectious TB, CMV, MAC, fungal, HIV
5. Adrenoleukodystrophy
6. Infiltrative d/o amyloidosis, hemochromatosis
7. Congenital adrenal hyperplasia
8. Familial glucocorticoid deficiency and hypoplasia
9. Drugs ketoconazole, etomidate, suramin, aminoglutethimide and metyrapone
Signs and symptoms:
Chronic primary adrenocortical insufficiency: weakness, fatigue, anorexia, wt loss,
hyperpigmentation (tan, buccal mucosa, gums, palmar creases due to increased
hypothalamic pro-opiomelanocortin release), hypotension, GI disturbances, salt craving,
postural symptoms, abdomi nal pain
Acute adrenal crisis: hypotension and shock, fever, dehydration, volume depletion, N/V, anorexia,
weakness, apathy, depressed mentation, hypoglycemia
Secondary Adrenal Insufficiency
Etiology: Most commonly ACTH deficiency due to exogenous glucocorticoid therapy. Also may
see pituitary or hypothalamic tumors.
Signs and symptoms: Similar to primary adrenal insufficiency, but no hyperpigmentation or
mineralcorticoid deficiency.
Diagnosis
- Check early AM random serum cortisol: <3 g/dl is diagnostic, >18 g/dl rules out
diagnosis unless pt in critical illness, then proceed to cosyntropin test.
- Cosyntropin stimulation test:
o Check random serum cortisol
o Administer 250 g cosyntropin (some advocate 1 g for more
physiologic dosing)
o Recheck serum cortisol 45-60 min post cosyntropin
Adrenal insufficiency diagnosed if serum cortisol increases by 9 g/dl.
Treatment
Acute adrenal crisis:
1. Hydrocortisone 100mg IV q8h x 24 hrs (longer if major complications, ie sepsis).
2. When stable, decrease to hydrocortisone 50mg q6h. In primary Addisons disease,
add mineralocorticoid replacement with fludrocortisone (see below).
3. Taper to maintenance dose of 10mg TID/QID by day 4 or 5.
Maintenance therapy:
1. Hydrocortisone 10-15mg qAM, 5-10mg qPM
2. Fludrocortisone 0.05-0.1mg qAM. (NOT needed for secondary adrenal insufficiency)
3. Increase hydrocortisone dose during stress. Double oral dose for mild illness.
Provide pt with injectable form of glucocorticoid (ex: dexamethasone 4mg IM) for
emergency use.
Steroid coverage for surgery:
1. Hydrocortisone 100mg IM on call to OR.
2. Hydrocortisone 50mg IM or IV in recovery room and then q6h x 24 hrs.
3. If stable, reduce dose to 25mg q6h x 24 hrs then taper to maintenance dose over 3-
5 days. Resume fludrocortisone dose when pt taking PO meds.
4. Maintain/increase hydrocortisone dose to 200-400mg/d if fever, hypotension or
other complications.
108
PRIMARY HYPERPARATHYROIDISM
Etiology:
- 80% parathyroid adenoma
- 15% primary parathyroid hyperplasia
- 1-2% parathyroid carcinoma
Signs and Symptoms:
1. Bone disease:
- Osteitis fibrosa cystica: bone pain, pathologic fractures. Elevation of alk phos on
labs. Increase in osteoclasts, marrow fibrosis, cystic lesions that contain fibrous
tissue (brown tumors). Subperiosteal resorption of cortical bone most sensitive
radiologic finding.
- Osteoporosis: preferential loss of cortical bone. Mass and mechanical strength of
trabecular bone are maintained in mild disease.
2. Kidney disease:
- Ca oxalate stones: indication for parathyroidectomy.
- Nephrocalcinosis: usually not clinically evident, but do see gradual loss of renal
fxn.
- Polyuria and polydipsia: from loss of renal concentrating ability due to chronic
hypercalcemia.
3. Nonspecific features: lethargy, fatigue, depression, difficulty concentrating, personality
changes, muscle weakness, dyspepsia, nausea, constipation, chondrocalcinosis.
Labs:
- hypercalcemia, low normal/low phosphorus (2/2 phosphaturic effects of PTH), mild
hyperchloremic metabolic acidosis (always check albumin at same time)
- Intact PTH elevated or upper normal
- 24 hour urinary calcium and urinary Cr excretion to exclude FBHH
Imaging:
- Sestamibi scanning, ultrasound, CT scan to localize abnormal gland for surgery ONLY if
hyperparathyroidism with clinically significant signs and symptoms (i.e. osteoporosis) or palpable
nodule.
Treatment:
- Indications for surgery: Serum Ca >1mg/dL above upper limit of normal, previous episode
of life threatening hyperCa, CrCl reduced below 70% normal, kidney stone, UCa
>400mg/24h, BMD T< - 2.5 at lumbar spine, hip or distal radius, <50 yo, long term medical
surveillance not possible.
- Parathyroidectomy: definitive treatment of primary hyperparathyroidism. High recurrence
rates in patients with parathyroid hyperplasia. 95% cure rate for single parathyroid
adenoma.
- Medical therapy: High dose estrogen, oral bisphosphonates and raloxifene can be tried in
postmenopausal woman. Cinacalcet for secondary hyperparathyroidism in pts on dialysis.
Variants of Primary Hyperparathyroidism
1. Familial benign hypocalciuric hypercalcemia (FBHH)
- Autosomal dominant inheritance
- Mild hyperCa, mild hypophosphatemia, hypermagnesemia
- PTH normal/slightly elevated
- HYPOCALCIURIA, UCa <50mg/24h, Ca/CrCl ration <0.01
- Important to distinguish from primary hyperparathyroidism to avoid unnecessary
surgery.
2. MEN Syndromes: hyperparathyroidism seen in both MEN 1 and MEN 2A
3. Lithium therapy
- Exposure to extracellular lithium shifts set point higher for inhibition of PTH
secretion.
- Also produces hypocalciuria, looks similar to FBHH.
- Difficult to diagnose primary hyperparathyroidism in lithium treated patients.
However, primary hyperparathyroidism likely with serum Ca >11.5 mg/dL
109
OSTEOPOROSIS
Definition: DEXA T score -2.5 (Osteopenia is T score between -1 and -2.5) OR dx of low-impact
fracture (i.e. vertebral compression fracture). Low bone mass and deterioration of bone
microarchitecture. This increases fracture risk.
Who to screen:
- NOF recs: Women > 65yo, Men > 70yo, Younger postmenopausal women with a clinical
risk factor, Men 50-70yo with a clinic risk factor, personal hx of adult fracture.
- USPSTF recs: Woman > 65 yo, woman 60-65 with a clinical risk factor
Medication risks: Glucocorticoids (equivalent of >5mg prednisone/d x 3 mos), dilantin,
phenobarbital, suppressive doses of thyroxine, depoprovera , heparin, TCAs, antipsychotics,
benzos, lithium, GnRH analogue, cyclosporin, tacrolimus, methotrexate
Disease risks: Primary hyperparathyroidism, hypogonadism, thyrotoxicosis, IDDM,
hyperprolactinemia, premature ovarian failure, RA, SLE, Sprue, IBD, HIV/AIDS, Cushings,
ESRD.
Who to treat:
- Men > 50yo or postmenopausal women with T score -2.5
- Men > 50yo or postmenopausal women with hx of hip or spine fracture
- T score between -1 and -2.5 AND 10 yr hip fracture risk >3% OR 10 yr major osteoporotic
fracture risk >20%.
- Calculate fracture risk with online FRAX: www.shef.ac.uk/FRAX
Treatment:
- NOF recs for Ca/Vit D:
o <50 yo Ca 1000 mg/d, Vit D 400-800 IU/d
o >50 yo Ca 1200 mg/d, Vit D 800-1000IU/d
o Take Calcium in divided doses for better absorption
- Weight bearing exercise and muscle strengthening exercise
- Avoid smoking and excess alcohol
- Antiresorptives: fills in resorptive cavity, increases mineralization
o Estrogen: preserves/increases BMD, reduces spine, hip and nonvertebral
fractures. Increases risk of breast cancer, VTE, coronary disease, stroke.
o Raloxifene: increases spine and hip BMD, reduces risk of vertebral fractures.
Reduces risk of breast cancer. Increases risk of VTE, leg cramps.
o Bisphosphonates: Alendronate, Risedronate, Ibandronate, Zoledronate
Spine fracture reduced by 50% in all
Hip fractures reduced by 40%with alendronate, risedronate
and zoledronate
Poorly absorbed, GI intolerance with oral agents
50% excreted by kidney, 50% binds to bone
Not recommended for GFR <30-35 ml/min
Risk of osteonecrosis of jaw, stronger association with IV
bisphosphonate for metastatic bone disease,
risk of spontaneous subtrochanteric femur fx
in those using bisphosphanates >10 y.
- Anabolic: produces osteoid, increases connectivity
o Teriparatide: increases BMD at spine and hip, increases bone turnover
markers, decreases spine and nonvertebral fractures. Black box warning:
osteosarcoma with lifetime use of this drug.
110
VITAMIN D DEFICIENCY
Who is at risk:
- African Americans, elderly, institutionalized, hospitalized patients, obese.
- Vit D deficiency can precipitate osteopenia, osteoporosis, osteomalacia, muscle weakness
and increase risk of fracture.
Pathophysiology:
- We get vitamin D from the sun, food or supplements.
- Vitamin D metabolized in liver to 25-hydroxy vitamin D then metabolized by kidney to
active form, 1,25-hydroxy vitamin D.
Labs:
- 25-hydroxy vitamin D used to determine vitamin D status.
- <20 ng/mL = vitamin D deficiency
- Goal level 30-40 ng/mL
Treatment:
- 2 forms of vitamin D supplements: D3 and D2
- Vitamin D3 (cholecalciferol) preferred, more physiologic, more effective
o 2000IU daily x 12 weeks
o Repeat vitamin D level, if <20 ng/mL, refer to endocrine.
o If repeat 20-30 ng/mL, increase dose by 2000IU daily x 12 more weeks. Then
recheck, if <20ng/mL, refer to endocrine. If 20-30 ng/mL, continue increasing
by 2000IU daily until levels normalize.
o If 30-40 ng/mL, decrease to maintenance dose 1000IU daily. Dose depends
on body weight, ethnicity, sun exposure, co-morbidities.
- Vitamin D2 (ergocalciferol) is the only prescription vitamin D in US
o 50,000IU qweek (equivalent to D3 2000IU daily) x 8 weeks
o Then repeat vitamin D level check, if still <30 ng/mL, repeat 50,000IU qweek
x another 8 weeks.
o If 30-40 ng/mL, decrease to maintenance dose of 50,000IU q2-4weeks.
- Foods high in vitamin D: salmon, sardines, mackerel, tuna, cold liver oil, shiitake
mushrooms, vitamin D fortified milk/orange juice/yogurts/margarine/cheeses/cereals.
- Exposure to sunlight: 10-15 min of full body summer noon-day sun or artificial UVB
radiation (tanning beds) equals >10,000IU of vitamin D in light-skinned adults. Obvious
caution against risks of sun exposure.
111
HEME/ONC
HEMATOLOGY/ONCOLOGY
ANEMIA
Microcytic Normocytic Macrocytic
Iron deficiency Iron deficiency (early) B12/Folate
Thalassemia Acute blood loss ETOH
ACD (late)
Sideroblastic anemia
Anemia of Chronic Disease
Hypothyroidism
MDS
Liver disease
Copper deficiency CKD Hemolysis
Drugs(AZT,HU)
Terminology Description Associated disease states
Discocyte
A.k.a. biconcave
disc
Disc form of RBC with two
concentric concavities
Echinocyte (I-III)
A.k.a. burr cell,
crenated cell,
berry cell
Spiculated RBC with short
equally spaced projections
over entire surface;
progressing from the
crenated disc (I) to
crenated sphere (IV) with
near complete loss of
spicules
Uremia
Pyruvate kinase deficiency
Low K+ red cells
Immediately post-transfusion with aged or
metabolically depleted blood
Carcinoma of the stomach or bleeding
peptic ulcer
Acanthocyte
A.k.a. spur cell,
acanthoid cell
Irregularly spiculated RBC
with projections of varying
length and position
Alcoholic liver disease
Abetalipoproteinemia
Postsplenectomy
Malabsorptive state
Stomatocyte
A.k.a. mouth cell,
cup form,
mushroom cap,
uni-concave disc
Bowl-shaped RBC with
single concavity
progressing from shallow
bowl to near sphere with
small dimple (seen as
mouth form on peripheral
smear)
Hereditary stomatocytosis
Alcoholism
Cirrhosis
Obstructive liver disease
Erythrocyte sodium pump defect
Spherocyte
A.k.a. prelytic
sphere,microsphe
rocyte
Spherical RBC with dense
hemoglobi n content, EM
shows persistent minimal
dimple
Hereditary spherocytosis (cells actually
spheromastocytes)
Immune hemolytic anemia
Posttranfusion
Heinz body hemolytic anemia
Water dilution hemolysis
Fragmentation hemolysis
Schistocyte
A.k.a. helmet cell,
fragmented cell
Split RBC, often showing
half-disk shape with two or
three pointed extremities;
may be small irregular
fragment
Microangiopathic hemolytic anemia (TTP,
DIC, vasculitis, GN, renal graft rejection)
Heart-valve hemolysis (prosthetic or
pathologic valves)
Severe burns
March hemoglobinuria
Elliptocyte
A.k.a. ovalocyte
Oval to elongated ellipsoid
cell (with polarization of
hemoglobi n)
Hereditary elliptocytosis
Thalassemia Iron
deficiency
Myelophthisic anemias
Megaloblastic anemias
Drepanocyte
A.k.a. sickle cell
Cells containing
polymerized Hgb S showing
varying shapes from bipolar
speculated forms to holly
leaf and irregularly
speculated forms
Sickle-cell disorders (S, S trait, SC, SD, S-
thal, etc.)
Hemoglobinopathy
C-Harlem Hemoglobin Memphis/S
Codocyte
A.k.a. target cell
On EM these bell-shaped
cells assume a target
shape on dried films of
blood
Obstructive liver disease
Hemoglobinopathies (S, C)
Thalassemia
Iron deficiency
Postsplenectomy
LCAT deficiency
Dacrocyte
A.k.a. teardrop
cell, tennis racket
cell, poikilocyte
Cells with a single
elongated or pointed
extremity
Myelofibrosis with myeloid metaplasia
Myelophthisic anemias
Thalassemias
Leptocyte
Thi n cell, wafer
cell
Thi n, flat cell with
hemoglobi n at periphery
Thalassemia
Obstructive liver disease
(+/- iron deficiency)
112
DISSEMINATED INTRAVASCULAR COAGULATION
Excessive thrombin causes massive activation of coagulation leading to hemorrhage and
thrombosis. Underlying causes include sepsis, malignancy (AML M3, prostate cancer),
trauma (head injury, crush), liver disease, and pregnancy (abruption, amniotic fluid
embolism).
113
HYPERCOAGULABLE DISORDERS
Virchow's triad:
1. Alterations in blood flow
2. Endothelial damage
3. Hypercoagulable state
Common inherited:
1. Factor V Leiden
2. Prothrombin gene mutation
3. Protein S deficiency*
4. Protein C deficiency*
5. Antithrombin deficiency*
6. Dysfibrinogenemia
Common acquired:
1. Malignancy
2. Surgery/trauma/burns
3. Pregnancy
4. OCPs, HRT, Tamoxifen
5. Immobilization
6. CHF
7. Antiphospholipid antibodies
8. Myeloproliferative disorders and PNH
9. CKD, nephrotic syndrome
10. Hyperhomocysteinemia
11. HIT
If age >50 and no family history of thromboembolism it is unlikely they have antithrombin deficiency, protein C
deficiency or protein S deficiency
Thrombophilia workup: Effects of anticoagulant therapy and acute thrombosis
Hypercoagulable disorder
Acute
Confounding Factors
Heparin Coumadin therapy
for testing
Thrombosis therapy
Antithrombin (deficiency) Can be lowered* Lowered NC; Rarely increased
Antiphospholipid antibodies NC NC NC
Factor V Leiden NC NC NC
Factor VIII level
Acute phase reactant. Do not test while inflammation is still present.
Lupus anticoagulant NC Cannot measure False positives
possible
Protein C (deficiency) Can be lowered* NC Cannot measure
Protein S (deficiency) Can be lowered* NC Cannot measure
Prothrombin gene mutation NC NC NC
Acquired AT deficiency: neonatal period, pregnancy, liver disease, DIC, nephrotic syndrome, major surgery,
acute thrombosis, treatment with L-asparaginase, heparin, or estrogens
Acquired Protein C deficiency: neonatal period, liver disease, DIC, chemotherapy (CMF), inflammation,
acute thrombosis, treatment with warfarin or L-asparaginase
Acquired Protein S deficiency: neonatal period, pregnancy, liver disease, DIC, acute thrombosis,
treatment with warfarin, L-asparaginase, or estrogens
NC: not changed; CMF: cyclophosphami de, methotrexate, 5-fluorouracil.
* Results can be affected by acute thrombosis; it is most cost effective to avoid testing for these deficiencies during the
initial presentation. However, if plasma levels are well within the normal range at presentation, deficiency of these
proteins is essentially excluded. 2010 UpToDate

VENOUS THROMBOEMBOLISM PROPHYLAXIS


1. Pulmonary embolism: The major complication of venous thromboembolism (VTE)
accounts for approximately 5-10% of hospital deaths. Many VTEs are asymptomatic and
will go undiagnosed, but some will present as symptomatic DVT or pulmonary embolism.
Presenting symptoms of hypoxia, tachycardia, SOB, pleuritic/atypical chest pain. Large
PE can present with hypotension, acute right heart failure.
2. Risk factors: The most significant risk factors found in inpatients
- age > 50 yrs, Varicose Veins, CAD, Cancer, Ischemic stroke, DM2
3. Strategies to Prevent VTE:
- Mechanical:
- Elastic compression stockings (TEDS)
- Intermittent pneumatic compression devices: Also called SCDs or Venodynes,
have not been studied well in medical inpatients, but likely provide some
substantial benefit to those patients who have contraindications to anticoagulants.
Just as efficacicous when applied to upper extremities
- Pharmacological:
- Unfractionated heparin 5000 units SQ q8h
- Enoxaparin 40 mg SQ daily or Dalteparin 5000 units SQ daily (only in GFR > 30)
4. Role in Thrombocytopenia/Coagulopathy: There is no data at this time regarding
pharmacologic DVT prophylaxis and liver failure or thrombocytopenia, however patients
with INR > 2.0 or a platelet count below 50,000 can be considered to have
contraindications to pharmacologic prophylaxis.
References: Geerts WH et al, Chest 2001; 119:132S-175S.Kikura M et al, Arch Surg 2005; 140:1210-17.Samama MM et al. NEJM 1999;
341(11):793-800.Leizorovicz A et al, Lechler E et al, Haemostasis 1996; 26(Suppl 2):49-56. Kleber
FX et al, Am Heart J 2003; 145(4). Bergmann JF et alThromb Haemost 1996 76(4):529-34
114
THROMBOCYTOPENIA
1. Definition: platelet count less than 150k (normal in 2.5% of population)
2. Bleeding Risk: prolonged lab bleeding time <100k, surgical <50k, spontaneous <10-20k
Decreased production Destruction Sequestration
Viral infection (rubella, mumps,
varicella, parvovirus, HCV, EBV)
HIV direct megakaryocyte damage
Chemotherapy or XRT
Bone marrow aplasia or hypoplasia, EtOH
toxicity
ITP
TTP
DICantiphospholipid ab
HELLP
Drugs (heparin, quinine,
quinidine, valproate) Infection
(EBV, CMV, HIV) Physical
destruction (CP bypass)
Evans syndrome
Portal HTN
Cirrhosis
Splenomegaly
Hypothermia
3. General workup: cbc (isolated thrombocytopenia or other cell lines affected?), peripheral
smear (looking for schistocytes, immature forms, etc.), coags, DIC panel, chemistries to
eval for hemolysis (bilirubin, reticulocyte count, LDH, haptoglobin if smear equivocal), and
of course careful review of medications. Many medications are associated with
thrombocytopenia.
4. DVT prophylaxis in thrombocytopenia: In general, platelet count <50k is a relative
contraindication to SQ heparin, <30k is absolute. For patients at very high risk for VTE,
weigh the risks and benefits. Some may warrant prophylaxis no matter how low the platelet
count.
5. Heparin Induced Thrombocytopenia
- Type I:
- Transient, mild, asymptomatic drop in platelet count (rarely below 100k)
- Occurs during first 1-2 days of therapy.
- Resolves spontaneously and does not require suspension of drug.
- Type II (the real deal):
- Platelet count often falls below 50-60k or by 50% of baseline platelet count
- Generally occurs on day 10-14. Can be earlier in pts who have had heparin in the
last 3-4 months; late presentations can occur even after heparin discontinued
- Immune mediated disorder characterized by antibodies against heparin-pltF4
complexes leading to platelet activation.
- Thrombotic sequellae occur in 30-80% of people (arterial and venous)
- LMWH is less likely to cause HIT when used first line but has 100% cross-
reactivity with antibodies
- HIPA (Heparin Induced Platelet Aggregation) protocol
- If scheduled and ordered STAT, test is run M-F 8:30-16:30. Routine tests are run
M-F 11:00 a.m.
- Patient must be off of heparin therapy (including lines and flushes) for at least 18
hours.
- Specimen must be delivered within 1 hour of collection. Specimen cannot be
frozen, must be fresh... Must be in by 11:00 a.m.
115
BLOOD PRODUCTS
1. Packed Red Blood Cells (PRBCs): based on hemoglobin and risk factors
- Hgb < 7 gm/dL PRBC transfusion is indicated
- Hgb 7-10 gm/dL Consider transfusing patients who are high risk:
- Age >65, ACS, hypoxic respiratory failure
- Goal hgb 10 g/dL, hematocrit 30.
- Hgb > 10 gm/dL PRBC transfusion not indicated
- Each unit of PRBCs is 300mL and should raise the hemoglobin by about 1gm/dl and
hematocrit by 3-4 percentage points unless there is continued bleeding. Each unit of
PRBC also delivers about 200mg of iron, so iron studies in an anemia work-up should be
sent before transfusion.
- Consider volume status with each transfusion (ESRD, CHF patients). Each unit of blood
increases intravascular volume ~1L of crystalloid. Post-treat with lasix if needed, consider
slow transfusion to avoid flash pulmonary edema.
- Leukoreduced PRBCs are preferable in: chronically transfused patients, potential
transplant recipients, patients with previous transfusion reactions, and cytomegalovirus
(CMV) seronegative at-risk patients for whom seronegative components are not
available.
2. Platelets: transfusion threshold varies based on risk of bleeding
- Keep platelets > 10,000 if: low risk of bleeding, ecchymoses/petechiae as only
manifestation of thrombocytopenia, acute leukemia, hematopoietic cell transplantation,
solid tumors
- Keep platelets > 20,000 for minor bleeding (eg. mucosal bleeding) and for bone marrow
biopsy
- Keep platelets > 50,000 for most invasive surgical procedures (as long as there arent
any other coagulation abnormalities) or if actively bleeding
- In general, one random donor platelet concentrate is expected to increase the platelet
count by 5000 to 10,000/uL in a 70 kg patient who is not refractory. Refractoriness is
roughly equivalent to an absolute platelet count increment of <2000/microL per unit of
platelet concentrate given to an average-sized adult.
3. Fresh Frozen Plasma: contains coagulation factors
- Indications: coagulopathy in warfarin overdose, vitamin K deficiency, liver failure, TTP-
HUS (plasmapheresis). FFP should not be used as primary therapy for a specific
coagulation defect (eg, hemophilia A, hemophilia B, factor VII or XIII deficiency) when
specific coagulation factor concentrates are available.
- Each bag is about 250mL. Initial dose should be 15mL/kg.
4. Cryoprecipitate: contains factor VIII, factor XIIII, fibrinogen, fibronectin, and von
Willebrand factor
- Indications: Low fibrinogen states (eg. DIC), bleeding in von Willebrand disease, factor
VIII deficiency, factor XIII deficiency
- Each bag is 10-15mL. It is usually ordered as 10 bags, which contain 2 gm of fibrinogen
and will raise the fibrinogen level about 70 mg/dL in a 70 kg recipient.
*Nomenclature of red cell shapes and associated disease states.Williams, Beutler, Rundles, et.al. Hematology.
For images: http://www.som.tulane.edu/classware/pathology/Krause/AbnormalRBC/AbnormalRBC.html
DEEP VEINTHROMBOSIS (ADAPTED FROMUP TO DATE)
Pathogenesis: Invol ves three factors known as Virchow's triad: damage to vessel wall (prevents
the endothelium from inhibiting anticoagulation and initiating local fibrinolysis), venous stasis
(inhibits clearance of activated clotting factors), and hypercoagulability.
Risk factors: History of immobilization or prolonged hospitalization/bed rest, recent surgery,
obesity, prior episode(s) of venous thromboembolism, lower extremity trauma, malignancy, oral
contraceptives or hormone replacement therapy, pregnancy or postpartum status, stroke
Diagnosis: classic symptoms of DVT are pain, unilateral swelling, discoloration of leg. Because
these symptoms are relatively nonspecific, the Wells criteria were developed to aid in the
diagnosis via assessment of the PRE-TEST probability that a DVT is present.
116
Wells criteria: (patients get 1 point for having each of the following)
History of active cancer (within the last six months)
Paralysis, paresis, or recent plaster immoblization of lower extremity
Bedridden for > 3 days, or has has major surgery within the last 4 months
Localized tenderness along distribution of venous system
Entire leg is swollen
Asymmetric swelling of calf of > 3 cm (measured below tibial tuberosity)
Pitting edema greater in symptomatic leg
Presence of collateral superficial veins
Then SUBTRACT TWO POINTS IF THERE IS AN ALTERNATIVE DIAGNOSIS MORE
LIKELY THAT DVT AS ETIOLOGY OF SYMPTOMS
High probability: 3 or more points
Moderate probability: 1-2 points
Low probability: 0 points
If patient is LOW PROBABILITY - check D-dimer, if -, no further testing indicated
If patient is MODERATE PROBABILITY - check D-Dimer. If -, unlikely patient has
DVT. If +, get ultrasound
If patient is HIGH PROBABILITY - no need to check D-Dimer, proceed with ultrasound
Treatment: Treatment is indicated for all patients with proximal deep vein thrombosis. It is
estimated that >50% of patients with symptomatic proximal DVT develop PE
Initial anticoagulation: Unless patient has contraindication to anticoagulation (hemorrhagic
CVA or other life threatening bleed, patients can be treated with low molecular weight
heparin (LMWH), fondaparinux, or unfractionated heparin (either as continuous IV infusion
or adjusted-dose given subcutaneously).
Oral anticoagulation: For most patients, warfarin should be started at the same time to allow
the level to become therapeutic (an INR 2-3) over fi ve days. Warfarin inhibits the vitamin K
dependent clotting factors, including protein C. However, levels of protein C fall before the
other clotting factors, causing patients to become transiently hypercoagulable when starting
warfarin.
Inferior vena caval filter placement is recommended if patient has contraindication to
anticoagulation, a massive PE causing hemodynamic instability (indicating that the patient
would not tolerate further emoblization from the deep venous system into the pulmonary
vasculature), or massive iliofemoral thrombosis.
Fibrinolytics: activate plasminogen to form plasmin, resulting in clot lysis. Not generally used
in isolated deep vein thrombosis unless DVT is limb or life threatening. Used in massive PE
if patient is hemodynamically unstable or in cardiogenic shock.
Duration of treatment: duration of anticoagulation varies by clinical setting and patient
preferences.
First DVT in setting of obvious precipitant (i.e. surgery): three months of treatment
First distal (vein below knee) DVT and unknown precipitant: three months of treatment
First proximal (vein above knee) DVT and unknown precipitant: six months of treatment, can
consider longer therapy if bleeding risk is small
Patients with active malignancy: consider life long anticoagluation, optimally with LMWH, but
warfarin can be used if LMWH not available.
Recurrent DVT: consider lifelong anticoagulation.
Complication of heparin therapy: bleeding and heparin-induced thrombocyptopenia (HIT):
HIT type I: non immune mediated, platelets fall within three days after initiation of heparin.
No further treatment required.
HIT type II: immune medicated disorder in which antibodies form against the heparin-
platelet factor 4 complex. Platelets fall > 50% 5-10 days after exposure to heparin. Typical
nadir is 60,000.
Heparin MUST BE STOPPED and direct thrombin inhibitors (i.e. argatroban, fondaparinux,
bivalirudin) started. Although HIT is a clinical diagnosed based on a drop in platelet count by
50% within 5-10 days of exposure to heparin, it is a hypercoagulable disorder characterized
by both arterial and venous thrombosis. Diagnosis should be made and treatment started
(cessation of heparin and initiation of direct thrombin inhibitors) before confirmation studies
are back due risk of venous and arterial clots. HIT with thrombosis (HIT-T) carries a 30-50%
mortality.
117
CHEMOTHERAPY&COMPLICATIONS
1. Tumor Lysis Syndrome (adapted fromUp to Date)
Definition: oncologic emergency that is caused by massive tumor cell lysis with the release of
large amounts of potassium, phosphate, and nucleic acids into the systemic circulation
Who is at risk?
- Patients with bulky disease, mainly leukemia (especially ALL, sometimes AML) or poorly
differentiated/high-grade lymphomas
- Patients treated with cytotoxic chemotherapy, though steroids may be sufficient to
precipitate
- Particularly high risk if present with elevated lactate dehydrogenase (LDH), uric acid, or
creatinine levels
Pathogenesis: Initiation of chemotherapy in patients with a large tumor burden or highly
proliferating malignancies leads to rapid lysis of tumor cells. Massive quantities of intracellular
contents are released, leading to hyperkalemia, hyperphosphotemia, hypocalcemia (binds
phosphate in circulation and precipitates out), and hyperuricemia (from the cataboism of nucleic
acids). Both uric acid deposition and calcium phosphate deposition in the renal tubules can lead
to renal failure.
Clinical manifestations: related to underlying metabolic abnormality such as hyperkalemia,
hyperphosphatemia, hypocalcemia or due to flank pain due to uric acid nephrolithiasis - nausea,
vomiting, diarrhea, anorexia, lethargy, hematuria, heart failure, cardiac dysrhythmias, seizures,
muscle cramps, tetany, syncope, and possible sudden death
Medical treatment options
IV fluid: In absence of contraindication (i.e. heart failure), patients should receive 2-3L per
day with goal urine output 80 to 100 cc/hour
Sodium bicarbonate: the role of urinary alkalinization is unclear, only clear role is in patients
with metabolic acidosis.
Allopurinol: blocks catabolism of hypoxanthine and xanthine into uric acid.
Rasburicase: lowers uric acid levels by catalyzing oxidation of uric acid to the much more
water-soluble compound allantoin
Risk stratification with prophylactic treatment recommendations:
High Risk features: all patients with Burkitt NHL or Burkitt ALL, ALL with WBC > 100,000 or
AML with WBC > 50,000 should receive aggressive IV hydration and prophylactic
rasburicase instead of allopurinol if available PRIOR to initiation of treatment
Medium Risk features: patients with diffuse large B cell lymphoma, non B-ALL with WBC
50,000 to 100,000, AML with WBC between 10,000 to 50,000, patients with CLL receiving
fludarabine with WBC between 10,000 and 50,000 should receive IV fluid resusitation and
prophylactic allopurinol when pre treatment uric acid level is normal. If pre treatment uric
acid level are elevated, can consider using one time rasburicase in addition to allopurinol
Low Risk features: patients with indolent NHL, ALL with WBC < 50,000, AML or CLL with
WBC < 10,000, all patients with solid tumors. Prophylaxis with IV hydration in generally
sufficient prophylaxis
118
2. Neutropenic Fever:
Definition
Fever: single oral T >38.3 (101.3) or sustained T >38.0 (100.4) for over 1 hour
Neutropenia: ANC <500, or ANC 500-1000 with predicted decline within 48 hours
Etiology
Similar incidence of gram negative and gram positive infection; therapy targeted to
gram negative infection, especially Pseudomonas, due to faster rate of
decompensation and death
Candida and Aspergillus most common fungal infections; become more prevalent
with increasing duration of neutropenia
No infection found in 50-70% of cases; 10-20% bacteremia, 20-30% clinically
evident infection
Initial Evaluation and Management
Physical exam looking carefully for catheter site infections, mucositis
Panculture including any lines and start empiric antibiotics immediately
Empiric antibiotics - IDSA guidelines:
Monotherapy: Appropriate for uncomplicated neutropenic fever
Ceftazidime 2g IV q8h, cefepime 2g IV q8h, imipenem-cilastatin 500mg IV
q6h, or meropenem 1g IV q8h (for patients with normal renal function)
Combination therapy: Indicated in severe sepsis or septic shock or high prevalence of
multi- drug resistant gram negative rods, Includes any of the above plus aminoglycoside
or ciprofloxacin (if low enough prevalence of FQ resistance)
Vancomycin should not be part of empiric regimen unless (1) evidence of catheter or
soft tissue infection,(2) known colonization with MRSA or penicillin-resistant S.
pneumoniae, (3) severe mucositis, or (4) severe sepsis/septic shock.
Oral regimen consisting of amoxicillin/clavulanate and ciprofloxacin may be
appropriate for certain very low risk patients
119
C
o
m
m
o
n
C
h
e
m
o
t
h
e
r
a
p
i
e
s
120
NEUROLOGY
ALTERED MENTAL STATUS
- Investigate: circumstances and vitals
- SAFETY FIRST: patients are often confused and agitated so keep yourself between the
patient and the door and ask for assistance if any doubts.
- Things not to miss: meningitis, ICH or intracranial mass, delirium tremens, untreated
infection.
- Obtain Medication List: (the elderly are especially affected); look for medications causing
delirium (Anticholinergics, Benzos, Narcotics).
- Workup: FS glucose, Chem 10 (Ca, Na, BUN), CBC, TSH, NH3 if suspect hepatic, LFTs,
UA, Blood and Urine Cx. Consider ABG, EKG, non-con CT Head if h/o trauma or focal
neuro exam, LP if suspect meningitis.
- PE: pupils, neuro exam for focality, asterixis, fundoscopic exam, Kernigs/Brudinskis for
Meningitis, Battles/Raccoons eyes for basilar skull fracture (fall/trauma), skin for
petechiae.
- Once patient is stable consider Differential: MOVE STUPID
- Metabolic: hepatic encephalopathy, B12/thiamine deficiency, niacin deficiency
(pellagra), Wilsons disease, electrolyte abnormalities
- Oxygen: hypoxia, hypercarbia, anemia, sepsis, CO poisoning
- Vascular: CVA, hemorrhage (intracranial, subdural, epidural, subarachnoid),
vasculitis, TTP
- Endocrine: hyper/hypoglycemia, hyper/hypothyroid, high/low cortisol, HONK/DKA
- Seizures: post-ictal confusion (ie. Todds paralysis), status epilepticus
(nonconvuslive), complex partial
- Structural: mass effect, hydrocephalus (normal pressure hydrocephalus &
pseudotumor cerebri)
- Tumor, Trauma, Temperature
- Uremia
- Psych: sundowning and ICU psychosis are dxs of exclusion. Porphyria (rare)
- Infection
- Drugs: intoxication/withdrawal, neuroleptic malignant syndrome, serotonin
syndrome
- Intervention: Depends on cause (Benzos for withdrawal from ETOH, lactulose for hepatic
encephalopathy, dialysis for uremia, etc),
- Medications: Haldol often used 2-5 mg IM/IV as it causes minimal respiratory
depression-but beware of QT prolongation, sudden death (very rare). For elderly and
liver patients avoid Benzos. Risperdal* is useful in the elderly (dissolvable form: M-tab).
Benzos and Narcotics may worsen delirium.
*Black Box Warning does exist with regard to this use, though it still is commonly used with good efficacy
121
SEIZURES AND STATUS EPILEPTICUS
If you are concerned for status, get the Ativan (or your benzo of choice), and call your Resident
and Neuro ASAP.
- Definition: the occurrence of a single unremitting seizure >5 to 30 minutes or frequent
clinical seizures without an interictal return to the baseline clinical state.
- Risk factors: head injury, stroke, Alzheimer's disease, history of intracranial infection, and
alcohol or drug abuse, electrolyte perturbation.
- PE: Assess ABCs, call anesthesia to intubate if necessary; Labs: Chem, FSBS
- Initial Intervention: Lorazepam 2mg iv, Midazolam 5mg iv, or Diazepam 10mg iv
- Follow Algorithm: if seizures are refractory to initial boluses of benzodiazepines, follow
the Status Algorithm for further evaluation and treatment
Status Epilepeticus Algorithm:
122
STROKE CODE
- If you have high suspicion for CVA, call operator and call a Stroke Code. This will alert
the Stroke team to mobilize.
- Follow the Algorithm.
Reference: American Heart Association. 2005 American Heart Association Guidelines for Cardiopulmonary Resuscitation and Emergency
Cardiovascular Care Part 9: Adult Stroke. Circulation 2005; 112(24 Suppl): IV-111-IV-120.
123
PALLIATIVE CARE
CROSSCOVER TIPS
CONSTIPATION
1. Goal: Move the bowel
2. Etiology: Important to determine. Obtain a KUB to check for signs of obstruction,
which may sometimes reveal impaction of bowel with stool. Always keep the
possibility of ischemic colitis in the back of your mind. Check for use of narcotics &
check rectal exam.
3. Orders: KUB, and consider a CT scan. Decide if you want to move the bowel from
aboveor below. Starting from below first in severely constipated patients is probably
safest, then transition to oral laxati ves after a BM. Miralax is a GI favorite. Use caution
with fleet enemas and MOM in elderly or patients with renal insufficency. Lactulose
enemas work well but are messy. Senna is helpful in waking up sleepy bowels from
narcotics. Methylnaltrexone can be used in a single dose with repeat if patient is on
high dose narcotics. Ask nurses to carefully record stools.
- From Below: Dulcolax suppository, enemas (Fleets vs tap water vs mineral oil)
- From Above: MOM 30ml QID, MiraLax 17g in 8oz liquid qd to bid, Mag citrate,
Lactulose
- Bowel Stimulation: Senna 1-2 BID, oral dulcolax bid, Neostigmine (severe refractory)
- Stool Softener: Docusate 250 mg BID
- Gastroparesis: reglan, erythromycin (beware of QTc prolongation) (motilin agonist)
4. Keep An Open Mind: An insidious but potentially morbid process is always possible.
- Ogilvie's Acute Colonic Pseudoobstruction: these patient are at risk of
perforation because of acute dilation (often >10-15cm) of colon.
- Volvulus: Gastrograffin enema is both diagnostic and therapeutic
- Toxic Megacolon: part of the spectrum of C.diff (can also be seen in Hirschsprungs)
and can present without antecedant diarrhea.
INSOMNIA
* Beware of Benzos and Anticholinergics. Better to have a grumpy patient then one on the
vent from aspiration, hypercarbic respiratory failure.
* Care with oversedation especially in COPD or ESLD patients.
1. Medications:
- Neuroleptics: Risperdal 0.5mg, Seroquel 25mg, Haldol 0.5mg: better for insomnia
that is associated with delirium, not first line for the average patient.
- Zolpidem5-10mg: decreased rebound, tolerance, dependence compared to Benzos.
- Trazodone 50mg: good for elderly, okay in liver patients.
- Benzodiazepines: Temazepam15-30mg; beware of paradoxical response; can
cause delirium; potential for dependence; avoid in liver patients
- Diphenhydramine 25mg: beware of anticholinergic effects
NAUSEA
1. First line:
- Ondansetron (Zofran) 4-8 mg IV or PO q6 prn. Serotonin receptor antagonist
- Prochlorperazine (Compazine) 10 mg PO/IM/IV q6h or 25 mg PR q12h PRN. Side
effects: Extrapyramidal symptoms, seizures (CI), QT prolongation,
hyper/hypoglycemia, sleep disturbances, anticholinergic.
- Promethazine (Phenergan) 12.5-25 mg PO/PR/IM/IV q4-6h, SE: sedation, seizures,
hallucinations, respiratory depression, & anticholinergic. Do not give with narcotics.
- Lorazepam(Ati van) 0.5-2.0 mg PO/IV q4-6h prn.
- Metoclopramide (Reglan) 10 mg PO/IV q4-6h prn; good for gastroparesis (DMII),
SE: EPS, dystonia, tardive dyskinesia, fluid retention, AV block, insomnia/anxiety
- Dexamethasone (Decadron) 10 mg PO/IV q6h prn.
2. Second line:
- Dronabinol (Marinol) 2.5-10 mg po q6h
3. General tips:
- Beware neuroleptic malignant syndrome or dystonic reaction with excessive
use of compazine or droperidol (treat with benadryl 50 mg IV/IM or Cogentin 10
mg IV)
- Compazine and Marinol are relatively contraindicated in patients with seizure disorder.
- A common side-effect of many anti-emetics is drowsiness.
- Happy Bag: continuous gtt of famotidine, zofran, diphenhydramine, haldol, and ativan
124
PREVENTATIVE MEDICINE/EBM/STATS
CARDIOVASCULAR RISK MODIFICATION
1. Aspirin Therapy: 81mg qd, higher doses are associated w/ increased side effects and
little, if any, increased benefit.
- Primary Prevention: Recommended in patients who have a 10 year risk of CHD >6-
10%. The ADA recommends that clinicians consider aspirin in diabetic patients >30
years old or in those who have risk factors for CVD and no contraindications to aspirin
therapy.
- Secondary Prevention: Should be given to all patients with CHD/CHD risk equivalents
who do not have contraindications.
2. Diabetes mellitus: While A1c remains the best test for assessing management of
diabetes, it remains to be added as an accepted screening test.
- Fasting Glucose: ADA recommends fasting glucose q3yrs, starting at age 45.
- Early Screening: Consider initiation of screening at an earlier age if the patient has risk
factors for diabetes present (ie, obesity, hyperlipidemia, family history of DM, etc).
3. Dyslipidemia:
- Lipid Panel: every other year in men >35yrs or women >45yrs
- Yearly screening: Consider if any of the following are present:
- Diabetes
- Family history of early CAD (<50 in males; <60 in females)
- A family history suggestive of familial hyperlipidemia
- Multiple CAD risk factors (eg, tobacco use, hypertension) then start at age 20 for
men and women
4. Abdominal Aortic Aneurysm:
- Abdominal Imaging (usually AAA Screen Ultrasound) one time for all men who have
smoked (>100 cigarettes: 5 packs)
PSYCHOSOCIAL SCREENING
1. Alcohol Abuse:
- CAGE: To screen for at risk drinking
- Have you ever felt you should Cut down on your drinking?
- Have people Annoyed you by criticizing your drinking?
- Have you ever felt bad or Guilty about your drinking?
- Have you ever had a drink first thing in the Early morning to steady your nerves or
get rid of a hangover?
- A positive response to two or more questions suggests a high likelihood of alcohol
abuse.
2. Depression Screening:
- During the past month have you often been bothered by feeling down, depressed or
hopeless?
AND
- During the past month have you often been bothered by little interest or pleasure in doing
things?
- 97% sensitivity for depression when both positive
Reference: BMJ Nov 2003
125
CANCER SCREENING
1. Breast Cancer:
- Self-breast exams: at age 20
- Annual CBE: Clinical Breast exams after age 20 (10% of screening detected breast
cancer caught by exam despite negative mammogram).
- Annual Mammogram: starting at age 40.
- Early Screeing: At age 35 if family h/o breast cancer earlier than age 50 or 10yrs prior
to the age of diagnosis of breast cancer in first degree family member.
- Discontinuation: Some evidence supports stopping screening after age 74, however,
most expert groups recommend continued screening if patient has >10 year life
expectancy.
2. Cervical Cancer:
- Annual PAP smear starting 3 years after onset of sexual activity or at age 21, whichever
comes first (ACOG). This is based on the fact that it takes 3-5yrs after HPV infection to
develop dysplastic changes.
- Intervals: If 3 consecutive normal PAP smears AND patient has no risk factors for
cervical cancer (ie not sexually active or in a monogamous relationship, no DES
exposure, no history of abnormal pap, etc) PAP every 1-3yrs.
3. Colorectal Cancer:
- Screening Initiation: For average risk patients >50 years old.
- Annual fecal occult blood test FOBT(stool cards) WITH either Flexible sigmoidoscopy
every 5yrs, starting at age 50 (Not available at UCSD) OR double contrast barium
enema every 5yrs, starting at age 50
- Colonoscopy every 10yrs, starting at age 50
Increased risk patients:
- Includes adenomatous polyps, IBD, genetic syndromes (ie. FAP and HNPCC), prior CRC
and family history of CRC in first degree relative.
- Family history: if 1
st
degree relative developed CRC or adenoma >60 yo, then begin
colonoscopy at age 40, repeat q10yrs, if 1
st
degree relative developed CRC or
adenoma<60 yo or two 1
st
degree relatives with disease at any age, then begin
colonoscopy at 10 yrs before earliest age of diagnosis OR at 40 years of age,
whichever comes first, repeat q5yrs.
- Prior CRC: q3-5yrs
- Prior adenomatous polyp:
- In 3-6 months if large sessile adenoma (>2cm) and/or if poor prep
- In 3 years If adenoma >1cm or >2 adenomas found or adenoma with villous
histology or high grade dysplasia.
- In 5 years If 1-2 small adenomas (<1cm)
4. Prostate Cancer: Current guidelines recommend discussing the role of prostate screening
in asymptomatic men with them, but do not advocate any specific screening modality.
- Consider annual DRE, starting at age 40 if African-American (there is an increased
incidence of prostate cancer in this group); age 50 for others. No controlled studies have
shown a reduction in the morbidity or mortality of prostate cancer when detected by DRE
at any age.
- Consider annual PSA, starting at age 40 if African-American; age 50 for others.
- Start screening at age 40 if the patient has 1
st
degree relative w/history of prostate
cancer.
- African-Americans have a 60%higher incidence rate of prostate cancer; screening
should be discussed earlier in this population
- If PSA >7, refer for biopsy, if 4-7, repeat lab within few weeks and have patient refrain
from ejaculating and bike riding 48hrs before test.
- If PSA <4.0, but rise of 0.75 ng/ml/year based on 3 measurements over 1-2yr period
then refer for biopsy.
- Usually stop screening at 75 yo or when patient has <10yr life expectancy.
5. Testicular Cancer: The U.S. Preventive Services Task Force (USPSTF) recommends
against routine screening for testicular cancer in asymptomatic adolescent and adult
males.
126
PRIMARY PREVENTION
1. Osteoporosis:
- Intiation: Bone densitometry screening starting at age 65yo or <65 yrs and
postmenopausal and at increased risk of fracture (i.e. wt <127lbs, smoking, fracture in 1
st
degree relative, ETOH, immobility. Estrogen deficiency--> menopause < 45yo, BSO.
- Medications Associated with Bone Loss: Synthroid, Seizure meds, Steroids
- DEXA Scan: T score = the number of standard deviations (SD) above/below
- the mean in young adults of same sex and race.
- Osteopenia: T score of -1.0 to -2.5
- Osteoporosis: T score of > - 2.5
- Calcium: premenopausal: 1000mg daily, postmenopausal: 1500mg daily
- Vitamin D: 400-800 IU Daily
- Bisphosphonates: Indicated for osteoporosis
2. Vaccinations:
- Influenza:
- Annually for adults with chronic medical problems
- Annually for healthcare workers
- As desired for healthy adults
- Pneumovax:
- Once in any adult with chronic medical problems. Single booster should be given
at age 65 if patient received 1
st
dose >5 yrs previously.
- Once in any immunocompromised adult (ie. HIV, malignancy, CRF, asplenia,
chemotherapy, post organ/bone marrow transplant). Single booster after >5 yrs
- All adults > 65 years old.
- Tetanus: Booster should be given every 10 years.
- Hepatitis A:
- All children at age 1 year
- Children/adolescents ages 2-19 who live in states where mandated
- Persons traveling to or working in countries with high or intermediate prevalence
- Men who have sex with men
- Users of illegal injection and noninjection drugs
- Persons who have occupational risk for infection
- Persons who have chronic liver disease
- Persons who have clotting-factor disorders
- Hepatitis B:
- Age 19-39: Universal immunization
- Age 40 and over: Immunize high risk patients (MSMs, correctional facilities, IVDU,
HIV/AIDS, those exposed to high-risk patients)
- Meningococcus:
- Routine vaccination of young adolescents or prior to high school entry
- College freshman, military recruits, travelers to areas in which disease is
hyperendemic or epidemic, health-care workers exposed to Neisseria
- Functional or anatomic asplenia or terminal complement deficiency
- Varicella:
- Age 19-26: Verify second dose completed.
- Age 27 and over: For all adults without evidence of immunity to varicella, give two
doses of varicella vaccine with at least 28 days between first and second doses.
- HPV:
- Females age 11-12 (as young as 9); catch up vaccines for females age 13-26
- Consider MSMs
- NOT a substitute for routine cervical cancer screening
- Shingles: Immunize age 60 and over (single dose) irrespective of whether patient
reports a prior episode of herpes zoster.
Grading of Recommendations:
1 = strong recommendation (benefits
clearly outweigh costs)
2 = weak recommendation (benefits
probably outweigh costs, but unsure)
A D = high grade of evidence very
low grade of evidence
127
EVIDENCE BASED MEDICINE & STATISTICS: THE 5 AS
1. Assess: Information you need to take better care of your patient.
2. Ask: Focus in your information search by increasing the words in your query:
- P: Word or phrase for patient or problem
- I: Word for intervention youre considering
- C: Word for comparison intervention
- O: Word for outcome important to you/pt
Example: Will metoprolol, which is more affordable, provide as much mortality benefit as carvedilol?
P = heart failure or CHF; I = metoprolol; C = carvedilol; O = mortality
3. Acquire: Limit Medline using EBM criteria such as RCT, meta-analysis, or the extra limits
in Medlines Clinical Queries. Consider using pre-appraised information:
- ACP Journal Club (OCL) - Cochrane Library (OCL)
- ACPs PIER (thru STAT!Ref on OCL) - National Guideline Clearinghouse (OCL)
- Clinical Evidence (free thru unitedhealthfoundation.org)
4. Appraise: Assess the articles validity before proceeding to the results.
- Validity: one method to assess the validity of an article/article type can be found in the
Users Guides at the OCL.
- Results: methods to assess the results are also available via the Users Guides and
Center for EBM (OCL). For the common article types:
Appraising Treatments:
- EER = the experimental event rate (512/1511 carvedilol patients died = 34%)
- CER = the control event rate (600/1518 metoprolol patients died = 40%)
- Statistical significance: p < 0.05 or CI not crossing null effect
- RR = relative risk = EER/CER = 85%
(risk of death on carvedilol is 85% what it would have been on metoprolol)
- RRR = relative risk reduction = 100 RR = 15%
(risk of death on carvedilol is 15% LESS THAN it would have been on metoprolol)
- ARR = absolute risk reduction = CER EER = 6%
(proportion of population who will change outcome w/changing from metoprolol to carvedilol)
- NNT = number needed to treat = 100/ARR = 17
(number of patients treated with carvedilol instead of metoprolol to save one life)
Appraising Diagnostic Tests:
Sensitivity: true +/all with dz = 167/197 = 85%
- High Sensitivity: positively identifies most case but not just true cases, in other words, not
specific to disease; thus high false positive, but low false negative
- Mnemonic: snout - high sensitivity and neg test rules out dz
Specificity: true -/all without dz = 670/980 = 68%
- High Specifity: positively (and correctly) identifies true cases, but misses some due to
lack of sensitivity; thus low false positive, but high false negative
- Mnemonic: spin - high specificity and pos test rules in dz
LR Likelihood Ratio: likelihood ratio (with 1 being no increased likelihood) that you have
disease for a given test result.
- LR = proportion of pts w/ dz who came up + = 167/197 = sensitivity = 2.7
proportion of pts w/o dz who came up + 310/980 1 spec
- LR can be entered into a nomogram at OCL to obtain a post-test probability of disease.
5. Apply: your results to affect patient care
PE
w/ w/o
d-dimerr
+
_
167 310
30 670
1977 980
47
70
1177
*Online Clinical Library (OCL) available
thru Users Guides or Center for EBM
links on UCSD computers
128
MEDICAL SPANISH: THE BASICS
This guide is designed to get you through the basic admission work-up and daily pre-
rounding. Since one of the hardest things about speaking Spanish to patients is
understanding what they say back to you, most of what is included is in the form of yes/no
questions.
There are Two Basic Verbs for which you should learn conjugations:
1. Ser: to be, meaning a permanent state of being
- Yo soy (I am); Tu eres (you are, informal); Usted es (you are, polite); Nosotros somos
(we are); Vosotros sois (you are, plural, rarely used except in Spain); Ustedes son (you
are, plural)
2. Estar: meaning a temporary state of being
- Yo estoy (I am); Tue estas (you are, informal); Usted esta (you are, polite); Nosotros
estamos (we are); Vosotros estais (you are, pleural, again never use this); Ustedes estan
(you are, plural)
*Often you can just use the verb without the indicator in sentences. Examples:
- Yo soy Doctor Smith (I am Doctor Smith, I will ALWAYS be Doctor Smith)
- Donde estas? (Where are you at THIS moment?).
3. Greetings and formalities:
- Buenos dias (good morning, good day), buenas tardes (good afternoon), buenas noches
(good evening)
- Hola (hello)
- Yo soy doctor (doctora) _______. - I am Doctor _______.
- Como estas? - How are you?
- Como te llamas? - What is your name?
- Me llamo ______. - My name is _______.
- Gracias - thank you
- De nada - you are welcome
- Chau/adios - bye
4. HPI:
- Que problema tienes hoy? - What problem are you having today?
- Tienes dolor? OR Estas con dolor? (Both mean do you have pain?)
- dolor del pecho (chest pain)
- falta de aire (shortness of breath)
- fiebre o escalofrios (fevers or chills)
- dolor de la cabeza (headache)
- nausea
- vomitos (vomiting) con sangre? (with blood?)
- dolor del estomago/abdomen (abdominal pain)
- cambios en la vista (visual changes)
- dolor de la garganta (sore throat)
- dolor al orinar (dysuria)
- dificultad de hablar/tragar (difficulty speaking/swallowing)
- tos (cough) - con sangre? (with blood?)
- diarrea (diarrhea) - con sangre? (with blood?)
- sangre en la orina - hematuria
- sangre en el excremento - bloody stool
- debilidad de un brazo o de una pierna (weakness in an arm or leg)
- izquierdo/a (left)
- derecho/a (right)
129
5. PMH/FH/SH:
- Tienes algun problema medico? (Do you have any medical problems?)
- Tienes ______ (do you have ______?)
- diabetes
- presion alta (hypertension)
- ataque del corazon OR coronaria OR infarto
- cardiaco (MI)
- enfermedades cardiacas (heart disease)
- SIDA/VIH (AIDS/HIV)
- Tuberculosis
- Cancer (you can ask donde? for them to indicate where they have cancer)
- Asma (asthma)
- Enfermedades pulmonares (lung disease)
- Problemas del higado/del pancreas (liver/pancreatic problems)
- Hay historia en tu familia de ______ (do you have a family history of ______?)
- Quien (who?): papa (father), mama (mother), abuelo (grandfather), abuela
(grandmother), hijo (son), hija (daughter), tio/a (uncle/aunt), primo/a (cousin),
- nieto/a (grandson/daughter), hermano/a (brother/sister); Note: as a general
- rule with many exceptions masculine words end with o and feminine words
- with a.
- Tomas alcohol? (Do you drink alcohol?); Tomas drogas? (do you use drugs?); if
yes, con agujas o sin agujas? (with or without needles if patient answers yes to
drug use); fumas cigarillos? (do you smoke cigarettes?); donde vives (where do
you live?); con quien? (with whom); has viajado recien? (have you traveled
recently?); para donde? (to where?); has estado en la carcel? (have you been in
jail?); tienes trabajo? (do you work?); que tipo de trabajo haces? (what kind of
work do you do?)
6. Allergies: tienes alergia a medicamentos o comidas? (do you have a medication or food
allergy?); a cuales (to which ones?)
7. Medications: que medicamentos tomas? (what medications do you take?); tienes una
lista de los medicamentos? (do you have a list of your medications?)
8. Physical Exam:
- Te voy a examinar ahora, OK? (I am going to examine you now, OK?)
- Levantate (get up)
- Acuestate (lie down)
- Sientate (sit down)
- Abra la boca y saque la lengua (open your mouth and stick out your tongue)
- Por favor (please)
- Te duele? (Does it hurt?)
9. AM rounds:
- Como te sientes hoy? (How do you feel today?)
- Tuviste algun problema por la noche? (Did you have any problems overnight?)
- Tuviste ____ (did you have ____?, see above list for items to fill the blank.)
- Te sientes mejor/peor/igual? (Do you feel better/worse/the same?)
- Necesitas algo? (Do you need anything?)
- Comida/bebida (food/drink)
130
CONVERSIONS &TABLES
CORTICOSTEROID CONVERSION TABLE
Drug Equivalent
dose (mg)
Anti-
inflammatory
potency
Mineralo-
corticoid
potency
Biologic
Half-life
(hours)
Cortisone 25 0.8 2 8-12
Hydrocortisone* 20 1 2 8-12
Prednisone 5 4 1 18-36
Prednisolone 5 4 1 18-36
Methylprednisolone 4 5 0 18-36
Triamcinolone 4 5 0 18-36
Dexamethasone 0.75 20-30 0 36-54
Betamethasone 0.6 20-30 0 36-54
Fludrocortisone Not available 10 125 18-36
*roughly equivalent to endogenous Cortisol
INTRAVENOUS FLUIDS
Osmolality
(mOsm/kg)
Glucose
(g/L)
Sodium
(mmol/L)
Chloride
(mmol/L)
Other
D5W 278 50 0 0
D10W 556 100 0 0
D50W 2778 500 0 0
NS 154 -- 77 77
NS 308 -- 154 154
3%NS 1026 -- 513 513
LR 274 -- 130 109
K
+
4mmol/L
Ca
++
1.5mmol/L
Lactate
28mmol/L
NARCOTIC & OPIOID CONVERSION
1. Commonly Used Orals: Morphine Equivalence:
Tylenol #3
Acetaminophen 325mg
+
Codeine 30mg
Vicodin
Acetaminophen 500mg
+
Hydrocodone 5mg
Percocet
Acetaminophen 325mg
+
Oxycodone 5mg
Morphine 3-4mg PO Morphine 5-6mg PO Morphine 7-8mg PO
2. Opioid Conversion Table:
Oral/Rectal Dose (mg) Analgesic IV/SC/IM Dose (mg)
150 Meperidine 50
150 Tramadol -
150 Codeine 50
15 Hydrocodone -
15 Morphine 5
10 Oxycodone -
3 Hydromorphone 1
2 Levorphanol 1
- Fentanyl 0.050 (50mcg)
131
3. Transdermal Fentanyl: (2:1 Rule)
Morphine 50mg PO over 24 hours = Fentanyl 25mcg transdermal patch
- Serum Fentanyl takes 12-24 hours before reaching plateau so may need to continue
to treat pain with other medication for at least first 12 hours.
- Do not use in opioid nave pts or for post-operative pain; useful in chronic stable pain.
- Heat & Fever increase absorption.
- Never cut a fentanyl patch in half.
- Remove patch when pt goes to MRI to avoid burns.
4. Methadone
Daily Morphine Dose
(mg/24hr PO)
Conversion Ratios
Morphine PO : Methadone PO
<100 3:1
101-300 5:1
301-600 10:1
601-800 12:1
801-1000 15:1
>1001 20:1
5. Side Effects
- Constipation be sure to place all patients on bowel regimen.
- N/V treat w/ compazine, usually improves in 24-48 hrs.
- Itching treat w/ benadryl, usually improves in 48-72 hrs. (Hives are an allergic
- reaction and med should be stopped)
- Myoclonus/Hallucinations these are signs/symptoms of opiate toxicity.
6. The Transition:
- It is recommended to initially use lower than equivalent doses when switching between
different opioids, especially in the elderly.
132
ENTERAL FORMULARY
Two Cal HN
High calorie: As well as high nitrogen, complete formula with
reduced sodiumfor hypermetabolic patient requiring severe fluid
restriction.
Jevity 1.2
High Fiber: Normalization of bowel function; isotonic, nutrient
dense, high nitrogen, low residue, lactose free.
Promote w/ Fiber
High Fiber & Protein: Nutritionally complete high nitrogen, nearly
isotonic liquid with fiber for patients with increased protein needs.
Pivot
or
Promote
High Protein: Nutritionally complete high nitrogen, nearly isotonic
liquid with fiber for patients with increased protein needs.
Nepro
Dialysis Patients: Nutritionally complete high nitrogen, nearly
isotonic liquid with fiber for patients with ESRD.
Nutrihep
ESLD Patients: Rich in branched-chain amino acids and low in
aromatic and ammonogenic amino acids. Calorically dense, with a
high calorie-to-nitrogen ratio for patients with ESLD.
Peptamen AF
Elemental nutrition: complete and balanced tube feeding for
hypermetabolic conditions requiring partially hydrolyzed protein
(free amino acids, di and tri-peptides), |-carotene and L-arginine
with MCT oil. Useful in Pancreatic Insufficiency.
Glucerna 1.2
Diabetic Patients: Calorically-dense formula with a unique
carbohydrate blend for enhanced glycemic control, and fish oil to
reduce glycemic response.
Ensure Plus
High calorie, lactose free, nutrient dense formula when limited
volume required.
May be used orally or as tube feeding.
Ensure
A concentrated high protein liquid food for patients requiring
additional calories, protein, vitamins and minerals.
May be used orally or as tube feeding.
133
PHONE NUMBERS
VAMC PHONE NUMBERS
Main (858) 552-8585
AOD 2366/2367
Chief residents
Inpatient 7622
Outpatient 2759
Cast tech 3841
p347-1391
Cath lab 3464
Coding room 3950
Computer Help 4767
Coumadin clinic 3041
CPRS
J. Christmas 7802
B. Henry 1033
Cytology 7729
DAV 7470
Decedent Affairs 7568
Diabetes Educ 7245
ECG ((M-F 7-4pm) 3644
EEG/EMG 3688/3685
ER 2366
Escort 3311/66-791
Exercise Treadmill 3550
Eye Clinic 2205
FIRM (4N) 2907
Heart Station 1127
Hemodialysis 7517
Home Health/HBPC 3064
Home health planning
3N L. Suchocki 2973
Lindas pgr p347-1837
5E P. Pelican 3094
Pats pgr p347-1818
Home Oxygen 3634
Labs:
Add-On 7707
ABG/Gas Lab 7264
Blood bank 3200
Chemistry 7750/7751
Cytology 7729
Hematology 3420
Immunology 2383
Micro 3367
Pathology 7723
Processing 7707
Serology 2383
MAOD 4344
Med Admit pager 290-7173
Medicine Office 3481
Mission Valley Firm 5051/5052
Nuclear Med 7511/4385
Nutrition 3373
OT 7672
PFT Lab 7352
Pharmacy :
Outpatient 1669
Inpatient
3N 2214
p347-1408
5E 5343
p347-1409
ICU/DOU 2377/5344
p347-1444
Phlebotomy 3668/7707
Physical Therapy 7487/3683
PICCs
Andrea/Tracy 7979
Myra Williams 2362
p347-1849
Podiatry 3981
Prosthetics 7415
Psychiatry p347-1427
Radiology: 3226
Main 1164
Body 7773
Bone Pit 7793, 7194
Chest 7772/7774
CT scanner 7796
Head 2850
IR 6903/7801
MRI 2860/7980
Neurorads 2850
Ultrasound 7771
Tech 6471
p347-1428
RT 3518
Security 3647
Sleep study 2494/4701
Social Work
3N Eric Ramirez 7391
4S Marita McGoldrick 2279
Or 858-699-3507 (cell)
5E Katherine Ponchetti 3329
Speech path 7564
Surg Path 7738
Travel Office 7575/5848
Transfer Office 7458
Urgent Care 3386
Vascular Lab 3290
Floors:
1E SCI 3185/3173
2S 3795
3N-Main 3446
3N-B Pod 7248/7647
3N-C Pod 7645/7648
3S-Infusion Center 3521
4S-Main 3564
5E 3295
5E-B Pod 5478/5479
5E-C Pod 5428
5S DOU 7979
5S VICU 5400
Fax Machines:
DOU (858) 642-3494
UCC/ER (858) 552-7515
FIRM (858) 642-6325
134
Main (619) 543-6222
ACLS/PALS 294-5838
Case management 33516
Cath Lab 37166
Chief residents
Inpatient 36297
Computer help 34357
PACS 36760
CMH 692-8200
ER 32140
Nurses 32154
ER fax 33122
4
th
& Lewis Clinic 294-9520
471-9250
Back lines: 19270/19271
Flow Cytometry (858) 642-4774
GME Office 37768
Heart Station 36399/36485
Echo 35715
Telemetry 32703
Fellows Rm. 37230
Hemodialysis 36871
Infusion Center 32639
Labs:
Blood gas 36536
Blood bank 35640
Chemistry 36020
Cytology 35378
Hematology 36020
Micro 35940/32820
Rheum 35773
Surg path 35764
Toxicology 36020
Virology 35797
Serology 35587, 32249
Medical Records 36700
Med Rec Fax 33287
Dictations 32720, 32867,
32936
Med Admit pager 290-1150
Notary public 33620
Nuclear medicine 36680, 31985,
31991
Nursing Supervisor p2616
Owen Clinic 33995/33999
Paging 36440
Pharmacy:
Discharge 33279/32682
Inpatient 35924
Outpatient 36191
CCU On-call p3535
Physical Therapy 36530
PPD/Skin Testing 35890
Peach 37719
Path Hot Seat 33243
Psych On-call 290-5050
Rads On-call p5063, 37043
Rads scheduling 33405/32280
Chest RR 37043/35389
CT scanner 36893, 36894
Body CT RR 32511
KUB 19476
Bone 35277
Head CT RR 32566
IR 32158, 32476,
32138
MRI 36148, 32944,
32940
MRI Body 33177
MRI Neuro 32931, 10759
File room 36586
Ultrasound 32620/32621
Risk management 36630/96468
RTAS 37140
San Ysidro Clinic 619-205-6306
Security 33762
Speech Pathology 220-7570
Transfer Coord 35709
Floors:
2S SICU 37428
5E BICU 36502
5W IMU 36080
533 36344
6E 32091
6E fax 36224
6W 32870
7IMU 19533/19553
8E 36380
8W 36305
9W MICU 35960
10E 36300
10W CCU 36592, 33135
11E 35280
11W 36450
Administrative Support Staff
MPF Bldg, 402 Dickinson St, Suite 380
Chief Res Office fax 37186
Residency Admin Offi fax 36529
Anne Bamrick 36287
abamrick@ucsd.edu
ERAS Coordinator, Payroll/Personnel
Specialist, Duty Hours/New Innovations
HILLCREST PHONE NUMBERS
135
Main (858) 657-7000
Case Managers 76063
Weekend p7578
Elyse (3E) p0423
76067
Irma (3W) p8078
76602
Bonnie (2E) p3987
76805
Vangie (2W) p3606
76815
Liz (ER) p9861
77224
CF Clinic 77073
Jessica Hill 77103, 76774
p7255
Code Blue 76111
Computer Help 3HELP
EEG 76080
ER 77600, 77660
Fax (2W) 76061
Flow Cytometry (858) 642-4774
Heart Station 78530/78533
Tech p3960
Infusion Center 76301
Lab: 76595
Blood Bank 76161
Chem 76169
Cytology 35378
Hematology 76165
Microbiology 76959
Other 76595
Med Admit pager p7268
Med Records 76906
Nursing Stations:
ICU (2 North) 76700
IMU/2E (200s) 76295, 76290
2W (250s) 76886
3W/BMT (350s) 76390
3E (300s) 76340
Nursing Supervisor p2670
Nutrition 76470
OR 76500
OT/PT 76590
Weekend PT p4924
Weekend OT p1814
Tracy (PT) p3986
Tammy (PT) p4196
Kelly (PT) p4172
Jorge (PT) p3980
Resie (OT) p3562
Linda (OT) p7590
PACU 76540
Pain Service 76035
Pathology 76595
Pharmacy (inpt) 600-2330
p3952
76679
Pgr 4142
Pharm (outpt) 78512
Phlebotomy 78690
PTE Office 77100
Maureen p5141
Pam p3863
Radiology
RTAS 37140
Day Tech p3997
Night Resident p5063
Angio 76657
CT Scanner 76667
Tech p3954
Fileroom 76646
IR 76657
MRI 76671
Tech 76674
Neuro CT 32566
Nuc Med 76659
Reading Rooms
CT Body 76779
Bone 76780
Chest 37043
Ultrasound 76778
Radiation Oncology 26040
Respiratory 76690
Pgr 4166, 4186
3972, 3585
Security Pgr 2660
Speech Path 220-7570
Pgr 4288
Social Work 76804
Pgr 4288
Telemetry 76703
Transfer Center 35709
Pgr 5673
Urgent Care 77300
Workrooms
2W 76886
76282
3W 76380
76382
3E 76340
Any phone number: PCIS
General Menu UCSD Phone
Book
THORNTON PHONE NUMBERS
136
OUTSIDE HOSPITALS
Scripps Mercy (Hillcrest)
Main Number: 619-294-8111
Med Records: 619-260-8972
Med Records Fax: 619-260-7369
Scripps Mercy Chula Vista
Main Number: 619-691-7000
Med Records: 619-691-7336
Med Records Fax: 619-407-7677
Scripps Memorial La Jolla
Main Number: 858-626-4123
Med Records: 858-626-6848
Med Records Fax: 858-626-6141
Scripps Green
Main Number: 858-554-9100
Med Records: 858-554-4757
Med Records Fax: 858-554-2540
Sharp Memorial
Main Number: 858-939-3400
Med Records: 858-939-3404
Med Records Fax: 858-939-3491
Sharp Grossmont Hospital
Main Number: 619-740-6000
Med Records: 619-740-3887
Med Records Fax: 619-740-4442
Sharp Chula Vista Hospital
Main Number: 619-482-5800
Med Records: 619-482-3642
Med Records Fax: 619-482-3483
Alvarado Hospital
Main Number: 619-287-3270
Med Records: 619-229-3175
Med Records Fax: 619-229-3397
Paradise Valley Hospital
Main Number: 619-470-4321
Med Records: 619-470-4205
Med Records Fax: 619-470-4102
Fallbrook Hospital
Main Number: 760-728-1191
Med Records: 760-731-8238
Med Records Fax: 760-728-0617
El Centro Regional Medical Center
Main Number: 760-339-7100
Pioneers Memorial Hospital
(Brawley)
Main Number: 760-351-3333
137
DOOR CODES/PASSWORDS
Hillcrest
- Copier code: Employee ID or 5-4-3-6-2-8-7
- BICU: 5-4-3
- CCU: 6-1-9-*
- ER: 0-3-5-*
- ER side door: 5-3-2-Enter
- UCC: 1-5
- 10E workroom: 3-5
- Medicine office: 3-2-4-Enter
- CCU Office: 3-2-4-Enter
- Closet combo 3-1-5-3
- PACU/OR 4-0-0-0-*
- New PACU 7-7-8-8
- Senior Behavioral Health: 8-6-3-1
- SICU: 6-4-2
- Clean Utility (most floors): 2-5-Enter
- Storage Rooms (on ends): 3-5-1-Enter
- Cath lab: 1-6-3-*
- 4
th
& Lewis Clinic: 9-1-1-9-1
- Owens Clinic: 8-5-4-*
Thornton
- ER: 7-6-0-*
- 2
nd
Floor Pharmacy: 6-0-1-0-#
- ICU: 6-0-3-8
- BMT Office: 6-3-9-0
VAMC
- ER/UCC: 9-8-7-4
- Area 2: 7-9-2
- 3E call rooms (Main door): Check with VA chief at start of rotation
- 3E call rooms 3288, -89, -97: 3-2-8-7
- Team 3 work room: 3-2-1-4
- 3W access clinic: 3-0-3-2
- 5S VICU call rooms 5101: 3-0-0-4
- Supply rooms: 1-7-4-7
- Main medicine office: 3-0-1-8
- Conf Room 3004: 1-3-7-9-5-5
- 3S Lounge 3207B: 3-2-0-8
- MV Firm: 4+5-2
- MV Firm Copier Code: 1-2-0-4
- Hemodialysis Unit: 1-8-0-7
PASSWORDS
MUSE ECGs: Username: ad/emergency
Password: Emer!gency
MyFax http://www.myfax.com
Username: 888-569-7870 (same # as the fax)
Password: ucsdmedres2

You might also like